Sunteți pe pagina 1din 109

Universidad La Salle. Facultad Mexicana de Medicina.

Curso de Extensin Universitaria para la Preparacin del Examen Nacional para Aspirantes a Residencias Mdicas. Examen Final 1 Septiembre 2010. Modalidad a distancia.

1.- La profilaxis de una neumona por Pneumocystis en un paciente de cinco aos de edad, se encuentra indicada cuando los linfocitos ajustados para la edad son de:

a) b) c) d)

< 1500 Clulas/l < 750 Clulas/l < 500 Clulas/l < 200 Clulas/l

La NPC en nios y en el embarazo. La NPC es infeccin diagnstica de SIDA en un alto porcentaje de nios, sobre todo en el primer ao de vida. Los nios menores de 1 ao con CD4+ por debajo de 1500// l tienen un 90% de riesgo de padecerla. Las manifestaciones clnicas, el diagnstico y el tratamiento no difieren de los del adulto. Para la prevencin deben seguirse las siguientes recomendaciones: Se aconseja la profilaxis primaria en todos los nios menores de 1 ao hijos de madre VIH+ La profilaxis debera comenzar a las 4-6 semanas de vida y suspenderse en caso de que posteriormente no resulten estar infectados. Los nios infectados y aquellos cuyo estado de infeccin por VIH se desconozca debern seguir recibiendo tratamiento profilctico durante el primer ao de vida. En los mayores de esta edad se considerar continuar con la profilaxis atendiendo al recuento de linfocitos CD4+ segn la edad: de 1 a 5 aos, cuando el recuento sea < 500 clulas/ l o el porcentaje < 15%; en nios de 6 a 12 aos, cuando el recuento sea inferior a 200 clulas/ l o el porcentaje < 15%.

No se ha estudiado la seguridad de interrumpir la profilaxis en nios infectados con el VIH y sometidos a tratamiento antirretroviral. Los nios con un historial que incluya episodios de NPC debern ser tratados de por vida con quimioprofilaxis para evitar casos de recurrencia.

NCP en el embarazo Como ya se ha dicho, la quimioprofilaxis anti-NPC debe administrarse a las mujeres embarazadas al igual que al resto de adultos y adolescentes. En este caso, el agente profilctico recomendado sigue siendo el cotrimoxazol, con dapsona como alternativa. Debido a la posibilidad terica de una posible teratogenicidad asociada a la exposicin a los frmacos durante el primer trimestre de gestacin, durante dicho perodo puede considerarse la alternativa de pentamidina en aerosol debido a que dicho agente no se absorbe sistmicamente por lo que el feto en desarrollo no sufre exposicin al frmaco. Lectura recomendada: Neumona por Pneumocystis carinii en nios infectados por inmunodeficiencia humana (VIH). Gac Med Mex 2004; 140 (1): 59-70.

el

virus

de

2.- El Tratamiento de eleccin para la Isosporosise es:

a) b) c) d)

Trimetoprim Sulfametoxazol. Albendazol. Metronidazol. Pentamidina.

El tratamiento ms efectivo contra la isosporiasis consiste en una combinacin de dos medicamentos: trimethoprim y sulfamethoxazole (TMP-SMX, Bactrim, Septra). Para tratar la isosporiasis, se toman dos pastillas de TMP-SMX de doble potencia dos veces al da. Otra posibilidad es tomar una pastilla de doble potencia tres veces al da. El tratamiento con TMP-SMX suele continuarse durante dos a cuatro semanas.

Referencia Bibliogrfica: Ttulo Del Libro: Gua De Antimicrobianos, Antivirales, Antiparasitarios, Antimicticos E Inmunomod. Autor Principal: Dr. Napolen Gonzlez Saldaa Editorial: Mc Graw Hill Interamericana. Edicin: 6. Ao: 2004 Pginas 257-278.

3.- Un estudiante es tacleado mientras jugaba football y desarrollo dolor en rodilla severo. A la EF la rodilla se encuentra edematizada y el paciente tiene dolor a la palpacin directa del aspecto lateral de la rodilla. Cuando se flexiona la rodilla 30, la aduccin pasiva ocasiona dolor en la misma rea, y la pierna puede ser aducida ms que la pierna

contralateral. Cajon anterior, cajn posterior y Lachman negativos. Cul es el sitio ms probable de lesin?

a) b) c) d)

Ligamento cruzado anterior. Ligamento colateral lateral. Menisco lateral. Ligamento cruzado posterior.

Se produce por un trauma en varo al golpearle en la parte externa de la pierna por debajo de la rodilla o al caerle otro jugador o contusin en la cara interna de la rodilla. Presenta dolor y generalmente sensacin de parestesia en el territorio del nervio citico poplteo externo por distensin, que puede ser una parestesia transitoria en traumas de menor energa o una lesin definitiva con pie cado en traumas de alta energa. El paciente presenta dolor, derrame leve a moderado y equimosis en cara lateral de la rodilla. El diagnostico se hace al encontrar: 1. Bostezo en varo en 30 grados de flexin: para evaluar el ligamento colateral lateral, el dolor es en trayecto del ligamento. Puede ser de diferente magnitud: Grado I: muy discreta apertura de la interlinea comparado con lado contra lateral. Grado II: apertura de 5 mm de la interlinea articular. Grado III: apertura de 10 mm de la interlinea articular.

2. Bostezo en varo en extensin completa: para evaluar esquina postero lateral, capsula postero lateral y ligamento fibulo poplteo. Las estructuras laterales tienen un potencial de cicatrizacin menor que las estructuras mediales. En las lesiones grado I y II se debe inmovilizar y proteger el apoyo durante seis semanas. Se debe tener un alto ndice de sospecha y en los casos grado III y donde se presente compromiso de la capsula postero lateral, se debe realizar ciruga para reparo primario en las primeras dos semanas, pues el reparo primario funciona mejor que cualquier reconstruccin anatmica.

4.- Cul es la trada clnica clsica de la rotura espontnea del esfago o sndrome de Boerhaave?

a) Nuseas, vmitos y fiebre. b) Vmitos, dolor torcico y enfisema subcutneo. c) Fiebre, dolor abdominal y hematemesis. d) Dolor torcico, dolor abdominal y vmito.

Definicin: ruptura espontnea del esfago que ocurre sobre todo como consecuencia de un violento vmito. Este sndrome fue descrito por primera vez en 1724 por Herman Boerhaave. Se diferencia del sndrome de Mallory-Weiss en que en este caso se tratada de una perforacin transmural mientas que el sndrome de Mallory-Weiss es tan solo una laceracin. Dado que frecuentemente est asociado al vmito, no puede decirse que en realidad se trate de una ruptura espontnea. Hay que distinguirlo de la perforacin yatrognica que supone el 85-90% de los casos de rupturas esofgicas. Casi siempre en el lado izquierdo del tercio inferior esofgico a 2 o 3 cm de la unin gastroesofgica. A igual que el sndrome de Mallory-Weiss, se ha relacionado con los aumentos bruscos de la presin intrabdominal y con la hernia de hiato. Es ms frecuentemente observado en pacientes alcohlicos o que abusan de la comida. Se trata de un sndrome relativamente raro pero con un alto ndice de mortalidad (35%). De hecho, es considerada como la ms letal de todas las perforaciones del tracto digestivo. Los sntomas son vmitos, dolor torcico y enfisema subcutneo cervical.

REFERENCIAS C. Yeo. Shackelford' surgery of the Alimentary Tract, 2007. Saunders, 2700 pginas Khan, Aamir Z.; Forshaw, Mathew J.; Davies, Andrew R.; Youngstein, Taryn; Mason, Robert C.; Botha, Abraham J. Transabdominal Approach for Management of Boerhaave's Syndrome . American Surgeon , May2007, Vol. 73 Issue 5, p511-513, Hill, Andrew G.; Tiu, Albert T.; Martin, Iain G. Boerhaave's syndrome : 10 years experience and review of the literature: H. ANZ Journal of Surgery , Dec2003, Vol. 73 Issue 12, p1008-1010, Ochiai, T.; Hiranuma, S.; Takiguchi, N.; Ito, K.; Maruyama, M.; Nagahama, T.; Kawano, T.; Nagai, K.; Nishikage, T.; Noguchi, N.; Takamatsu, S.; Kawamura, T.; Teramoto, K.; Iwai, T.; Arii, S. Treatment strategy for Boerhaave's syndrome.. Diseases of the Esophagus, May2004, Vol. 17 Issue 1, p98-103.

5.- Ante un paciente portador de lcera gstrica Cul de las siguientes es la indicacin quirrgica? a) Sangrado de tubo digestivo alto con inestabilidad hemodinmica. b) Menor de 60 aos. c) Biopsia negativa a tumor maligno. d) La lcera no cicatriza despus de 12 semanas de tratamiento bien llevado por el paciente.

lcera Pptica: 50-75 % de casos. La hemorragia por lcera duodenal es 4 veces ms frecuente que la lcera gstrica, pero ambas tienen la misma tendencia a la hemorragia. La Hemorragia Masiva tiene una frecuencia del 10-15% y generalmente es producida por horadacin por el proceso inflamatorio de la arteria regional. * a) b) c) Localizacin: Bulbo duodenal. Curvatura menor. Zona prepilrica.

De ellas, la curvatura menor hace las hemorragias masivas ms frecuentes, pero la duodenal, en general, es la ms frecuente.

La relacin entre la localizacin de la lcera y la gravedad de la hemorragia se debe a la lesin de las arterias principales, de la curvatura menor, la coronaria estomquica, la pilrica y la gastroduodenal. TRATAMIENTO DE LA HEMORRAGIA DIGESTIVA ALTA. Es una emergencia Mdico-Quirrgica, de gran importancia y frecuencia, cuya morbilidad y mortalidad estn influenciadas por un manejo oportuno, coherente, en el que necesariamente concurren varios especialistas, en lo posible organizados en equipo. Es evidente que el manejo en equipo y la identificacin de los pacientes de alto riesgo se convierten en las armas ms importantes en el manejo actual de esta patologa.

TRATAMIENTO QUIRURGICO:

El momento de la indicacin y la tcnica a realizar son los pilares principales de una desicin correcta. Las tcnicas a realizar sern de acuerdo a la experiencia del cirujano y el riesgo quirrgico del enfermo. Tales como: Vagotoma troncular piloroplasta, es ms rpida y menos traumtica, elimina factor vagal, baja mortalidad postoperatoria, pero alto ndice de recidiva del sangrado.

Vagotoma ms Gastrectoma 4/5 elimina todo el factor sangrante, usada en fracaso de otras tcnicas.

Gastrectoma total: en casos especiales. Estado general del paciente, edad, obesidad, repeticin de sangrado indican la tcnica a usar.

En lcera Duodenal la Vagotoma ms piloroplasta y sutura de la lcera es lo ms adecuado. En lcera Gstrica, Gastrectoma y Sutura de la lcera: Gastrectoma subtotal Bilroht I, reseccin en cua de lcera. Vrices esofgicas: sonda de Sengstaken o de Linton. Tcnica de desconeccin portoacigos o Tanner modificada.

Lectura recomendada: Yamada Manual de gastroenterologia pagina 337.

6.- El agente causal ms frecuentemente encontrado como causa de Nocardiosis es: a) b) c) d) Nocardia Brasiliensis. Nocardia Asteroides. Nocardia Farcinica. Nocardia Otitidis Cavarum.

La nocardiosis es una enfermedad infecciosa relativamente poco comn, con una frecuencia anuale stimada entre 500 y 1000 casos por ao en los Estados Unidos de Amrica. Las dos especies ms prevalentes son Nocardia asteroides y N. brasiliensis (10) y pueden causar un amplio espectro de manifestaciones clnicas localizadas y diseminadas. El complejo N. asteroides ha sido considerado el responsable de la mayora de las infecciones invasivas severas tales como neumona y abscesos cerebrales Post-traumatismo, diseminndose

frecuentemente a otros sitios en pacientes inmunocomprometidos (3, 8). En Contraste, N. brasiliensis est generalmente asociada a infecciones de piel y partes blandas, comunicndose pocos casos de infecciones disemina- das (6, 8). Aqu se describe el caso de una nia de 4 aos de edad previamente sana, que present linfadenitis necrotizante postraumtica causada por N. asteroides y que evolucion satisfactoriamente con cefalotina.

Referencia Bibliogrfica: Ttulo Del Libro: Micologa Mdica Bsica Autor Principal: Dr. Alejandro Bonifaz Editorial: Mndez Cervantes.Edicin: 1. Ao: 1991, pginas 389-395.

7.- Masculino portador de una Micosis producida por una levadura capsulada que afecta pulmn y sistema nervioso, La patologa de este paciente es? a) b) c) d) Micetoma. Histoplasmosis. Criptococosis. Criptosporidiosis.

La criptococosis es una enfermedad sistmica causada por una levadura encapsulada de distribucin cosmopolita, Cryptococcus neoformans, cuyo estado perfecto es el Basidiomycete, Filobasidiella neoformans que tiene dos variedades: neoformans y bacillispora. Se han reportado cinco serotipos y dos variedades biolgicamente distintas: C.

neoformans variedades neoformans (A, B y AD) y gattii (B y C)1,3. Crece a 25 y a 37 en

agar-Saboraud y agar-sangre, y se encuentra en el suelo, vegetales descompuestos, excremento de aves y murcilagos. El agente es capaz de sobrevivir varios aos En ambiente oscuro y hmedo, y puede transportarse por el polvo. La enfermedad se adquiere por inhalacin de esporas, afecta en primer lugar al pulmn y se disemina luego por va hematgena afectando otros rganos y sistemas1. Arenas R. Micologa mdica ilustrada 2 edicin. Mc Graw Hill 2003;205-211.

8.- Masculino de 40 aos recin llegado del estado de Chiapas, Mx. Acude a consulta por referir presentar fiebre elevada de tres das de tres das de evolucin refiere artralgias francas y cefalea intensa. El mismo da de su visita a urgencias comenz a presentar un exantema maculo-papuloso pruriginoso. El examen de la sangre mostr los siguientes datos: Valor hematocrito 38%, Leucocitos 3600 p.mm3 con 82% neutrfilos y 12% linfocitos. Plaquetas 115.000 p.mm3. Placa de trax normal. El paciente fue dado de alta con el diagnstico de cuadro viral, en tratamiento con paracetamol y antihistamnicos para el picor.

A las 48 horas volvi a urgencias con muy mal aspecto: estaba afebril, tena confusin mental, se observaban petequias en antebrazos y piernas, edema en pies, TA 85/70. Pulso 110 l.p.min. de amplitud pequeo. En los nuevos exmenes de la sangre destacaban: Valor hematocrito 46%, leucocitos 3600 p.mm3 sin cambios en la frmula y plaquetas 65.000 p.mm3. Glucosa 106 mg.p. dl. Creatinina 1,8 mg.p.dl. Sodio 126 mEq/l.Potasio 4,2 mEq/l. La placa de trax mostraba un pequeo derrame pleural bilateral. Cul es, entre los siguientes, el diagnstico ms probable?: a) Dengue. b) Meningoencefalitis bacteriana. c) Fiebre tifoidea. d) Neumona por Legionella Neumophila.

El dengue es actualmente la ms importante arbovirosis que afecta al hombre. Su agente etiolgico son los 4 serotipos del virus del dengue (D1-4). Se transmite entre humanos mediante la picadura del mosquito Aedes aegypti. Se estima que el 40% de la poblacin mundial vive en reas de riesgo de esta entidad (1,2).

La forma clnica ms grave, la fiebre hemorrgica de dengue/sndrome de choque del dengue (FHD/SCD), fue casi exclusiva del Sudeste Asitico y el Pacfico Occidental hasta 1981, cuando una gran epidemia de dengue, la mayor de las Amricas, que incluy ms de 10.000 casos de FDH/SCD, ocurri en Cuba. Esta constituy la primera epidemia de DH en la regin (3). Posterior a este brote y hasta la actualidad la FHD contina presentndose de forma endmica en diferentes pases, ocurriendo epidemias de forma frecuente (4). Cuadro clnico El curso de la enfermedad, desde el punto de vista clnico, se puede dividir en cuatro fases: Inicial, crtica, de recuperacin y de convalecencia. Fase inicial: En ella el enfermo tiene un sndrome febril sin localizacin. Es muy sintomtica y adems de la fiebre predominan las manifestaciones generales como cefalea, artromialgias, dolor retroocular y malestar, puede aparecer rash. Suele durar alrededor de 3 das, tras lo cual la fiebre cede y algunos casos de dengue clsico comienzan a presentar manifestaciones hemorrgicas leves, pero la mayora tienden a mejorar. Otro grupo menor de pacientes desarrollar la FHD/SCD. Los llamados signos de alarma que preceden al choque pueden comenzar en esta fase. Fase crtica: Transcurre entre el 4to y 7mo da de la enfermedad, en ella se presentan los sntomas que definen al DH, como son la extravasacin de plasma, la trombocitopenia, y las manifestaciones hemorrgicas. Algunos pacientes desarrollan el sndrome de choque por dengue que es la forma ms severa de la enfermedad. Las manifestaciones hemorrgicas pueden ser tan leves que solo se hacen evidentes a travs de la prueba del torniquete, o tan severas como sangramientos digestivos graves con compromiso hemodinmico.

Los signos de extravasacin de lquidos (derrame en serosas, hemoconcentracin) son indispensables para hablar de FHD/SCD, ya que es la fuga de lquido la que casi siempre lleva el paciente al choque y no las hemorragias. Este hecho es de gran importancia pues influenciado por el nombre de fiebre hemorrgica de dengue, en no pocas ocasiones los mdicos asistentes esperan las grandes hemorragias que nunca llegan y el paciente cae en choque por la extravasacin de plasma.

Fase de recuperacin: Se inicia cuando cesa el escape de lquido y las manifestaciones de sangrado comienzan a disminuir. Pueden aparecer edemas o agravarse los derrames serosos producto de la sobre hidratacin. Tambin se puede apreciar un rash tardo asociado a prurito intenso. Se recuperan el apetito y el nmero de plaquetas. Fase de convalecencia: Puede prolongarse hasta ms de 6 meses, se caracteriza por cefalea discreta, cansancio fcil y artromialgias. No se presenta en la totalidad de los casos. Resumimos los signos de alarma de la siguiente manera: 1. Signos clnicos tempranos del choque. Cada brusca de la fiebre. Dolor abdominal intenso. Irritabilidad, somnolencia u otras alteraciones mentales. Fatiga extrema. Lipotimias. Dolor torcico. 2. Signos clnicos que agravan el choque. Vmitos frecuentes. Diarreas frecuentes. 3. Signos de laboratorio e imagenolgicos. Aumento progresivo del hematcrito y disminucin progresiva del conteo de plaquetas. Engrosamiento de la pared de la vescula biliar.

Referencias 1. Guzman MG, Kouri G. Dengue-an update. The Lancet Inf Dis 2002;2:33-42. 2. Gubler DJ, Clark CG. Dengue/dengue hemorrhagic fever: the emergence of a global health problem. Emerging Infectious Diseases. Atlanta USA. 1995;1:55-57. 3. Kour GP, Guzmn MG, Bravo JR, Triana C. Dengue haemorrhagic fever/ dengue shock syndrome: lessons from the Cuban epidemic. 1981. Bull World Health Organ 1989;87:37580. 4. Organizacin Panamericana de la Salud. 2003: Number of reported cases of dengue and

dengue hemorrhagic fever (DHF), region of the Americas (by country and subregion). [citado del 25 de agosto de 2003]. 5. Valdes L, Guzman MG, Kouri G, Delgado J, Carbonell I, Cabrera MV, Rosario D, Vazquez S. La Epidemiologa del Dengue en Cuba en 1997. Rev Panam Salud Publica/Pan American Journal of Public Health 1999;6:16-25. 6. Pelaez O et al. Havana dengue 3 epidemic, 2001. Enviada a Emerging Infection Diseases. 7. George R, Lum LCS. Clinical spectrum of dengue infection. In: Gubler DJ, Kuno G, eds. Dengue and dengue hemorrhagic fever. London: CAB International, UK, 1997; 89-113. 8. Kour G, Guzmn MG, Bravo J. Why dengue haemorrhagic fever in Cuba? 2: An integral analysis. Trans R Soc Trop Med Hyg 1987;81:821-23.

9.- Femenino de 35 aos con antecedentes de trasplante renal hace mes y medio, acude al servicio por referir fiebre desde hace ms de tres das, dolor en epigastrio as como cansancio leve.. En la analtica practicada destaca una moderada leucopenia (2400/mm3) con una leve elevacin en la cifra de transaminasas (ALT 75 UI/l; AST 89 Ul/l) Cul sera el primer diagnstico de sospecha?

a) Tuberculosis pulmonar. b) Infeccin por Helicobacter pilorii. c) Infeccin por Citomegalovirus. d) Hepatitis por VHC.

El Citomegalovirus (CMV) es una infeccin oportunista. Aproximadamente el 50 por ciento de la poblacin general y el 90 por ciento de las personas VIH positivas son portadoras del CMV. En general, un sistema inmune sano puede mantener la infeccin bajo control. Pero cuando el VIH o cualquier otra enfermedad debilitan al sistema inmune, el CMV puede atacar varias partes del cuerpo. Causas y Factores de Riesgo: La transmisin del CMV puede ser: Fetal, perinatal a travs de secreciones cervicales y por la leche materna. Posteriormente por va respiratoria, sexual (semen y secreciones cervicales), transfusiones de sangre, trasplante de rganos. Los individuos que particularmente tienen mayor riesgo de adquirir una infeccin seria por CMV son: 1. Pacientes infectados con el VIH, especialmente aquellos que desarrollan SIDA. 2. Pacientes que han recibido un trasplante de mdula sea de un rgano slido (Hgado, Rin, Corazn). 3. Pacientes con Cncer, en especial aquellos que involucran las clulas sanguneas. 4. Por transmisin Intrauterina de CMV.

Las infecciones adquiridas despus del nacimiento o en la fase adelantada de la vida, frecuentemente son asintomticas. Una enfermedad febril aguda, denominada mononucleosis por citomegalovirus o bien hepatitis por citomegalovirus, puede depender del contacto iatrgeno o espontneo con el CMV. Por su parte, el sndrome posperfusin se presenta de 2 a 4 semanas despus de una transfusin con sangre fresca que contenga CMV, y se caracteriza por fiebre que dura de 2 a 3 semanas, hepatitis de intensidad variable, con ictericia o sin ella, linfocitosis atpica con caracterstica similar a la de la mononucleosis infecciosa y, en ocasiones, una erupcin.12,24 Entre los episodios febriles, el paciente, aunque cansado, no se siente muy enfermo.23,25 Las infecciones por CMV son frecuentes en todas las formas de aloinjertos, y puede causar neumonitis intersticial, hepatitis, encefalitis, retinitis y enfermedad difusa de inclusin citomeglica. Se ha detectado en homosexuales y en pacientes con el sndrome de inmunodeficiencia adquirida (SIDA). De acuerdo con esto, gran parte de la morbilidad y cierta mortalidad relacionadas con el SIDA se atribuyen a infecciones por CMV de hgado, cerebro (con ndulos gliales), aparato digestivo (lesiones ulcerosas) pulmones, pneumonitis intersticial difusa que coexiste con frecuencia con la infeccin por Pneumocystis carinii, y en los ojos (retinitis). Se considera que desempea alguna funcin en la aterogenesis.10 Crumpacker CS, Wadhwa S. Cytomegalovirus. In: Mandell GL, Bennett JE, Dolin R, eds. Livingstone; 2005: chap 134.

Principles and Practice of Infectious Diseases. 6th ed. Philadelphia, Pa: Elsevier Churchill

10.- Masculino de 77 aos portador de virus C y diabtico desde hace 10 aos, que presenta lesiones ampollosas de contenido serohemorrgico en extremidades superiores e inferiores, y erosiones a nivel de la mucosa bucal. La biopsia cutnea muestra una ampolla subepidrmica, se realiza inmunofluorescencia directa que muestra la presencia de depsitos de IgG a nivel de la membrana basal. El diagnstico ms probable es:

a) Dermatitis herpetiforme. b) Penfigoide ampolloso. c) Toxicodermia ampollosa. d) Porfiria cutnea tarda.

Penfigoide ampolloso Casi siempre afecta a mayores de 60 aos. Se trata de una enfermedad ampollosa crnica autoinmune causada por la unin de autoanticuerpos tipo IgG y complemento C3 al antgeno del penfigoide, una glucoproteina transmembrana de 180 (antgeno menor) o 230 kD (antgeno mayor, desmoplaquina I) situada en los hemidesmosomas del queratinocito basal, a nivel de la lmina lcida. Aunque se haba dicho que el penfigoide poda ser paraneoplsico, existen estudios epidemiolgicos que demuestran que el incremento de carcinomas

asociados se debe unicamente a que ambas patologas, el penfigoide y el cncer, se dan con mayor frecuencia al mismo grupo de edad.

Patognesis Se cree que la unin de los auto anticuerpos IgG al antgeno del penfigoide produce activacin del complemento, lo que da lugar a la sntesis de C3a y C5a que causan desgranulacin de los mastocitos. stos a su vez liberan mediadores entre los que destaca el factor quimiotctico de los eosinfilos. Los eosinfilos reclutados liberan enzimas lisosomales que destruyen los hemidesmosomas y los filamentos de anclaje con lo que se separa la unin dermoepidrmica en la lmina lcida. Clnica: Se caracteriza por lesiones ampollosas grandes y tensas de contenido seroso o serohemtico, localizadas o generalizadas, con distribucin preferente en flexuras y que curan sin dejar cicatriz residual. La aparicin de ampollas puede ir precedida de un cuadro urticarial o eczematoso. Pueden ser o no pruriginosas. En cerca del 30% de los casos existe afectacin mucosa (generalmente mucosa oral).

Penfigoide ampolloso Diagnstico El diagnstico se establece por criterios clnicos, histolgicos e inmunopatolgicos. En el estudio histolgico de la biopsia cutnea de una ampolla se observa una hendidura subepidrmica en el interior de la cual se aprecia un rico infiltrado inflamatorio mixto con predominio de eosinfilos que tambin abundan en la dermis. Mediante inmunofluorescencia directa se comprueba la existencia de depsitos lineales de IgG y/o C3 en la regin de la membrana basal. La inmunofluorescencia indirecta, utilizando fluido de la ampolla u orina, pone de relieve en cerca de la mitad de los pacientes la existencia de IgG y/o C3 circulantes.

Bibliografa 1. Worjnarowska F, Kirtschig G, Highet AS, Venning VA, Khumalo NP. Guidelines for the management of bullous pemphigoid. Br J Dermatol 2002; 147:214-221. 2. Downham TF, Chapel TA. Bullous pemphigoid. Arch Dermatol 1978; 114:1639-1642. 3. Chosidow O, Saas V, Diquet B. Correlation between the pretreatment number of blisters and the time to control bullous pemphigoid with Prednisone 1mg/Kg/day. Br J Dermatol 1992; 127:185-195. 4. Fiveson D, Breneman D, Rosen G et al. Nicotinamide and tetracycline therapy of bullous pemphigoid. Arch Dermatol 1994; 130:753-758. 5. Fleming TE, Korman NJ. Cicatricial pemphigoid. J Am acad Dermatol 2000; 43:571-591. 6. Eisen D, Ellis CN, Voorhees JJ. Topical Cyclosporine for oral bullous disorders. J Am Acad Dermatol 1990; 23:936-937.

11. - Is the most common genetic feature in ankylosing spoondylitis a) b) c) d) BRCA-1. HLA B27. HNF 4alfa. K-ras de novo mutation.

La espondilitis anquilosante es un padecimiento reumtico inflamatorio, generalizado y crnico, que afecta primordialmente el esqueleto axial, con la presencia de dao de las articulaciones S-I (sacroiliitis) como su hallazgo fundamental Races griegas: ankilos (inclinado-fusin), espondilos (vrtebra) PREVALENCIA: Africanos y esquimales: 0.1 %. Blancos: 0.5 1 %. Indios Haida (Norte de Canad): 6 % PREVALENCIA DEL HLA-B27: 6 8 % PREVALENCIA EN FAMILIARES DE ENFERMOS, HLA-B27 +: 10 20 % INCIDENCIA: 6.3 6.9/100 habitantes por ao. CONCORDANCIA en gemelos: Monocigotos, 63 %; dicigotos, 12.5 % EDAD: Se inicia en la adolescencia o la edad adulta temprana. Rara despus de los 40 aos.

GNERO: 3 5:1 a favor del hombre.

FACTORES GENTICOS: Antgenos del CPH: HLA-B27. HLA-B60. HLA-DRB1. TNF. Otros de clase III: MICA, TAP, LMP2, LMP7, HSP70, Antgenos fuera del CPH:

complotipos.

- Regiones en los cromosomas 1, 2, 6, 9, 10, 16 y 19; 16q. FACTORES AMBIENTALES. ALTERACIONES INMUNOLGICAS.

Brown MA, Crane A, Wordswoth BP. Curr Opin Rheumatol 2003;14:354-60.

12.- Un hombre de 50 aos acude al servicio de urgencias por presentar fiebre de 39.0c, exantema mculo-papuloso generalizado, incluyendo palmas y plantas. El paciente labora en el campo ordeando vacas frecuentemente parasitadas por garrapatas. Seale la enfermedad a la que se refiere, el germen causante y el tratamiento adecuado:

a) Kala-azar, Leishmaniae Donovani: Antimoniales. b) Fiebre Q, Coxiella Burnetti, Doxiciclina c) Fiebre botonosa, Ricckettsia Conori: Doxiciclina. d) Fiebre de Malta, Brucella Mellitensis: Cotrimoxazol.

Las rickettsias son organismos coco-bacilares, Gram negativos, de 2-3 micras de dimetro, intracelulares, incapaces de crecer en ausencia de clulas vivas del husped. El gnero Rickettsia es ubicado taxonmicamente en la familia Rickettsiaceae, junto a otros dos gneros: Coxiella, con la especie C.burnetii, responsable de la fiebre Q, y Ehrlichia con las especies E.chaffeensis, agente de la ehrlichiasis monoctica, y E.phagocytophila, productora de la ehrlichiasis granuloctica humana.

Junto a la familia Rickettsiaceae est la familia Bartonellaceae con tres especies principales: Bartonella henselae, agente de la enfermedad por araazo de gato (cat scratch

disease); B.quintana, responsable de la angiomatosis bacilar(1), y B.bacilliformis, productora de la bartonelosis o enfermedad de Carrin (verruga peruana). Las rickettsiosis son zoonosis transmitidas desde los huspedes o reservorios animales al hombre a travs de picaduras de artrpodos diversos, que varan con cada enfermedad (piojos, pulgas, garrapatas, otros caros, esencialmente). La rickettsiosis por R.conorii es conocida con el nombre de fiebre botonosa o manchada del Mediterrneo o fiebre de Marsella, siendo transmitida al hombre desde el perro que constituye su reservorio por garrapatas de los gneros Amblyomma y Riphicephalus principalmente. En ellas el germen cumple un ciclo que incluye el pasaje transovrico a los descendientes por lo cual representan tambin otro verdadero reservorio del parsito. Clnicamente la enfermedad se caracteriza por la aparicin en el sitio de la picadura de la garrapata de una lesin inicial indurada con centro necrtico muchas veces (mancha negra o tache noir) rodeada de aureola inflamatoria, seguida de adenopatas regionales de carcter inflamatorio en los das subsiguientes. Concomitantemente, fiebre frecuentemente alta de 39-40C, malestar general, cefaleas a veces intensas, dolores musculares y articulares. Es relativamente frecuente la observacin de un exantema mculo-papuloso que explica el nombre de fiebre botonosa y que puede afectar varios territorios. Es una afeccin endmica en Sudfrica, Europa del Sur y Medio Este (2). El diagnstico se confirma esencialmente por la tcnica de inmunofluorescencia indirecta (IFI) empleando lminas que contienen antgenos de R.conorii y utilizando, siempre que sea posible, sueros pareados obtenidos al inicio del cuadro y 20-30 das despus para investigar la seroconversin. La histopatologa de las lesiones iniciales o taches noires fue estudiada en detalle por Montenegro y colaboradores en 1983(3). En otro trabajo posterior, Montenegro y colaboradores (4) demuestran en ratones inoculados con R.conorii la importancia crucial de la inmunidad celular con respecto a la humoral en el control de la infeccin experimental y reduccin del ndice de mortalidad. El ratamiento de eleccin es dicloxacilina, alternativas macrlidos y quinolonas (ciprofloxacino). Bibliografia: 1 . Sampaio SAP, Rivitti EA. Dermatologia. So Paulo: Artes Mdicas, 1998: 1155. 2 . Harris RL, Kaplan SL, Bradshaw MW, Williams Jr, Temple W. Boutonneuse fever in american travelers. J Infect Dis 1986; 153:126-8. 3 . Montenegro MR, Mansueto S, Hegarty BC, Walker DH. The histology of taches noires of boutonneuse fever and demonstration of Rickettsia conorii in them by immunofluorescence. Virchows Arch (Pathol Anat) 1983; 400:309-17. 4 . Montenegro MR, Walker DH, Hegarty BC. Infection of genetically immunodeficient mice with Rickettsia conorii . Acta Virol 1984; 28:508-14. 5. Conti Daz IA, Rubio I, Somma Moreira RE, Prez Bormida G. Rickettsiiosis cutneo-ganglionar por Rickettsiaconorii en el Uruguay. Rev Inst Med Trop (So Paulo), 1990.

13.- En una consulta prenatal de rutina, una mujer de 28 aos de edad, G5 P4, con 28 SDG, refiere que no ha sentido movimiento fetal durante los ltimos 2 das. Su embarazo ha sido complicado debido a que padece hipertensin crnica, para lo cual se le recetaron tabletas de alfa-metildopa 2 veces al da. Al examen, su FU es de 30cm, y las maniobras de Leopold demuestran que el feto se encuentra en situacin transversa. Su TA es 145/85mmHg. No se encuentra latido cardiaco con el Doppler. Cul de los siguientes es el paso ms apropiado a seguir en el manejo?

a) b) c) d)

Realizar un test sin estrs. Amniocentesis. USG. Beta-HCG (cuantitativa)

Probable bito: Sintomatologa y diagnstico Signos funcionales: No se perciben movimientos fetales por 12-24 horas. Disminucin o ausencia de sntomas y/o signos como nauseas vmito, hipertensin, albuminuria) Paraclnicos: *ecografa: diagnstico precoz y exacto: Doppler. *Radiologa: hay 3 signos: +deformacin del crneo +curvatura y torsin de la columna +presencia de gas en el feto *lquido amnitico: puede estar meconiado, o sanguinolento Signos locales: en los senos hay secrecin calostral, sangrado leve y oscuro por vagina, el feto se vuelve blando a la palpacin, fetocardia (-), puede haber detencin y/o disminucin de la altura uterina, bajo peso corporal, entre otros... Reduccin de la perfusin tero placentaria Luisa Fernanda Anaya Admad Dr. Orlando Peinado, gineclogo Peso bajo: 15.7%, RCIU: 11.4%, Anencefalia: 3.9%, Genopatas: 2.7%, circular de cordn: 2%, Fetopata diabtica: 1.6%, Hipoplasia pulmonar: 1.2%, hemlisis: 1.2%, mielomeningocele: 2%Enfermedades maternas: Enf. Hipertensiva del embarazo: 20.7%, cardiopatas: 16.7%, RPM 13%, Infeccin urinaria: 12%, diabetes: 11.3%, desprendimiento de placenta NI: 6%, LES:

Bibliografa: 1. OBSTETRICIA, Schwarcz R, editorial El ateneo, 2003. 2. Sociedad espaola de ginecologa y obstetricia, junio 2002. 3. OBSTETRICIA CLNICA, Llaca V, edicin 2000, captulo 24; Pg, 315-316.

14.- Una mujer de 21 aos, nuligrvida, acude a consulta para hablar sobre anticoncepcin. Es sexualmente activa desde hace 2 semanas y actualmente utiliza condn. Tiene antecedente de asma, la cual se ha mantenido inactiva por 2 aos. No toma medicamentos y niega alergias. No hay AHF de cncer. Su EF es normal. Despus de una pltica con su mdico, escoge tomar anticonceptivos orales combinados, y contina tomndola por 6 aos. Ahora ha disminuido su riesgo de desarrollar: a) b) c) d) Cncer de mama. Cncer cervical. Cncer heptico. Cncer ovrico.

PROTECCION CONTRA EL CANCER EPITELIAL OVARICO. Debido a la falta de estrategias eficaces para el diagnostico y tratamiento temprano del cncer de ovario, es de capital importancia la prevencin (76). Esta patologa es una importante causa de morbilidad y mortalidad. Se estim que para 1980 ocurrieron 137.600 casos nuevos en el mundo (86). Estudios a gran escala realizados por el Centro para el Control de las Enfermedades de EE.UU. y el Royal Collage of General Practitioners del Reino Unido (RCGP) indican que la supresin de la ovulacin causada por los anticonceptivos orales protege contra el desarrollo del cncer epitelial ovrico (20,76). Este efecto benfico es directamente proporcional al tiempo de uso y persiste muchos aos despus de suspendida la planificacin con este mtodo (87). La evaluacin norteamericana denominada: Estudio sobre cncer y hormonas (CASH) demostr que el uso de uno a cinco aos de anovulatorios orales disminuye en un 50 a 70% el riesgo de cncer ovrico (88). Este efecto protector aumenta entre ms sea el tiempo de uso y se extiende por lo menos hasta diez aos despus de interrumpido (70,89). La Organizacin Mundial de la Salud tambin realiz un estudio multicntrico confirmando el efecto protector de los anticonceptivos orales contra el cncer epitelial ovrico (90). Dos estudios de Cohortes realizados en Gran Bretaa, confirmaron el efecto protector de la pldora al encontrar riesgos relativos de 0.3 y o.6 en mujeres que haban usado el mtodo en algn momento (83,91). El efecto protector es tanto para tumores malignos como para Bordenline (92) y cada uno de los principales subtipos histolgicos de cncer epitelial (70,93).

TABLA N 3 BENEFICIOS NO CONTRACEPTIVOS DE LOS ANTICONCEPTIVOS COMBINADOS. MEJORIA DE LA DISMENORREA. CORRECCION DE LOS CICLOS MENSTRUALES IRREGULARES. PREVENCION DE QUISTES OVARICOS FUNCIONALES. PROTECCION CONTRA EL CANCER EPITELIAL OVARICO. MEJORIA DEL MITTELSCHMERZ. ORALES

PROTECCION CONTRA EL CANCER ENDOMETRIAL. PROTECCION CONTRA TUMORES BENIGNOS MAMARIOS. DISMINUCION DE LA ENFERMEDAD PELVICA INFLAMATORIA. DISMINUCION EN LA INCIDENCIA DE EMBARAZO ECTOPICO. PREVENCION DE ANEMIA FERROPENICA. MENOR INCIDENCIA DE ARTRITIS REUMATOIDEA. MENOR INCIDENCIA DE OSTEOPOROSIS POST-MENOPAUSICA. MEJORIA DEL SINDROME PREMENSTRUAL. PREVENCION DE LA MIOMATOSIS UTERINA. MEJORIA DEL ACNE. 1. Bagshaw S. the combined oral contraceptives. Risk and adverse effects in perspective. Drug-Saf 1995; 12 (2): 91 - 96. 2. American Collage of Obstetricians and Gynecologist. Hormonal contraception. ACOG technical bulletin N. 198 - October de 1994 Int J Gynaecol Obstet 1995; 48 (1): 115 - 126. 3. Mishell Jr DR. Oral contraception: past, present and future perspectives. Int J Fertil 1992; 37 (1) Suppl: 7 - 18. 4. Melo NR, Pinotti J.Advances in hormonal contraception. Adv. in contraception 1994; 10 (suppl 1): 33 - 39. 5. Winkler UH, Schindler AE, Endrikat J, et al. A comparative study of the effects of the hemostatic system of two monophasic Gestodene oral contraceptive containing 20 ug and 30 ug Etinil-Estradiol. Contraception 1996; 53: 75 - 84. 6. Coenen CMH, Thomas CMG, Borm GF, et al. Changes in androgens during treatment with four low-dose contraceptives. Contraception 1996; 53: 171 - 176 7. Wilde MI, Balfour JA. Gestodeno. A review of its pharmacology, efficacy and tolerability in combined contraceptive preparation. Drug 1995; 50 (2): 364 - 395. 8. Monterrosa A. Anticoncepcin hormonal. EN : Caraballo J, Parra E, Taylor H. Memorias del 1 Curso de actualizacin en Ginecologa y Pediatra. Imprenta U. de Cartagena. Cartagena.1994; 241 - 250. 9. Hannaford PC, Combined oral contraceptives : do we know all of their effects. Contraception 1995; 51: 325-327. Rosenberg MJ, Waugh MS, Meehan T. Use and misuse of oral contraceptives : risk indicators for poor pill taking and discontinuation. Contraception 1995; 51: 283- 288.

15.- Una mujer de 32 aos de edad es llevada a quirfano por una laparoscopa diagnstica debido a dolor plvico y en cuadrante superior izquierdo crnicos (durante los ltimos 2 aos). No tiene alteraciones funcionales vesicales o intestinales. Tiene antecedente de 2 episodios de gonorrea previos. Bebe una cerveza al da. Labs: HCG urinaria negativa; Hto 39%; Leuc. T 8 000; Plt 200 000; AST 12; ALT 14. Intraoperatoriamente se observan adhesiones densas que involucran los oviductos, ovarios y tero. Tambin se observan adhesiones perihepticas que se extienden desde la superficie heptica hacia el diafragma. Cual de los siguientes es el diagnstico ms probable? a) b) c) d) Sx de Fitz-Hugh-Curtis. Hepatitis. Carcinoma hepatocelular. Sx Wolff-Parkinson-White.

El sndrome de Fitz-Hugh-Curtis se define como la presencia de una perihepatitis asociada a salpingitis. Los agentes etiolgicos reconocidos hasta la fecha son Chlamydia trachomatis y Neisseria gonorrhoeae. El cuadro clnico de este sndrome es inespecfico y puede ser confundido con procesos inflamatorios o infecciosos del tubo digestivo, aparato urinario y respiratorio, en los cuales la manifestacin sintomtica fundamental es el dolor en hipocondrio derecho. El diagnstico debe de sospecharse en aquella mujer joven con vida sexual activa que tenga antecedentes de promiscuidad en ella o en su pareja, que se queje de dolor subcostal derecho. Es ms probable el diagnstico si se cuenta con el antecedente de enfermedad plvica inflamatoria y ms an, si se tiene evidencia de que sta sea causada por Neisseria gonorrehoeae y/o Chlamydia trachomatis. El diagnstico definitivo se realiza con la visualizacin directa de la adherencia periheptica por laparoscopa o laparotoma. Se recomienda la primera. El tratamiento mdico es a base de cefalosporinas y dicloxacilina y en algunos casos se requiere de la extirpacin quirrgica del proceso adherencial para mitigar el dolor. El Sndrome de Fitz-Hugh-Curtis. Causa frecuente de error de diagnstico en hepatologa y gastroenterologa / The Fitz-Hugh-Curtis Syndrome. a frequent misdiagnosis in hepatology and gastroenterology Rev. gastroenterol. Mx;60(4):223-8, oct.-dic. 1995.

16.- Una mujer de 67 aos refiere intenso prurito vulvar y sensacin quemante, al examen el introito vaginal se encuentra estentico. Cul de los siguientes es el tratamiento apropiado? a) b) c) d) 5-fluoracilo. Testosterona tpica. Corticoesteroides fluorados. Estrgeno tpico.

Vulvovaginitis atrfica
El hipoestrogenismo conduce a atrofia de la vagina y el vestbulo vulvar, que los hace fcilmente irritables y susceptibles a infecciones secundarias. Las pacientes refieren sensacin de quemadura, prurito, disuria, hipersensibilidad y dispareunia. Puede encontrarse al examen fsico atrofia, fisuras superficiales, y un flujo vaginal acuoso1. Hay disminucin del tamao del introito2, prdida de la rugosidad y la vagina toma una apariencia lisa y brillante. Los hallazgos histolgicos revelan un epitelio vaginal delgado, disminucin de los lechos capilares, y la citologa muestra, a medida que la atrofia progresa, aumento de las clulas basales y disminucin o ausencia de las clulas superficiales2. Se aconseja evitar el uso de jabones y dems irritantes de la piel. Se pueden utilizar lubricantes simultneamente con los estrgenos o como terapia nica, si hay alguna contraindicacin a las hormonas. El tratamiento con estrgenos por va sistmica o transvaginal mejora y restaura los signos y sntomas, y una a dos semanas despus de iniciar el tratamiento los cambios de atrofia empiezan a mejorar rpidamente, se reduce el pH y se induce maduracin vaginal y de la mucosa uretral, reduciendo la frecuencia de las infecciones urinarias3. La dosis y va de administracin debe ser debidamente individualizada4. Contraindicaciones al tratamiento con estrgenos, incluyen: la presencia de tumores estrgenosensibles, falla heptica terminal y antecedentes de tromboembolizacin relacionada con ellos. Menopausia y Piel. ParteII:Manifestaciones- clnicas dermatolgicas durante la menopausia. MARA ISABEL BARONA C. Docente adjunto. Dermatloga Universidad del Valle-Cali.

17.- .- El dolor localizado, las parestesias, la sensacin de cuerpo extrao farngeo, el prurito nasal, la sialorrea, el nistagmus, las fasciculaciones linguales, se presentan en una intoxicacin por: a) Cocana. b) Vbora de cascabel. c) Araa Loxosceles reclusa. d) Alacrn. El veneno del alacrn acta sobre los canales de sodio, potasio, calcio. Existe liberacin de acetilcolina y catecolaminas, con efectos neuro y cardiotxicos. Es producido en el telson e inyectado a travs del aguijn del alacrn.

Grado I O Leve Dolor local intenso Parestesias locales y a distancia Inquietud Prurito nasal y farngeo Sialorrea Grado II o Moderado Diaforesis Sensacin de cuerpo extrao en faringe Nistagmus Dislalia Distensin abdominal Fasciculaciones linguales

Grado III o Grave Taquicardia Hipertensin arterial Visin de halos rojos Ceguera transitoria Disnea Dolor retroesternal Espasmos musculares Edema agudo pulmonar Priapismo Malestar vaginal

1.- Montoya-Cabrera MA. Intoxicaciones y envenenamientos en nios. Mxico, Intersistemas , 2000. 2.-Montoya CMA. Toxicologa clnica. 2. Ed, Mxico, Mndez Editores, 1997.

18.- Paciente de 4 aos de edad tiene lesiones eczematosas crnicas en flexuras de brazos y piernas que producen intenso picor, asociadas a una queilitis descamativa de labios. Cul, entre los siguientes, es el diagnstico ms probable? a) Una dermatitis atpica. b) Un eczema seborrico. c) Un prrigo nodular. d) Un eczema microbiano

La dermatitis atpica llamada comnmente eccema (atpico), es una enfermedad que consiste en un estado reaccional de la piel caracterizado por erupciones pruriginosas y con aspecto de escamas, ms frecuente en nios, multifactorial, en la cual intervienen factores tanto ambientales como constitucionales. Las personas con eccema a menudo tienen antecedentes de condiciones alrgicas como asma, fiebre del heno o eccema. La dermatitis atpica fue originalmente conocida como prrigo de Besnier y eccema constitucional, actualmente tambin es llamada neurodermatitis diseminada, por las escuelas europeas. Clnica Las manifestaciones clnicas tpicas de la dermatitis atpica se dividen en tres etapas, que suelen denominarse del lactante, infantil y del adulto. Junto a ellas se encuentran otras, con frecuencia llamadas atpicas, a pesar de que muchas, como la xerosis, son muy constantes.

Dermatitis atpica del lactante:

Suele empezar hacia los cinco meses de vida, pero puede hacerlo antes. Algunos nios desarrollan lesiones de eccema seborreico, que de forma gradual va adquiriendo el aspecto de la dermatitis o eccema atpico. La localizacin ms habitual es en la cara, respetando las zonas alrededor de los ojos, la nariz y la boca (. Son tambin frecuentes en el cuero cabelludo, las orejas, el dorso de las manos y las zonas de extensin de las extremidades. Las lesiones suelen ser ppulas o placas eritematosas y edematosas, muchas veces con erosiones, exudacin y costras. Es muy raro que se aprecien las vesculas caractersticas del eccema. El prurito es un sntoma constante.

Dermatitis atpica infantil:

Este periodo suele considerarse con un inicio hacia los dos aos y un final entre los siete aos y la pubertad. Las lesiones caractersticas se observan sobre todo en las flexuras, en especial en los codos y las rodillas (Fig. ), pero pueden aparecer en otras zonas. En esta fase es ms fcil ver lesiones eccematosas con vesculas, pero el intenso prurito hace que enseguida se transformen en erosiones, con exudacin y formacin de costras.

Bibliografa 1. Bielsa Marsol I. Eccemas (II). En: Ferrndiz C, ed. Dermatologa Clnica. Madrid, Mosby/ Doyma Libros 1996, 113-124. 2. Fernndez Vozmediano JM y cols. Dermatitis atpica. Madrid, Jarpyo 1994. 3. Fernndez Vozmediano JM, Armario Hita JC. Tacrolimus. Piel 2001;16:48-54. 4. Fonseca Capdevila E. Dermatitis atpica. Protocolo teraputico. (En lnea) (14.02.2001). Disponible en www.especialistasdermatologia.com. 5. Fonseca E. Dermatitis atpica en la infancia. Salud Rural 1997;14:92-105. 6. Guerra Tapia A. Dermatitis atpica. En: Fonseca Capdevila E, ed. Dermatologa Peditrica. Madrid, Aula Mdica 1999, 83-180.

19.- Femenino de 62 aos de edad el cual, experimenta episodios frecuentes de ptosis, diplopia y fatiga generalizada. A la EF se encuentra timo palpable y parlisis del nervio oculomotor que se corrige transitoriamente al administrar edrofonio. Cul es el diagnstico ms probable en su caso?

a) Miastenia gravis. b) Parlisis de Bell. c) Sndrome de Horner. d) Lupus.

Aunque la miastenia gravis puede afectar cualquier msculo voluntario, los msculos que controlan el movimiento de los ojos y los prpados, la expresin facial y el deglutir se ven afectados con mayor frecuencia. El inicio del trastorno puede ser repentino. A menudo, los sntomas no se reconocen inmediatamente como miastenia gravis. En la mayora de los casos, el primer sntoma perceptible es la debilidad en los msculos oculares (de los ojos). En otros, la dificultad para tragar e impedimentos en el habla pueden ser los primeros sntomas. El grado de la debilidad muscular de la miastenia gravis vara sustancialmente entre los pacientes, pudiendo manifestarse desde una forma localizada, limitada a los msculos oculares (miastenia ocular), hasta una forma grave o generalizada en la cual se afectan muchos msculos-incluyendo a veces los msculos que controlan la respiracin. Los sntomas, que varan en tipo y gravedad, pueden incluir la cada de uno o ambos prpados (ptosis), visin nublada o doble (diplopia) a consecuencia de la debilidad de los msculos que controlan los movimientos oculares, marcha inestable o irregular, debilidad en los brazos, las manos, los dedos, las piernas y el cuello y un cambio en la expresin facial, dificultad para deglutir y respirar y trastornos en el habla (disartria). Una de las pruebas paa realizar el diagnstico llamada la prueba del edrofonio. Este esquema requiere la administracin intravenosa de cloruro de edrofonio o Tensilon(r), un medicamento que bloquea la degradacin (interrupcin) de la acetilcolina y aumenta temporalmente los niveles de acetilcolina en las uniones neuromusculares. En individuos que padecen de miastenia gravis en los msculos oculares, el cloruro de edrofonio tiende a aliviar transitoriamente la debilidad. Otros mtodos para confirmar el diagnstico incluyen una versin del estudio de la conduccin nerviosa que examina el nivel de fatiga de un msculo en especfico mediante una estimulacin repetida de los nervios. Esta prueba registra respuestas de debilitamiento muscular cuando los nervios se estimulan en forma repetida y ayuda a distinguir entre trastornos nerviosos y los trastornos musculares. La estimulacin repetida de los nervios durante un estudio de conduccin nerviosa puede demostrar disminuciones en el potencial de accin muscular debidas a un deterioro en la transmisin del nervio al msculo.

GUIA PRCTICA PARA LA MIASTENIA GRAVE. Por: John E. Keesey, M.D. y Rena Sonshine. Traduccin al Espaol: On R. Pagn-Ojeda.

20. Un hombre de 55 aos acude a consulta por una historia de 2 meses de dificultad para tragar. En un principio, la dificultad era nicamente con bocados grandes de alimentos slidos, pero ahora tiene problema hasta con los lquidos. Tiene una historia de uso de alcohol y tabaco. La esofagoscopa demuestra una masa polipoide grande e irregular que ocluye casi completamente el tercio superior del esfago. Cual de los siguientes es el tipo histolgico de este tumor?

a) b) c) d)

Linfoma de clulas gigantes. Carcinoma de clulas pequeas. Linfoma de clulas pequeas. Carcinoma de clulas escamosas.

Hay varios subtipos, principalmente adenocarcinoma (aproximadamente 50-80% de todos los cnceres del esfago) y el cncer de clulas escamosas. El cncer de clulas escamosas surge de las clulas que recubren la parte superior del esfago. El adenocarcinoma se deriva de las clulas glandulares que estn presentes en la unin del esfago y el estmago. Los tumores de esfago por lo general llevan a la disfagia (dificultad para tragar), dolor.

Clasificacin: Los cnceres de esfago son generalmente carcinomas que surgen a partir del epitelio o revestimiento de superficie, del esfago. La mayora de los cnceres del esfago caen en una de dos clases: Los carcinomas de clulas escamosas, que son similares a cncer de cabeza y cuello en su apariencia y su asociacin con el tabaco y el consumo de alcohol, y los adenocarcinomas, que se asocian a menudo con antecedentes de enfermedad por reflujo gastroesofgico y esfago de Barrett. Una regla del pulgar general es que un cncer en la parte superior de dos tercios es un carcinoma de clulas escamosas y uno en el tercio inferior es un adenocarcinoma.

REVISTA ESPAOLA DE PATOLOGA Vol. 37, n. 4, 2004 Protocolo e informacin sistematizada para los estudios histopatolgicos relacionados con el carcinoma esofgico Francisco Colina, Guadalupe Lpez Alonso, Carolina Ibarrola.

21.- En un paciente masculino de 35 aos que se presenta con meningitis bacteriana, el tratamiento antibitico emprico de eleccin que deber ser indicado es: a) Imipenem. b) Penicilina. c) Ciprofloxacino. d) Cefotaxima.

La meningitis puede definirse como una inflamacin de las leptomeninges (piamadre y aracnoides) con afectacin del LCR que ocupa el espacio subaracnoideo; puesto que tanto las leptomeninges como el LCR se extienden por el cerebro y canal medular, el trmino implica siempre una afectacin cerebroespinal. La meningitis bacteriana es quizs una de las ms claras emergencias de todas las enfermedades infecciosas puesto que el tratamiento tardo o inadecuado incrementa el riesgo de muerte o de morbilidad neurolgica en aquellos que sobreviven. La meningitis bacteriana es ms frecuente en las edades extremas y entre los inmunodeprimidos, pero puede ocurrir en cualquier grupo de edad.

ETIOLOGA
Tres

Haemophilus influenzae, Neisseria meningitidis (meningococo) y Streptococcus pneumoniae (neumococo), son los responsables del 70-85% de los casos de
organismos, meningitis bacterianas . Cada organismo predomina en una poblacin especfica, que puede ser establecida en funcin de la edad y las condiciones de base del huesped.

Examen fsico Fiebre, cefalea, signos de irritacin mengea, y alteracin del nivel de conciencia se presentan en ms del 85% de los adultos con meningitis. En una amplia serie, el rango del estado de conciencia observado a la admisin fu la siguiente: alerta (17.8%), irritable o letrgico (52.1%), estuporoso/obnubilado (20.3%), o comatoso (9.7%).

Pueden presentarse signos de incremento agudo de la presin intracraneal (PIC) que incluye prdida de conciencia, pupilas dilatadas o con reactividad perezosa, oftalmopljia, afectacin de la funcin respiratoria, inestabilidad cardiovascular, posturas motoras anmalas, hiperreflexia y espasticidad. Aproximadamente el 50% de los adultos con meningitis desarrollan complicaciones neurolgicas. A la infeccin del SNC se asocia con frecuencia neumonia (25-50%) y otitis media (33%). Las complicaciones sistmicas incluyen shock septico (11.6%), SDRA (3.5%) y CID (8.15%) Pruebas de laboratorio La valoracin del LCR es esencial para el diagnstico de meningitis, permitiendo establecer el diagnstico de infeccin del SNC y diferenciar las infecciones bacterianas de las no bacterianas. El riesgo de herniacin cerebral debe ser siempre considerado antes de la realizacin de la puncin lumbar (PL), mediante el exmen neurolgico e incluso estudios de neuroimagen. Al realizar la puncin, si la presin de apertura estuviese muy elevada (>40 cm de H2O), el LCR se extraer lentamente y se reducir al mnimo el volumen obtenido. Deben obtenerse al menos dos muestras de 2-4 ml en tubos estriles: uno para recuento-frmula celular y bioqumica, y otro para pruebas microbiolgicas.

Tratamiento antimicrobiano El conocimiento de la escasa actividad opsonica del liquido cerebroespinal, y considerando que el LCR es un rea con resistencia disminuida a la infeccin, los antibiticos utilizados deben tener actividad bactericida en el LCR. En la tabla 5, se muestran las recomendaciones para el tratamiento antibitico en pacientes con meningitis bacteriana que tienen tincin de Gram o cultivo de LCR positivo, y en las tablas 6 y 7, las recomendaciones para el tratamiento emprico en pacientes con sospecha de meningitis bacteriana con tincin de Gram o cultivo de LCR no diagnstico (o la realizacin de la puncin lumbar ha sido postpuesta), pudiendo ser dirigida la antibioterapia contra el patgeno probable en funcin de la edad y las condiciones de base del paciente.

Tabla 5 Antibioterapia en meningitis bacterianas Antibioterapia basada en el organismo sospechado o aislado Organismo Antibitico Cefotaxima, ampicilina, penicilina G, vancomicina Penicilina G, ampicilina, cefotaxima, cloranfenicol Cefotaxima, ceftriaxona, ampicilina Oxacilina Vancomicina + Rifampicina Ampicilina Streptococos Penicilina G, ampicilina Bacilos Gram-neg Ceftriaxona, cefotaxima, TMP-SMZ Enterobacterias Ceftriaxona, cefotaxima P. aeruginosa Ceftazidima S. epidermidis Vancomicina + Rifampicina Anaerobios Cefotaxima + metronidazol + rifampicina

S. pneumoniae N. meningitidis H. influenzae S. aureus (MS) S. aureus (MR) L. monocytogenes

Tabla Antibioterapia en Tratamiento antibitico emprico* Nios

meningitis

6 bacterianas

Cefotaxima (75-100 mg/kg/6 h) o Cefotaxima (50 mg/kg/6 h) + vancomicina (15/mg/kg/6 h) o Ceftriaxona (100 mg/kg/24 h) + vancomicina (15 mg/kg/6 h) Nios alrgicos a la penicilina Vancomicina (15 mg/kg/6 h) + aztreonam (120 mg/kg/6 h) o Vancomicina (15 mg/kg/6 h) + cloranfenicol (100 mg/kg/6 h) en los nios vacunados contra H. influenzae Adultos Cefotaxima** (75 mg(kg/6 h; mximo, 24g/da) + ampicilina (60 mg/kg/6 h), si se desea cubrir L. monocytogenes

Adultos alrgicos a la penicilina Vancomicina(15 mg/kg/12 h) + aztreonam(120 mg/kg/6 h; mximo,8 g/dia) *Tincin de Gram y pruebas de deteccin de antgeno en LCR negativas. **Si la etiologa neumoccica se considera muy improbable puede administrarse ceftriaxona (100 mg/kg/6 h; mximo, 8 g/da)

Tabla 7 Antibioterapia en meningitis bacterianas Antibioterapia emprica segn edad y factores predisponentes: Edad y predisponente Neonatos (<1 mes) factor Tratamiento Ampicilina Cefotaximaa Ampicilina Cefotaximaa Cefotaximaa Cefotaximaa Cefotaximaa,b,c de o crneo fstula Cefotaximaa,d + Grmenes a cubrir Enterobacterias,

Estreptococo Listeria

1-3 meses 3 meses-5 aos > 5 aos y adultos

Meningococo, H. influenzae Meningococo, H. influenzae, Neumococo Meningococo,

Estreptococo A

Neumococo,

Ancianos Fractura cerrada de LCR

Neumococo, Meningococo, Bacilos gram-negativos,

Listeria

Neumococo

Estreptococo A
+ Enterobacterias + Pseudomonas S.aureus

Fractura de crneo Vancomicina abierta o craneotoma. Ceftazidima Neutropenia Amikacina Inmunodepresin neutropenia. Alcoholismo.
a b c d

epidermidis

sin

Cefotaxima Ampicilina

Igual que en adultos + enterobacterias, H. influenzae y Listeria

Puede usarse tambien ceftriaxona Muchas autoridades recomiendan antibioterapia de amplio espectro Si se quiere cubrir Listeria, debe aadirse amplicilina En pacientes graves, cubrir tambin estafilococos

22.-Is the easiest method to dermine if a patient is in prerenal azotemia:

a) b) c) d)

Plasma creatinine 24 hours cuantification. GFR cuantification. Granular casts in urinary sediment. BUN: Cr ratio.

La IRA prerrenal, tambin llamada azoemia prerrenal, es la causa ms frecuente de IRA, representando en realidad una respuesta fisiolgica a la hipoperfusin renal. Por definicin el tejido renal se mantiene ntegro, como lo prueba el que estos riones trasplantados a otras personas funcionan adecuadamente en el receptor y el que la funcin renal se normaliza rpidamente si se corrigen las causas que originaron la hipoperfusin renal. De mantenerse estas, la isquemia continuada puede terminar lesionando el parnquima renal, conduciendo a la situacin de NTA isqumica. Por lo tanto la IRA prerrenal y la NTA isqumica son parte de un mismo espectro de hipoperfusin renal, que en casos extremos puede llegar a la necrosis cortical. La relacin BUN: Crea en la azotemia prerenal es superior a 20. La razn para esto yace en los mecanismos de filtracin de ambos compuestos. Los niveles de filtracin glomerular disminuyen por la hipoperfusin, conduciendo a un mayor incremento en el BUN que en la creatinina. Como el rin est funcionando correctamente, la respuesta al disminuir la filtracin glomerular es el incremento en la reabsorcin. El aumento en la reabsorcin de sodio produce tambin un aumento en la reabsorcin de agua y urea a nivel del tbulo proximal. En contraste, la creatinina es secretada en el mismo nivel de los tbulos, teniendo una fraccin excretada mayor. Este mecanismo de autoregulacin renal, lleva a que BUN: Cr sea mayor a 20 adems de una fraccin de sodio excretado menor a 1%, junto a una elevada osmolaridad urinaria (debido principalmente a la reabsorcin de agua). Kumar, Vinay; Fausto, Nelson; Fausto, Nelso; Robbins, Stanley L.; Abbas, Abul K.; Cotran, Ramzi S. (2005). Robbins and Cotran Pathologic Basis of Disease, 7th edicin, Philadelphia, Pa.: Elsevier Saunders, pp. 960,1012.

23.- Masculino de 42 aos como antecedentes refiere malos hbitos alimenticios comenta que con mucha frecuencia consume alimentos en la va pblica, actualmente se presenta con fiebre, edema facial, fotofobia y mioartralgias estos datos son sugestivos de: a) b) c) d) Larva migrans visceral. Fasciolosis. Criptosporidiosis. Trichinellosis.

DEFINICIN Se denomina triquinosis a la Infeccin parasitaria producida por nemtodos del gnero Trichinella, transmitida por carnivorismo, y caracterizada por un sndrome febril, signos oculopalpebrales, mialgias y eosinofilia elevada. Sinonimia: Trichinellosis. AGENTE Y CLASIFICACIN La Trichinella es un pequeo nemtodo blanquecino y filiforme, con su extremidad anterior ms adelgazado que la posterior. La hembra mide entre tres y cuatro milmetros, en tanto que el macho es de menor tamao. Originalmente, se reconoca como nica especie a la Trichinella spiralis (Owens, 1835), pero en diversas reas geogrficas se han descrito recientemente triquinas que, aunque morfolgicamente similares, presentan sutiles diferencias en sus caractersticas biolgicas. As, en la actualidad se distinguen: 1. 2. Trichinella spiralis, propia de las zonas geogrficas templadas. Trichinella pseudospiralis, la cual, aunque no es frecuentemente observada afecta ms a las aves que a los mamferos y se caracteriza por ser de menor tamao y por no formar quistes en la musculatura del hospedador. Trichinella nelsoni del Africa tropical, la cual se encuentra en los grandes carnvoros de la regin, presenta un bajo grado de infectividad para los cerdos domsticos y ratas de laboratorio, y en el hombre provoca intensas infecciones con un gran nmero de larvas por gramo de msculo; aunque ha sido fatal en ocasiones, es muy bien tolerada a pesar de la masividad de la infeccin. Trichinella nativa de las zonas rticas que, distintivamente, presenta una considerable resistencia a la congelacin, tiene bajo grado de infectividad para el cerdo domstico, y en el hombre provoca importantes sntomas digestivos, principalmente diarreas prolongadas. Si bien T. spiralis, T. nelsoni y T. nativa son morfolgicamente similares, se ha establecido la diferenciacin entre ellas por sus caractersticas isoenzimticas y mediante el uso de anticuerpos monoclonales. En cambio, existen diferencias estructurales entre esas tres "especies" y la T. pseudospiralis. Ciclo domstico: El cerdo adquiere la infeccin, principalmente, por la ingestin de ratas infectadas, lo que es posible cuando es criado en malas condiciones higinicas o, simplemente, cuando debe buscar su propia fuente de alimentacin en sitios eriazos o basurales; adems, el cerdo se infecta con carnes de otros animales que encuentra en los criaderos o en los basurales. Las ratas, debido principalmente a sus hbitos de canibalismo, mantienen y propagan la infeccin en la naturaleza.

3.

4.

Los principales huspedes domsticos de la T. spiralis son la rata, el cerdo y el hombre. El hombre adquiere la infeccin a travs de la ingestin de carne de cerdo cruda o insuficientemente cocida, con larvas de triquina. Los jugos digestivos digieren la carne y las larvas quedan en libertad en el intestino, donde rpidamente, ya a las cuarenta y ocho horas, se diferencian en hembras y en machos adultos. Copulan en el lumen intestinal y, mientras los machos son eliminados con las deposiciones del husped luego de cumplida su funcin gensica, las hembras grvidas - que son vivparas- se localizan en el interior de la mucosa del duodeno y del yeyuno. Entre el tercero y el quinto da, comienza la postura de larvas. Cada hembra coloca alrededor de 1.500. Estas larvas miden entre 80 y 120 micrones, se profundizan en la mucosa intestinal, penetran a travs de los capilares linfticos y venosos y llegan a la circulacin general, diseminndose por todo el organismo, pero enquistndose slo en la musculatura, esqueltica. Las larvas se localizan en el interior de las fibras musculares, destruyndolas parcialmente; al cabo de unos quince das, quedan rodeadas por una envoltura constituida por el sarcolema. As se origina el quiste larval, que mide entre 250 a 400 micrones y que, en consecuencia, no es visible a simple vista, aunque puede observarse con una lupa o con un microscopio de poco aumento. Tiene un aspecto fusiforme o alargado, que recuerda la forma de un limn, y contiene, enrollado en su interior, una o varias larvas de triquina. La invasin de la musculatura esqueltica por las larvas, comienza alrededor del sptimo da de ocurrida la infeccin y contina mientras existan hembras grvidas en el intestino. Al cabo de un mes, las larvas completan su encapsulamiento y a los seis meses, se inicia el depsito de calcio en las paredes del quiste. La calcificacin total se alcanza en un plazo aproximado de un ao. En consecuencia, un mismo individuo es, sucesivamente, husped definitivo e intermediario del parsito. Es hospedero definitivo cuando alberga en su intestino las formas adultas, y es intermediario cuando las larvas se localizan en su musculatura. Sin embargo, para completar todo su desarrollo, la Trichinella requiere siempre de dos huspedes

1. Leo X. Liu Peter F. Weller. Helminthic Infections: Trichinella and other tissue Nematodes. In Baunwald E, Faucy AS, Kasper DL, Hauser SL, Longo DL, Jameson JL, editors. Harrisons Principles of Internal Medicine. 15 Th ed. New York . McGraww-Hill; 2001; 1231-3. 2. Pozio E. New patterns of Trichinella infections. Vet Parasitol 2001; 98: 133-48. 3. Murrel KD. Trichinellosis: now and forevermore?. Parasite 2001; 8 (2 supl.): 11-3. 4. Snchez Rodrguez A, Martnez Lpez de Letona J, Arias paciencia M, Snchez Garca AM, Paz Bouza J, Jarrin J, et al. Triquinosis. Estudio de 21 casos en un mismo brote. Rev Clin Esp 1982; 165: 79-84.

5. Perucha Gonzlez M, Lezaun Larrumbe ME, Torres Baile JL, Campo Hernndez JM, Bernal Martnez A. Brote de Triquinosis en varias localidades de la Rioja Baja. Rev Sanid Hig Pblica (Madrid) 1987; 61: 1035-47. 6. De la Torre Cecilia C, Espino Aguilar R, Crdenas Talavern C, Canuelo Ruiz O, Garrido Palomo R, Baena Sez J, et al. Triquinosis: presentacin de 2 casos. An Esp Pediatr 1989; 30: 227-8.

24.-Una mujer de 27 aos llega al servicio de urgencias con disnea y dolor torcico pleurtico. Tambin refiere que en los cuatro das previos tuvo tumefaccin e hipersensibilidad en la pantorrilla y el muslo derechos. Con base en la presentacin clnica se sospecha una trombosis venosa profunda que podra haber causado embolia pulmonar. Cul de los siguientes fragmentos de informacin de los antecedentes de la enferma apoyan mejor este diagnstico? a) b) c) d) Antecedente de tabaquismo. Antecedente de diabetes mellitus en la familia de la paciente. Antecedente de lesin en la extremidad inferior. Antecedente de hipertensin.

Allen R. M. MMS Medicina Interna. 5. Edicin. National Medical Series. Mc. Graw Hill. 2006. (captulo 1 VIIIA 2 d, 3 a) La lesin de las extremidades inferiores puede producir formacin de cogulos sanguneos y desarrollo de tromboflebitis. Otros factores que contribuyen al desarrollo de trombosis venosa profunda incluyen uso de compuestos con estrgenos (p. ej., anticonceptivos orales) o inmovilizacin de la extremidad inferior (p. ej., durante ciruga o reposo en cama prolongado) que da lugar a estasis venosa. La hipertensin, la diabetes mellitus y el abuso de drogas intravenosas no tienen relacin con la trombosis venosa profunda. La disnea y el dolor pleurtico sugieren que la trombosis venosa profunda ha causado embolia pulmonar, que es provocada por desplazamiento de un trombo de las venas de las extremidades inferiores o la pelvis a la arteria pulmonar.

25.- Una mujer de 24 aos refiere que sus manos se tornan blancas y luego azules en el fro. Cul de los siguientes datos sugiere ms fuertemente esclerodermia como causa de sndrome de Raynaud en esta paciente?

a) Engrosamiento cutneo distal que se extiende en direccin proximal hasta las articulaciones metacarpofalngicas. b) Anticuerpos anticentrmero en suero. c) Anticuerpos antinucleares en suero. d) Cambios capilares distales en la valoracin del lecho ungueal.

Allen R. M. MMS Medicina Interna. 5. Edicin. National Medical Series. Mc. Graw Hill. 2006. (captulo 10 VIII F 1). El engrosamiento de la piel es una caracterstica definitoria de esclerodermia; si se limitan las reas distales puede ser manifestacin de la variante CREST de esclerodermia (esclerodermia que coexiste con calcinosis subcutnea, fenmeno de Raynaud, disfuncin de la motilidad esofgica, esclerodactilia y telangiectasia). Los pacientes con fenmeno de Raynaud seropositivos para anticuerpos antinucleares (ANA) o anticuerpos anticentrmero o que muestran cambios capilares digitales, estn en ms alto riesgo de presentar esclerodermia que un paciente con fenmeno de Raynaud sin esas caractersticas. La dismotilidad, el espasmo o la estenosis esofgicos distales son posibles datos de esclerodermia, pero no son especficos.

26.- La rigidez cadavrica alcanza su mayor extensin e intensidad.

a) 4 hrs. b) 6 hrs. c) 12 hrs. d) 24 hrs.

La rigidez cadavrica suele ser completa en un periodo de 8 a 12 horas, alcanzando su mxima intensidad a las 24 horas y casi siempre inicia su desaparicin de 36 a 48 horas, siguiendo el mismo orden en que se instaur. La LEY de la RIGIDEZ CADAVRICA indica que cuando esta comienza precozmente, es de intensidad escasa y de duracin limitada. Existen excepciones claras a esta ley, como ocurre con las muertes por fro, calor o determinadas intoxicaciones. (Ley de NYSTEN) Martnez Murillo - Saldivar S. Medicina Legal. 16. Edicin, Francisco Mndez Oteo, 2003. (Captulo V, p. 43).

27.- El cncer de tiroides que puede producir un sndrome paraneoplsico y que se asocia a elevaciones de calcitonina es: a) Cncer anaplsico. b) Cncer de clulas de Hrttle. c) Cncer medular. d) Cncer papilar.

El cncer medular de tiroides surge de las clulas parafoliculares de la tiroides, que normalmente producen calcitonina. La medicin de calcitonina es importante sobre todo en el seguimiento de los pacientes para detectar enfermedad residual o recidivante. Jimnez RSA, Gmez VE, Bolaos GF. Tiroides. En Flores JF, Cabeza A, Calarco Z (ed): Endocrinologa. 5 ed. Mxico. Mndez Oteo Mxico, 2005: 584-92.

28.- Paciente que acude por presentar herida por mordedura de perro de 8 cms de longitud que afecta piel y tejido celular subcutneo, localizada en pierna derecha con 48 hrs de evolucin. , la herida se encuentra eritematosa, dolorosa y con secrecin verde amarillenta la primera medida teraputica es: a) Lavado quirrgico y cierre primario con drenaje con aplicacin de toxoide tetnico y antibiticos. b) Lavado quirrgico y cierre primario sin drenaje con aplicacin de toxoide tetnico y antibiticos. c) Lavado quirrgico y debridacin con aplicacin de toxoide tetnico y antibiticos. d) Lavado quirrgico y afrontamiento de bordes con drenajes.

Current Pediatric Diagnosis and Treatment 17 Ed Mc Graw Hill. Pag 336 . 2005.

29.- Un nio de 12 aos en Yautepec Morelos, se encuentra dormido sobre una toalla a la orilla de una alberca, sbitamente presenta dolor intenso en muslo derecho y se observa una pequea ppula eritematosa El nio refiere sensacin de ardor intenso en el muslo, su madre aplica una pomada con antihistamnico y le administra un antihistamnico oral. Dos horas despus el paciente inicia parestesias, nausea y vmito, comienza a presentar dolor abdominal intenso, en episodios, sntomas que en una hora se hacen ms intensos, la causa ms probable de este cuadro es: a) b) c) d) Picadura de viuda negra. Picadura de alacrn. Picadura de cara de nio. Picadura de nauyaca.

El veneno que inocula es 15 veces ms potente que el veneno de una serpiente de cascabel y puede condicionar efectos sistmicos graves e incluso la muerte. El primer sntoma generalmente es un dolor similar a una puncin con un alfiler y la sensacin se experimenta cuando realmente se ha efectuado la picadura de la araa. Sin embargo, es posible que algunas personas no lo sientan. Puede haber hinchazn y enrojecimiento leve y una lesin en forma de diana. De 15 minutos a una hora ms tarde, un dolor muscular sordo se irradia desde el rea de la picadura a todo el cuerpo. Sntomas Nuseas Ataque al estado general Diaforesis Contracturas musculares Dolor muscular Retencin urinaria Estreimiento Taquicardia Insuficiencia cardiaca Hipertensin arterial Inquietud Ansiedad Sensacin de muerte Inminente Current Pediatric Diagnosis and Treatment 17 Ed Mc Graw Hill. Pag 346 . 2005

30.- La indicacin quirrgica es obligada en un paciente afectado de enterocolitis necrotizante cuando presente uno de los siguientes signos clnicos: a) b) c) d) Heces mucosanguinolentas. Vmitos biliosos. Distensin abdominal. Neumoperitoneo.

Enfermedad adquirida que afecta principalmente a RN prematuros o patolgicos y que se caracteriza por necrosis de la mucosa o de capas incluso ms profundas del intestino, sobre todo en el leon terminal y con menos frecuencia del colon y del intestino delgado proximal. Sntomas, signos y diagnstico La enfermedad puede iniciarse con un leo que se manifiesta con distensin abdominal, residuos gstricos biliosos (tras las tomas) que pueden progresar a vmitos de bilis o presencia de sangre macroscpica o microscpica en las heces. La sepsis puede ponerse de manifiesto con letargia, inestabilidad trmica, aumento de las crisis de apnea y acidosis metablica. La deteccin sistemtica de sangre oculta o de sustancias reductoras en las heces de los prematuros (que han recibido alimentacin oral o enteral) puede ayudar a diagnosticar la ECN. Las radiografas iniciales pueden ser inespecficas o mostrar slo el leo. Sin embargo, un asa intestinal fija y dilatada que no cambia en las radiografas posteriores indica una ECN. Las radiografas diagnsticas son las que muestran neumatosis intestinal y gas en el territorio de la vena porta. El neumoperitoneo es un signo de perforacin intestinal e indica

la necesidad urgente de una intervencin quirrgica.

BOL PEDIATR 2006; 46(SUPL. 1): 172-178. Protocolos de Neonatologa. Enterocolitis necrotizante neonatal. I. FERNNDEZ JIMNEZ1, I. DE LAS CUEVAS TERN2. 31.- El principal estmulo para la sntesis y secrecin de prolactina es: a) b) c) d) La menstruacin. El embarazo. La succin. El trabajo de parto- el dolor.

La funcin principal de la prolactina en la mujer es estimular y mantener la lactancia puerperal, accin directa sobre las clulas acidoflicas conocidas como lactotrofas de la glndula mamaria. Para que aumente el sistema ductal se requiere de estrgenos, hormona del crecimiento, corticoides, lactgeno placentario y prolactina. Para el desarrollodel sistema lbulo alveolar se requiere de estrgenos, progesterona y prolactina, por lo que se deben considerar los niveles de estas hormonas en estados patolgicos como mastopata fibroqustica, mastodinia (dolor mamario), carcinoma mamario, etc.

NIVELES DE PROLACTINA DURANTE EL EMBARAZO Durante la gestacin las concentraciones de prolactina aumentan gradualmente a partir de la implantacin y siguen en ascenso casi lineal hasta el parto. Preparando a la glndula mamaria para la lactancia. Las clulas lactotrofas se ven aumentadas en nmero y tamao considerablemente, tal vez inducidas por el aumento de estrgenos hasta 10 veces ms que en la etapa pregestacional. El mayor estmulo para su produccin refleja es la succin y estimulacin del pezn, efectuada por las vas neurales desde la mama hasta los centros cerebrales. Durante la gestacin normal en el primer, segundo y tercer trimestre los niveles estn alrededor de 75, 116 y 216 ng/mL. Respectivamente segn reportan Tyson y col. El nivel mximo lo alcanza en la semana 25 y empieza a disminuir hacia la 38 semana. El ritmo circadiano y su secrecin por pulsos se mantiene durante todo el embarazo.

BIBLIOGRAFIA Azz D.C. Use and Interpretation of Tests in Endocrinology. 1997. Specialty Laboratories. Captulo 10 Desordenes pituitarios Pginas 129-130. Benson R.C. Diagnstico y Tratamiento Ginecoobsttricos. 1990. Editorial el Manual Moderno,S.A. Captulo 3 Fisiologa del sistema reproductor de la mujer. Pagina 64 y Captulo 37 El puerperio Paginas 818-820. Comparato M.R. Teraputica Hormonal en Ginecologa. 1988. Editorial "El Ateneo". Captulo 2 Hormonas sexuales. Pginas 36-37 Captulo 13 Teraputica hormonal en endocrinologa ginecolgica Pginas 207-216. Farreras P.Valenti y Rozman C. Medicina Interna.1982. Dcima Edicin. Ediciones Doyma. Captulo 15 Endocrinologa. Pginas 1805, 1806, 1811, 1812, 1814.

32.- Le envan a su consultorio paciente femenino de 62 aos con diagnstico de miastenia gravis. Cul de los siguientes medicamentos est indicado? a) Curare. b) Neostigmina. c) Quinidina. d) Sumatriptn. e) Succinilcolina.

Manejo farmacolgico: Existen diferentes pautas teraputicas dirigidas a contrarrestar

los sntomas de la enfermedad o el mecanismo inmunolgico que la gatilla. Los frmacos utilizados son:

- Inhibidores de la Acetilcolinesterasa (Neostigmina, Piridostigmina). Dirigidos al manejo sintomtico de la MG, mejorando la fuerza motora pero no la progresin de la enfermedad. Su mecanismo de accin es la inhibicin reversible de la acetilcolinesterasa, lo cual genera un aumento de ACh en la placa motora. La dosis a utilizar es variable y debe modificarse en distintas etapas de la enfermedad, incluso siendo frecuente no lograr un efecto uniforme en los diferentes grupos musculares en un mismo paciente. El objetivo por lo tanto ser utilizar la dosis mnima con la que se genere la mejor respuesta clnica. El efecto se obtiene de 30 minutos a 2 horas de la administracin y tiene una duracin de hasta 6 horas. Las dosis recomendadas de Piridostigmina son de 15-60 mg cada 4-6 horas va oral y de Neostigmina 0,5-2 mg/kg cada 4-6 horas intramuscular. Las reacciones adversas asociadas son: dolor abdominal, hipersalivacin, aumento de las secreciones respiratorias y bradicardia y se relacionan con el efecto colinrgico generado, por lo que es necesario administrar concomitantemente atropina2,3.

1.-Ponsetia JM: Miastenia Gravis. Manual Teraputico. Barcelona; Springer Verlag Ibrica,
1995.

2.- Ponsetia JM, Espina E, Armengola M: Diagnstico y Tratamiento de la Miastenia grave. Med Clin (Barc) 2000; 115: 264-70. 3.- Drachman DB: Myasthenia gravis. N Engl J Med 1994; 330: 1797-810. 4.- Andrews PI: Autoimmune myasthenia gravis in childhood. Semin Neurol 2004; 24: 10110. 5.- Anlar B: Juvenile myasthenia: diagnosis and treatment. Paediatr Drugs 2000; 2: 161-9. 6. - Gajdos P: Myasthenic syndrome. Diagnosis trends. Rev Prat 2000; 50: 419-23 7. Arroyo H: Myasthenia gravis in childhood and adolescence. Rev Neurol 1996; 24: 1385-9.

33.- El examen microscpico de una biopsia pulmonar de un paciente con mucormicosis pulmonar mostrar: a) b) c) d) Micelio Dicotomizado Septado Con Conidias. Micelio Dicotomizado Hialino Cenocitico. Micelio Con Clamidoconidios Y Blastoporas. Micelio Septado Y Esclerotes De Mediar.

Examen directo. Se realiza a partir de exudados o secreciones nasales, expectoracin, lavados bronquiales y heces, inclusive se puede hacer a partir de biopsias. La muestra se debe aclarar con KOH al 10 %. Al microscopio se observan numerosas hifas no tabicadas, hialinas, dicotmicas, de aproximadamente 5F de ancho por 20-25 de largo, esta imagen es patognomnica. Harada M, Manabe T, Yamashita K, Okamoto N. Pulmonary mucormycosis with fatal massive hemoptysis. Acta Pathol 1992;42(1):49-54.

34.- En la infeccin por virus de la inmunodeficiencia humana por lo comn la linfadenopata difusa en una persona clnicamente sana suele ser un signo de: a) b) c) d) Linfoma. Sarcoma de Kaposi. Tuberculosis. No indica infeccin o tumor especficos.

Allen R. M. MMS Medicina Interna. 5. Edicin. National Medical Series. Mc. Graw Hill. 2006. (captulo 8 VIII G 1 b). La linfadenopata difusa en una persona infectada por virus de la inmunodeficiencia humana (VIH) que se encuentra clnicamente bien suele ser signo de que no hay infeccin especfica o tumor. Aunque todas esas respuestas pueden ser ciertas, las personas con mltiples ganglios aumentados de tamao, tuberculosis o trastornos malignos tienden a encontrarse enfermas. Con mayor frecuencia tambin experimentan prdida de peso y fiebre. Es ms probable que el linfoma se presente con afeccin orgnica en pacientes infectados con VIH que en otros. El sarcoma de Kaposi puede producir afeccin linftica, pero en general slo se encuentra en etapa tarda de la enfermedad con lesiones cutneas y mucosas extensas. La linfadenopata moderada es un dato comn en infeccin por VIH en etapa media. Se desconoce su causa exacta, pero la desaparicin de la linfadenopata prolongada puede preceder al deterioro clnico. La sfilis produce adenopata local o difusa en pacientes con infeccin por VIH o sin ella. Sin embargo, esta adenopata siempre se acompaa de algn otro dato de sfilis.

35.- Esteroide de origen principalmente placentario: a) b) c) d) Progesterona. Estrona. Estradiol. ACTH.

estrona y el estradiol son hormonas de origen maternoplacentofetal, la lipotropina y la ACTH son de origen fetales.

Moderno. Pag 187. La progesterona es el esteroide de origen principalmente placentario, la

DeCherney A. (1999) Diagnstico y tratamiento ginecoobsttricos.Mxico. Ed. Manual

36.- Acude a su consulta paciente femenino de 26 aos que cursa el tercer trimestre del embarazo tiene edema en miembros inferiores sin sintomatologa agregada, usted indica a la paciente: a) b) c) d) Elevacin de los miembros inferiores en decbito lateral. Restriccin Hdrica. Tiacidas. Diurtico de asa.

venoso es comn en la parte final del embarazo, La paciente debe tratarse solo si est molesta. La elevacin de los miembros inferiores (especialmente en decbito lateral) mejora la circulacin. Estn contraindicados los diurticos. 37.- En un paciente preescolar se debe sospechar el diagnstico de adenoiditis si ste presenta:

Moderno. Pag 237. El edema de las partes bajas producido por la impedancia del retorno

DeCherney A. (1999) Diagnstico y tratamiento ginecoobsttricos. Mxico. Ed. Manual

a) Roncus,fiebre y dificultad respiratoria. b) Tos nocturna, respiracin oral y voz nasal. c) Rinorrea, conjuntivitis y disfona. d) Halitosis,sibilancias y tos hmeda.

Signos y sntomas de adenoiditis Respiracin bucal y dificultad para la respiracin por la nariz. Voz nasal. Respiracin ruidosa. Ronquidos durante el sueo. Apnea (perodos en que deja de respirar durante unos segundos mientras duerme). Resfriados frecuentes. Tos nocturna. Mordida Abierta. Otitis Media Aguda y Sinusitis a repeticin. Hiperactividad. Dificultad para la concentracin. Somnolencia diurna. Nelson. Tratado de Pediatra. Tomo 1. 529-534. Ed. Interamericana

38.- La manifestacin clnica ms frecuente del hiperparatiroidismo es: a) b) c) d) Crisis convulsiva. Osteoporosis y fracturas. Arritmias y movimientos anormales. Litiasis renoureteral.

La hipersecrecin de hormona paratiroidea, por aumento del umbral en que la calcemia suprime a la PTH o por aumento de la masa de tejido paratiroideo, provoca hipercalcemia, la que es la responsable principal de la sintomatologa del HPT1. Que puede presentarse en forma: a) sintomtica o b) asintomtica (mnimamente sintomtica); esta ltima es la ms frecuente desde que se dosa en forma sistemtica la calcemia. No obstante, la expresin clnica no esta relacionada con el grado de elevacin de la calcemia (68). Forma sintomtica Las manifestaciones seas son ms frecuentes en mujeres, y la litiasis renal y lcera gstrica en el hombre. Las manifestaciones seas, condrocalcinosis, falla renal y episodios agudos son ms frecuentes en viejos, y la litiasis renal en jvenes (42). a) Manifestaciones renales: la hipersecrecin de PTH, especialmente por aumento de absorcin intestinal de calcio, provoca hipercalcemia, si esta supera el umbral de filtracin glomerular y resorcin tubular se produce hipercalciuria, la que se acompaa de orina alcalina debido a la acidosis renal tubular proximal, causada por el exceso de HPT, llevando a nefrocalcinosis y/o nefrolitiasis. Estas se presentan en el 30 a 70% de los HPT1, la funcin renal se afecta hasta llegar a la insuficiencia. Era la causa ms frecuente de diagnstico de HPT en la dcada del 60 a 70 y lo sigue siendo, actualmente, en nuestro medio. b) Manifestaciones seas: el exceso de PTH moviliza el calcio seo por aumento de la ostelisis osteoctica y por estimulacin de la proliferacin de osteoclastos, lo que lleva inicialmente a osteopenia difusa y finalmente a ostetis fibrosaqutica. Antes de 1960 esta era la forma ms frecuente de presentacin o sea quistes, tumores seos, fracturas mltiples y deformacin esqueltica con disminucin de estatura. En la mayora de los pacientes an sin enfermedad sea clnica, se comprueba prdida progresiva de masa mineral sea. Revista Mdica Universitaria, Volumen 1, Nmero 1, Diciembre 2005, ISSN 1669-8991 Hiperparatiroidismo primario, secundario y terciario: actualizacin Perinetti H.A. Instituto de Patologa de la Tiroides. Departamento de Medicina Quirrgica. F.C.M. UNCuyo.

39.- Le reportan larvas de Ancylostoma braziliensisi y Ancylostoma caninum son los agentes causales en el hombre de: a) b) c) d) Dermatitis de los nadadores. Oncodermatitis. Dermatitis verminosa reptante. Larva currens cutnea.

Helmintos

Dermatitis verminosa reptante


Agnte causal:

Ancylostoma caninum, A. braziliense, Uncinaria stenocephala, Gnatostoma spinigerum, A. tubaeforme, Bunostomum phlebotomum.
Introduccin La geohelmintiasis denominada larva migrans cutnea o dermatitis verminosa reptante (DVR) es una entidad clnica que forma parte del complejo sndrome de las larvas migratorias cutneas y viscerales. Se define como una erupcin autolimitada de la piel producida por larvas de nemtodos parsitos de diversos animales, cuyo husped natural no es el hombre, por lo que no pueden completar en l su ciclo evolutivo. Los agentes etiolgicos predominantes son A. caninum y A. braziliense. Se ha comprobado que el primero puede alcanzar el tubo digestivo y producir el cuadro clnico conocido como enteritis eosinoflica.

40.- Son indicaciones para oxigenoterapia suplementaria a largo plazo:

a) PaO2 menor a 55 y SaO2 menor a 88. b) Pa02 mayor a 55 y SaO2 menor a 88. c) PaO2 mayor a 55 y SaO2 mayor a 88. d) PaO2 menor 20 y SaO2 menor 80. Las indicaciones para oxigenoterapia suplementaria a largo plazo incluyen una Pao2 de 55 o Menor y Sao2 (SATURACION DE OXIGENO) de 88 o menor. Fishman, A; Manual de Enfermedades pulmonares, tercera edicin, Mc Graw Hill, pags 142-143, 2004. 41.- El cncer de tiroides ms frecuente es: a) Medular. b) Papilar. c) Medular. d) De clulas de Hrtle.

El cncer papilar de tiroides es el tipo de cncer de tiroides ms frecuente y constituye 70 a 90% de los tumores malignos bien diferenciados de tiroides. El cncer papilar suele ser multifocal e invadir localmente la glndula tiroides y extenderse a travs de la cpsula tiroides invadiendo estructuras adyacentes. Sin embargo la mayor parte de los casos este cncer no es agresivo cuando se detecta en etapas tempranas y alcanza una sobrevida de ms del 90% a 20 aos. Jameson JL, Weetman AP. Transtornos de la glndula tiroides. En Jameson JL (ed): Harrison. Endocrinologa. 1a ed. Madrid. MacGraw-Hill Espaa, 2006: 106.

42.- La imagen en cabeza de medusa puede observarse en un ultrasonido en casos de: a) b) c) d) Sndrome de hiperinfeccin por Strongyloides. Oclusin por Ascaris lumbricoides. Fasciolosis mltiple biliar. Uncinariasis masiva.

Ascaris lumbricoides es el agente etiolgico de la ascariasis; es un nematodo intestinal cosmopolita y es uno de los ms comunes helmintos que parasitan al hombre. Esta helmintiasis se adquiere por la ingestin de huevos larvados; las larvas durante su migracin pasan por el pulmn para completar su maduracin, ascienden por el rbol respiratorio para posteriormente ser deglutidas y llegar al intestino delgado en donde se transforman en adultos.

Las radiografas abdominales muestran niveles hidroareos y mltiples imgenes lineales de A. lumbricoides en las asas intestinales dilatadas (Khuroo, 1996).En el ultrasonido abdominal se puede demostrar un asa dilatada, con pared engrosada y una masa de gusanos que causa la obstruccin. Las imgenes de helmintomas se describen como una masa ecognica compleja de aire intestinal, helmintos y materia fecal, con morfologa de cabeza de medusa en el eje longitudinal y de roseta en el corte transaxial (Malde y Chadha, 1993). Los helmintos se identifican como estructuras ecognicas que revelan un canal anecoico, el cual representa el canal digestivo del parsito, y presentan movilidad (Khuroo 1996). El tratamiento primeramente es conservador con manejo hidroelectroltico adecuado, succin nasogstrica, antibiticos y terapia antihelmntica. Se ha llegado a utilizar el Gastrografn, que es una sustancia hiperosmolar que produce un exceso de lquido en la vecindad y alrededor de la masa de gusanos condicionando su separacin, a razn de 15 a 30 ml introducidos en el estmago, a travs de una sonda nasogstrica para tratar la obstruccin intestinal parcial (Maor y col., 1984).

REFERENCIAS American Academy of Pediatrics. Ascaris lumbricoides infections. In: Peter G. Ed. 1997. Red Book: Report of the Committe on Infectious Diseases 24th edition. American Academy of Pediatrics. Khuroo, S. 1996. Ascariasis. Gastr. Clin. North Am. 25:553-577. Malde, H. and Chadha, D. 1993. Roundworm obstruction: sonographic diagnosis. Abdom. Imagin. 18:274-276. Maor , B., de Carvalho, F. and Chapell, J. 1984. Gastrofn treatment of intestinal obstructions due to Ascaris lumbricoides. J. Ped. Surg. 19:174-176. Marinho, HA., Shrimpton, R., Giuliano, R. and Burini, RC. 1991. Influence of enteral parasites on the blood vitamin A levels in preschool orally supplemented with retinol and/or zinc. Eur. J. Clin. Nutr. 45:539-544. Rao, P.L., Satyanarayana, G. and Venkatesh, A. 1988. Intraperitoneal Ascariasis. J. Ped. Surg. 23:936-38. Rosner, B. 1995. Hypothesis testing: Categorical data. In: Rosner Bernard (ed.) Fundamental of Biostatistic. Duxbury de. Fourth ed. Harrisonburg, VA.: 345440. Salman, B. 1997. Management of intestinal obstruction caused by ascariasis. J. Ped. Surg. 32:585-587.

43.- Masculino de 44 aos originario del Edo de Mxico, ganadero, se le diagnostica Fasciola Heptica, su forma infectante es: a) b) c) d) Quiste. Ooquiste. Metacercaria. Esporoblasto.

subclase Digenea, caracterizado por su forma lanceolada, con dos ventosas, una bucal y otra ventral, y un ciclo biolgico con dos generaciones (digeneo) en dos hospedadores, un molusco gasterpodo anfibio y un mamfero. Es parsito de los canales biliares y la vescula biliar de herbvoros y omnvoros, incluido el hombre; es el agente causal de una de las parasitosis ms difundidas del ganado, la fascioliasis (o fasciolosis), que es considerada como una de las enfermedades parasitarias ms importantes del mundo de los rumiantes domsticos.

Fasciola hepatica o duela del hgado es una especie de platelminto trematodo (duela) de la

Metacercaria La metacercaria es la forma infectante para el hombre y para los dems animales que sirven de hospedador definitivo. Generalmente se encuentran enquistadas en la vegetacin acutica semisumergida que normalmente comen los animales, pero el hombre tambin acostumbra a ingerirlas. Tambin se adquiere la infeccin tomando aguas contaminadas. Al llegar al duodeno se desenquistan liberando un parsito juvenil que perfora la pared intestinal y en unas 3 horas, se aloja en la cavidad peritoneal en donde pasa de 3 a 16 das; posteriormente avanza por el peritoneo, llega a la cpsula de Glisson, la perfora, penetra al parnquima heptico del cual se alimentan los parsitos juveniles durante su migracin hacia los conductos biliares en donde se desarrolla hasta el estado adulto, lo que sucede en unos 2 meses; despus empezar a reproducir huevos que salen al exterior con la bilis y materias fecales, complementando as el ciclo biolgico.

Bibliografa
Abrous, M.; Rondelaud, D. and Dreyfuss, G. 1999. Paramphistomum daubneyi and Fasciola hepatica: influence of temperature changes on the shedding of cercariae from dually infected Lymnaea truncatula. Parasitol. Res., 85(8-9). p765-9. Acuna, R. 1998. Human fascioliasis: seasonal variations and female preponderance of complicated forms. J. Infect.; 37(1). p88-9. Agatsuma, T. 2000. Molecular evidence of natural hybridization between Fasciola hepatica and F. gigantica. Parasitol.Int., p231-38. Anderson, H.R.; Fairweather, I.; Bamford, D.R. and Montgomery, W.J. 1998. The effect of diamphenethide on protein synthesis by the liver fluke, Fasciola hepatica. International Journal for Parasitology. 23. p1053-1062.

Basso, N.; Calceta Resio, E.; Dughetti, R.P.; Gimnez, R.A.; Peres Tort, G.B.; Rosa, A.B. y Welch, E.L. 1992. Fundamentos de Parasitologa Veterinaria. Ed. Hemisferio Sur. Argentina

44. - A 6-year-old child presents with flesh-colored papules on the hand that are not pruritic. Examination reveals lesions that are approximately 4 mm in diameter with central umbilication. A halo is seen around those lesions undergoing regression. Which of the following is the most likely diagnosis?

a) b) c) d)

Verruca vulgaris. Molluscum contagiosum. Keratoacanthoma. Herpetic whitlow.

El Molusco Contagioso es el nombre de una infeccin viral del grupo de los Poxvirus. Es frecuente, transmisible, autoinoculable (uno mismo la puede trasmitir a diferentes partes del cuerpo), se puede curar sola y es benigna. Afecta principalmente a los nios menores de 10 aos, adultos sexualmente activos y a pacientes inmunosuprimidos (con defensas bajas) como los pacientes con SIDA, ocurriendo

en un 5 18 por ciento. Afecta ms a hombres que a mujeres, y su frecuencia aumenta tambin en climas tropicales y durante el verano. CLINICA El periodo de incubacin de la infeccin es de 14 a 50 das, aunque hay datos de recin nacidos con lesiones al cabo de 7 das postparto. Las lesiones se inician como neoformaciones que miden generalmente de 2 a 6 mm, aunque pueden llegar a medir 3 cm (13,14), son hemisfricas, cupuliformes, lisas, del color de la piel o perladas, algunas (20%) tienen una umbilicacin central(15); la base es levemente eritematosa y son de consistencia firme(13,14). Se localizan en cualquier parte del cuerpo e incluso pueden afectar mucosas, generalmente se agrupan en un rea especfica, pero pueden estar diseminadas en personas infectadas con el virus de la inmunodeficiencia humana adquirida, siendo un marcador de enfermedad avanzada (16). En el caso de los nios las lesiones se localizan normalmente en cara, tronco, brazos y piernas a diferencia de los adultos jvenes en quienes el molusco contagioso se adquiere por transmisin sexual, las lesiones tienen predileccin por genitales, abdomen y cara interna de los muslos; en otras series no hay diferencia (17). Las lesiones del molusco suelen aparecer entre los 14 das a 6 meses despus de la exposicin, hay datos de recin nacidos con lesiones al cabo de 7 das postparto. Se pueden propagar por auto inoculacin, pero es tpico que se resuelvan espontneamente en pocos meses. Las lesiones son asintomticas en la mayora de los pacientes (13,18), aunque en el 10% de los casos puede haber prurito y desarrollarse una reaccin eccematosa (14). CRITERIO DIAGNSTICO El diagnstico se hace clnicamente y en algunos casos dudosos puede efectuarse biopsia con tincin H-E donde se encuentran los cuerpos de molusco ( inclusiones intracitoplasmticas grandes) o de Hendersen-Paterson; el 90% de los pacientes posee Ac tipo Ig G. Puede realizarse microscopa electrnica, PCR, Elisa, e inmunohistoqumica. REFERENCIAS 1. Schotz J, Rosen-Wolft A, Bugert J et al. Molecular epidemiology of molluscum contagiosum. J Infect Dis 1988; 158: 898-900. 2. Porter CD, Nlake NW, Archard LC et al. Molluscum contagiosum virus type in genital and non genital lesions. Br J Dermatol 1989; 120: 37-41. 3. Overfield TM, Briody JA. An epidemiologic study of molluscum contagiosum in Achorage, Alaska. J Pediatr 1966; 69: 640-42. 4. Gottlieb SL, Myskowwski PL. Moluscum contagiosum. Intern J Dermatol 1994; 33: 45361.

5. Telner P, Solomon LM. Eruptive molluscum contagiosum in atopic dermatitis. Can Med Assoc J 1966; 95: 978-79. 6. Pauly CR, Artis WM, Jones HE. Atopic dermatitis, impaired cellular immunity and molluscum contagiosum. Arch Dermatol 1978; 114: 391-93. 7. Dohil, P. Lin, J. Lee, A. Lucky, A. Paller, L The epidemiology of molluscum contagiosum in children. J Am Acad Dermatol. 2006;54:47-54. 8 Rosenberg EW, Yusk JW. Molluscum contagiosum. Eruption following treatment with prednisone and methotrexate.Arch Dermatol 1970; 101: 439-41. 9. Koopman JJ, Van Merrienboer FCJ, Vreden SGS, Dolmans WMV. Molluscum contagiosum: a marker for advanced HIV infection. Br J Dermatol 1992; 126: 528-29. 10. Schwartz JJ, Myskowski PL. HIV-related molluscum contagiosum presenting as a cutaneous hom. Int J Dermatol1992; 31: 142-44.

45.- Femenino de 43 aos portadora de DM tipo II, e HTAS, es ingresada al servicio de Medicina Interna por cetoacidosis. Posterior a su recuperacin metablica inicia con fiebre, cefalea, dolor facial, disminucin del nivel de conciencia y enrojecimiento nasal con lesin negruzca en fosa nasal derecha. El diagnstico ms probable en sta paciente es:

a) Endocarditis por S. aureus. b) Infeccin por Mucor. c) Carcinoma epidermoide. d) Infeccin por M. tuberculosis. Mucormicosis es el nombre comn dado a varias diferentes enfermedades causadas por hongos de la orden de los Mucorales. Muchas diferentes especies han sido implicadas como agentes de sndromes clnicos similares.

Los Mucoraceos son hongos que se encuentran en todas partes y son comunes habitantes de materia en descomposicin. Por ejemplo, Rhizopus sp. Frecuentemente puede ser recuperado de pan mohoso. Por su rpido crecimiento y prolfica capacidad de formar esporas, inhalacin de conidias debe ser una experiencia cotidiana. La presencia de esporas Mucorales en cinta adhesiva no estril se demostr fue la fuente de mucormicosis cutnea primaria. Aun cuando estos hongos crecen en muchos nichos ecolgicos, la infrecuencia de enfermedad debida a estos organismos da fe de su baja potencial virulencia en el husped humano. En contraste a la amplia distribucin de este hongo, la enfermedad en humanos esta limitada, en muchos casos a poblacin con severo inmunocompromiso, diabetes mellitus o trauma.

Manifestaciones clnicas En 1973 Meyer y Armstrong12 categorizaron las diferentes presentaciones clnicas de mucormicosis, considerando el rgano involucrado y haciendo una divisin en seis entidades: rinocerebral, cutnea, gastrointestinal, pulmonar, diseminada y formas miscelneas. Existe una fuerte asociacin entre la entidad subyacente y la forma de presentacin. La mucormicosis rinocerebral se presenta ms frecuentemente en pacientes diabticos con acidosis, y debido al advenimiento de quimioterapias cada vez ms potentes, se aprecia con ms frecuencia en pacientes leucmicos con neutropenia prolongada, en aquellos que reciben mltiples esquemas de antibiticos y corticoesteroides, as como en pacientes con trasplantes de rganos. En pocas ocasiones se ha documentado esta forma invasora en personas sin enfermedades subyacentes. La forma pulmonar puede presentarse en pacientes con leucemia y neutropenia; la mucormicosis gastrointestinal se observa ms comnmente en pacientes con desnutricin calrico-proteica y en prematuros, y la diseminada en pacientes con dficit inmunolgico grave como trasplantados, leucmicos y nefrpatas tratados con deferoxamina.1,2 La infeccin rinocerebral es la presentacin ms frecuente y caracterstica de mucormicosis, siendo la rinoorbitaria y la mucormicosis paranasal estadios tempranos de esta. La infeccin generalmente inicia en senos paranasales o paladar duro, y se extende a senos adyacentes con diseminacin a travs de senos etmoidales y zona retroorbitaria; puede tener acceso al cerebro a travs del pex orbitario, lmina cribosa y por va vascular. Una costra necrtica sangrante en paladar o en mucosa nasal y un drenaje ocular de pus negruzco orientan al diagnstico. Puede haber una progresin rpida y presentarse el deceso en pocos das o ser indolente si la enfermedad subyacente se logra controlar. Inicialmente puede haber dolor facial, cefalea, fiebre y algn grado de celulitis orbitaria, conforme progresa la invasin de la rbita la prdida de la funcin del II, III, IV y VI nervios craneales puede ocurrir, as como tambin puede haber prdida de la funcin de msculos extraoculares, proptosis, quemosis progresiva, congestin nasal, epistaxis y letargia. La disfuncin de nervios craneales, especialmente el V y VII, ocurre de manera tarda manifestando ptosis y midriasis, lo cual es un factor pronstico grave, en caso de trombosis retiniana, hay prdida de la visin y puede haber afeccin intraocular. El compromiso cerebral es la complicacin ms seria y puede manifestarse como: infarto, absceso, trombosis del seno cavernoso, hematoma subdural y necrosis del lbulo frontal.2 Otras complicaciones son trombosis de arteria cartida interna y vena yugular, e incluso se ha reportado infarto de miocardio por oclusin de coronaria y aborto sptico.

Puede haber presentaciones crnicas y secuelas tardas a pesar del tratamiento aparentemente satisfactorio por lo cual siempre debe realizarse un seguimiento del paciente a largo plazo.

BIBLIOGRAFIA:

Mayo Clinic. Pulmonary Diseases. Mandele Douglas. Infections Diseases. Pennington. Respiratory infections, Diagnosis and Management. Ronald B. george., Richard W. Light, Michael A. Matthay. Chest Medicine 3era. edicin. Scott E Davis. Neumona Mictica. Clnicas mdicas de Norteamrica 1997; 5: 1092?1094.

46.- Los agentes causales ms frecuentes de meningitis viral son: a) Virus de la parotiditis epidmica. b) Enterovirus. c) Arbovirus. d) Herpesvirus.

Introduccin
Con el trmino meningitis asptica (MA) se denomina al sndrome clnico caracterizado por: cefalea, fiebre, signos meningeos y lquido cefalorraqudeo (LCR) de aspecto claro pero con pleocitosis linfocitaria y que se presenta como consecuencia de la inflamacin de las leptomeninges (aracnoides y piamadre). Implcita a la definicin es la ausencia de signos de compromiso parenquimatoso (encefalitis) o inflamacin de la mdula espinal (mielitis) (1). La principal causa de la MA es la viral. Pero se debe tener en cuenta que ante un paciente con MA se debe plantear un diagnstico diferencial. Epidemiologa y etiologa Respecto a la etiologa viral, las series iniciales publicadas hasta fines de los aos cuarenta identificaban slo un 25% de los agentes etiolgicos, entre los cuales se contaban el virus de la parotiditis, de la coriomeningitis linfocitaria y de la polio; causas infrecuentes hoy en da (2). En las dcadas siguientes, en la medida que se ha logrado identificar a los enterovirus, estos han emergido como la principal etiologa reconocible, ms an, la aplicacin de la reaccin de polimerasa en cadena (PCR) ha permitido identificar al enterovirus no polio como el agente causante en un 85%-95% de los casos (2,3). Otros agentes virales son: Arbovirus, Herpes Simple, Varicela Zoster, Herpesvirus humano tipo 6, virus de la inmuno deficiencia humana (VIH), y virus Influenza, ver tabla 2 (1,2,3). A continuacin revisaremos los virus que con mayor frecuencia producen una MA.

Enterovirus Los enterovirus (EV) son los virus que predominan como agente etiolgico en la MA, causando entre un 85-95% de los casos. Pertenecen a la familia de los picornavirus, se clasifican en 2 clases: el grupo poliovirus (tipo 1, 2 y 3) y el grupo no poliovirus (coxsackie, echovirus y enterovirus no clasificados). Se han identificado 71 serotipos de EV. El ser humano es el nico reservorio (4,5).

Los individuos suelen infectarse por va de contaminacin fecal-oral, y menos frecuentemente por va respiratoria. La superficie celular del tracto gastrointestinal sirve como receptor viral, observndose que la replicacin viral inicial se desarrolla en el tejido linftico local. Aproximadamente al tercer da de la infeccin, el virus penetra al sistema circulatorio causando la primera viremia e invadiendo diferentes rganos, viremias sucesivas durante la primera semana, se asocian a una mayor replicacin y se correlacionan con los sntomas y signos de la infeccin viral, en esta etapa inicial ocurre la invasin del SNC (1,2,3). No existe variacin por raza ni sexo, la poblacin de lactantes y nios es la ms susceptible a la infeccin (4).

Referencias:
1. Harley A, Rotbart MD. Viral Meningitis. Sem Neurol. 2000; 20: 277-92. 2. Rotbart HA. Viral meningitis and aseptic meningitis syndrome. In: Scheld WM, Whitley RJ, Durack DT. (Eds). Infections of the central nervous system. Philadelphia, Lippincott Raven. 1997: 23-46. 3. Tunkel AR, Scheld WM. Acute meningitis. En: Mandell G, Bennett J, Dolin R. Principles and Practice of Infectious Diseases. New York. Churchill Livingstone. 2000: 959-89: 1016-28. 4. Mowad M. Enteroviral Infections. December 20, 2002. Emedicine.com. 5. Nigrovic L. What`s new with enteroviral infections?. Curr Op Pediatr. 2001; 13: 8994. 6. Dolin R. Enterovirus-71. Emerging infections and emerging questions. N Engl J Med.1999; 341: 984-5. 7. Sawyer MH, Saez-Llorenz X, Aviles CL. Oral pleconaril reduces the duration and severity of enteroviral meningitis in children. Proceedings of 1999 APS/SPR meetings, San Francisco; 1999.

47.- Se trata de paciente femenino de la tercera edad que acude por presentar durante las ltimas 3 semanas astenia, febrcula vespertina, cefalea global y, durante los ltimos das, le han notado confusin intermitente y somnolencia progresiva. A la exploracin se observa una temperatura de 38C, somnolencia, dudosa rigidez de nuca y paresia de VI par derecho. La placa de trax muestra un discreto engrosamiento pleural apical derecho. La biometra hemtica y qumica sanguinea se encuentra normal. Se realiza una puncin lumbar con los siguiente resultados en LCR: presin de apertura 170mmHg, protenas 140 mg/dL, glucosa 42 mg/dL, 270 clulas (70% mononucleares), tincin de gram, de Zielhl-Nielsen, as como investigacin de antgenos bacterianos son negativos. Que diagnstico es el ms probable?:

a) Meningitis tuberculosa. b) Encefalitis herptica. c) Meningitis bacteriana. d) Meningitis vrica.

Meningitis TB Causas, incidencia y factores de riesgo La meningitis tuberculosa es un trastorno muy poco comn, causado por la diseminacin de la Mycobacterium tuberculosis al cerebro, desde otro sitio en el cuerpo. Generalmente, sus sntomas se inician gradualmente. Entre los factores de riesgo se pueden mencionar antecedentes de tuberculosis pulmonar, consumo de alcohol en forma excesiva, SIDA u otros trastornos que comprometen el sistema inmunitario. MANIFESTACIONES CLINICAS. Los fenmenos clnicos evolucionan en varios periodos o etapas cuya duracin y contenido semiolgico son muy variables, pero que tienen unas caractersticas generales que permiten definirlos.En 1948 el British Medical Council defini los tres estadios evolutivos de la enfermedad.(7) Estadio I: consciente, sntomas inespecficos, ausencia de afectacin neurolgica. Estadio II: cierto grado de afectacin de la conciencia, aparicin de signos de afectacin neurolgica. Estadio III: afectacin profunda del estado de conciencia, convulsiones, signos de focalidad neurolgica.

La recuperacin total es la regla en los casos en estadio I mientras que las secuelas y muerte son muy elevadas en los estadios II y III. En esta ltima etapa el porcentaje de fallecimientos puede ser superior al 25%.(8,9). Estadio I En esta fase, los sntomas son muy inespecficos, sin que puedan atribuirse a alteracin del SNC, salvo por la aparicin de alteraciones del carcter, que si bien no son especficas si son bastante constantes. Fiebre, generalmente de escasa entidad. Otros sntomas de este periodo son cefalea moderada, apata, prdida de la alegra, anorexia, y con mucha frecuencia en el nio dolores abdominales, acompaados o no de vmitos, estos ltimos sin caractersticas que sugieran un origen central. Los sntomas de esta etapa en el lactante pueden pasar desapercibidos. Estadio II Junto a la persistencia de otros signos, entre ellos la fiebre, se ponen de manifiesto los signos de afectacin neurolgica, coincidiendo con los signos menngeos. Al principio predominan la cefalea y los vmitos, estos ltimos sobre todo en menores de tres aos. Aparece rigidez de nuca, que puede ser muy intensa, y signos de Kernig y Brudzinski. En ms de 1/3 de pacientes con meningitis tuberculosa hay afectacin de pares craneales. El sexto par, es el que se afecta con ms frecuencia, seguido por los pares III y IV, de forma uni o bilateral. El facial se afecta ms raramente, as como otros pares craneales. Puede afectarse el nervio ptico, conduciendo a la atrofia ptica y ceguera. Las convulsiones son en el nio una manifestacin habitual y precoz de la enfermedad.(10)Los signos de hipertensin endocraneal se van acentuando, aprecindose una fontanela tensa en el lactante, aumento del permetro craneal, edema de papila en el nio mayor, diplopia y visin borrosa. Loa dficits motores que pueden presentarse a lo largo de la evolucin son secundarios a los fenmenos vasculares descritos, que conducen a isquemia e infarto. Puede verse hemipleja por afectacin del territorio de la arteria cerebral media o cartida interna. Sntomas de afectacin medular pueden existir en casos muy evolucionados, con presentacin progresiva de parapleja. Durante este estadio las alteraciones del sensorio aun son discretas, y su empeoramiento es el que define el paso al siguiente periodo.

Estadio III. Presidido por la alteracin profunda del estado de conciencia, pasando de la apata e irritabilidad, a la confusin, estupor y coma profundo en los casos avanzados. Los casos terminales se caracterizan por coma profundo, rigidez de descerebracin, siendo la muerte inminente. Encefalopata tuberculosa. En los nios hay un cuadro, denominado encefalopata tuberculosa por Udani en 1958 (11), que se caracteriza por la presencia de signos de afectacin difusa con convulsiones estupor o coma, y signos moderados de afectacin menngea. El coma sobreviene precozmente, y la muerte sobreviene entre 1 y 2 meses despus del comienzo, aun con teraputica especfica. Dominan el cuadro los fenmenos de hipertensin intracraneal. Anatomopatolgicamente el cuadro est constituido por severo edema de la sustancia blanca, sin presencia de hemorragias. Microscpicamente hay una prdida difusa de mielina en la sustancia blanca, atribuyndose la causa a fenmenos de hipersensibilidad frente a la tuberculoproteinas Es una infeccin de las meninges, las membranas que recubren el cerebro y la mdula espinal, causada por la bacteria Mycobacterium tuberculosis que produce la tuberculosis.

Signos y exmenes
Para cualquier paciente con meningitis, es importante llevar a cabo una puncin lumbar: Tincin del lquido cefalorraqudeo (LCR) positiva para Mycobacterium . LCR con alto nivel de protenas, bajo nivel de glucosa y aumento en el nmero de linfocitos. Cultivo de LCR que muestra la proliferacin de Mycobacterium tuberculosis. Reaccin en cadena de la polimerasa (PCR) que muestra LCR positivo para M.

tuberculosis .
Prueba cutnea positiva para tuberculosis. Biopsia cerebral o menngea que muestra la presencia de M. tuberculosis.

BIBLIOGRAFIA 1. Sudre P,Dam G,Kochi A. La tuberculose aujourd'hui dans le monde.WHO Bull 1992;70:297-308. 2. Dolin PJ,Raviglione MC,Kochi A. Global tuberculosis incidence and mortality during 19902000.WHO Bulletin 1994;72:213-220. 3. March Ayuela P. Trend in tuberculous meningitis in Barcelona in children aged 0-4 years:correlation with the annual risk of tuberculous infection. Tubercle and Lung Disease 1994;75:423-428. 4. Barry R Bloom.Tuberculosis: Pathogenesis, Protection, and Control. Tuberculosis. American Society for Microbiology. Washington DC 2005. 5. Anggard E. Nitric oxide:mediator,murderer,and medicine. Lancet. 1994;343: 1199-206. 6. Jinkins JR,Gupta R,Hyun Chang K,Rodriguez-Carvajal J.MR imaging of cental nervous system tuberculosis. Radiol Clin North Am 1995;33:771-786. 7. Medical Research Council.Streptomycin in Tuberculosis Trials Committee:Streptomycin treatment of tuberculous meningitis.Lancet. 1948;1:582-596.

48.- Es la patologa que asocia distensin de los espacios areos respiratorios distales a los bronquolos terminales acompaada de destruccin de los tabiques alveolares:

a) Bronquitis crnica. b) Enfisema. c) Asma. d) Asbestosis.

Un enfisema se define en trminos anatomopatolgicos por el agrandamiento permanente de los espacios areos distales a los bronquiolos respiratorios, con destruccin de la pared alveolar, con o sin fibrosis manifiesta.1 Es una enfermedad crnica comprendida junto con la bronquitis crnica en la Enfermedad Pulmonar Obstructiva Crnica (EPOC).2 El nombre viene del griego emphysema que significa "soplar el aire" o "insuflar'.3

Fisiopatologa del enfisema Le enfermedad por sus efectos incapacita y debilita enormemente la calidad de vida del paciente, sometindolo a una vida restringida y sedentaria. El deterioro progresivo de su funcin pulmonar, desencadena no slo cambios fsicos en el paciente, sino tambin alteraciones a nivel psicolgico.

El enfisematoso, se ve ceido a realizar mnimos esfuerzos. Cualquier tipo de actividad fsica, se convierte en verdadero sufrimiento y evita realizarlas. Las ms elementales necesidades fisiolgicas, requieren de enormes esfuerzos y gastos extras de energa. El enfisema es una enfermedad crnica, progresiva, caracterizada por un agrandamiento anormal y permanente de los espacios areos distales al bronquiolo terminal, acompaado de destruccin de sus paredes sin fibrosis (8) La caracterstica ms relevante de la enfermedad, es su limitacin al flujo areo durante los movimientos respiratorios. Esta limitacin, medido en valores del volumen espiratorio forzado al primer segundo (VEF 1) por debajo de los valores predictivos del paciente (< 1,0 l), ocasiona una gran morbilidad, con deterioro severo de su capacidad vital (CV). El 25% de la resistencia total pulmonar al flujo areo, ocurre normalmente en las vas areas < 3mm, pero estos valores se aproximan al 80% en los pacientes enfisematosos (9,10). Este flujo, esta determinado por el juego entre presin retroceso pulmonar elstico. Como el volumen pulmonar durante la expiracin disminuye, ocurre cierre prematuro de las vas areas perifricas, debido a la prdida de elasticidad pulmonar y a la destruccin de los alvolos que se encuentran fijados a las delgadas paredes de los bronquiolos. Al final, estos pulmones enfisematosos son de mayores dimensiones a lo normal. Ello se traduce en disminucin del VEF 1, y otros flujos (FEF 25-75, FEF 50), aumento del volumen residual (VR) de la capacidad residual funcional (CRF) y de la capacidad pulmonar total (CPT), as como, una disminucin a la prueba de difusin al monxido de carbono (DLCO). El atrapamiento de aire en las zonas enfisematosas, se traduce en hallazgos radiolgicos tpicos; un pulmn hiperaireado, con escasa vasculatura pulmonar, aplanamiento de los diafragmas y un trax ms alargado. Al examen fsico, vemos un trax con dimensiones mayores en el dimetro antero posterior y escaso desarrollo de la masa muscular. Hay hipersonoridad a la percusin y los sonidos pulmonares se encuentran muy disminuidos. Todos estos cambios en la funcin pulmonar, ocasionan desigualdad en la relacin ventilacin / perfusin (V/Q), resultado de un incremento de espacio muerto, hipoxemia sola en fases tempranas, acompaadas de hipercapnia en las fases terminales. Basndose en estos datos, la indicacin quirrgica no slo se limita al pulmn enfisematoso puro, sino tambin, al pulmn con enfermedad bulosa (11-16).

Gordon LS. Emphysema: The first two centuries and beyond. A historial overview, with suggestions for future research: Part I. Am Rev Respir Dis 1992; 146:13341344. Gaast A, Molard-Dietmenn A, Pelletier A, Pauli G, Bieth JG. The antielastase screen of the lower respiratory tract of alpha I-proteinase inhibitor suficiente patients with emphysema or pneumotorax. Am Rev Respir Dis 1990;141:880-883. Janoff A. Elastases and emphysema, current assesment of the Protease-Antiprotease hypothesis. Am Rev Respir Dis 1985;417-433. Silverman EK, Speizer FE. Risk factors for the development of chronic pulmonary disease. Med Clin North Amer 1996;80:501-522. Deslaries J. A perspective on the role of surgery in chronic obstructive lung disease. Chest Surg Clin North Amer 1995;5:575-602. Brantigan OC, Mller E. Surgical treatment of pulmonary emphysema. Am Surg

1957;23:789-804. Cooper JD, Trulock EP, Triantafillou AN, Patterson GA, Pohl MS, Doloney PA, et al. Bilateral pneumonectomy (volume reduction) for chronic pulmonary disease. J Thorac Cardiovasc Surg 1995,109:106-119.

49.- Masculino de 64 aos, jubilado es ingresado al servicio de urgencias. Los familiares refieren, que el da de ayer de manera brusca el paciente inici con confusin, comenz a decir cosas raras, a no responder a lo que le preguntaban, incluso present confusin de su propio nombre. Al interrogatorio se muestra distrado y parece no entender lo que se le pregunta, no recuerda nada de lo que le ha pasado y no sabe ni el da, ni el lugar en el que est. El diagnstico ms probable de ste paciente es:

a) Delirium. b). Psicosis breve. c) Alzheimer. d) Esquizofrenia.

MANIFESTACIONES CLNICAS Los hallazgos cardinales del delirium incluyen su aparicin aguda y la inatencin. Para definir el momento exacto del inicio es necesario recurrir a la informacin dada por el cuidador del paciente. Otra de las caractersticas es la fluctuacin que presenta el cuadro a lo largo del da. El individuo fcilmente se distrae ante los diferentes estmulos externos, y es muy difcil que sostenga una conversacin y obedezca rdenes. Aunado a lo anterior puede presentarse desorganizacin del pensamiento, alteraciones de la sensopercepcin y malteracin del estado de conciencia (generalmente letrgico). Aunque no son sntomas principales, tambin presentan desorientacin, dficit cognoscitivo, agitacin o retardo psicomotriz, delirios, labilidad emocional e inversin del ciclo sueo-vigilia.

Criterios para el diagnstico de F05.0 Delirium debido a... (indicar enfermedad mdica) (293.0) A. Alteracin de la conciencia (p. ej., disminucin de la capacidad de atencin al entorno) con disminucin de la capacidad para centrar, mantener o dirigir la atencin. B. Cambio en las funciones cognoscitivas (como dficit de memoria, desorientacin, alteracin del lenguaje) o presencia de una alteracin perceptiva que no se explica por la existencia de una demencia previa o en desarrollo.

C. La alteracin se presenta en un corto perodo de tiempo (habitualmente en horas o das) y tiende a fluctuar a lo largo del da. D. Demostracin a travs de la historia, de la exploracin fsica y de las pruebas de laboratorio de que la alteracin es un efecto fisiolgico directo de una enfermedad mdica.

Nota de codificacin: Si el delirium est superpuesto a una demencia vascular


preexistente, indicarlo codificando F01.8 Demencia vascular, con delirium [290.41].

Nota de codificacin: Incluir el nombre de la enfermedad mdica en el Eje I, por ejemplo, F05.0 Delirium debido a encefalopata heptica [293.0]; codificar tambin la enfermedad mdica en el Eje III.

50.- El mecanismo de accin de la glibenclamida es:

a) Estimula la liberacin de insulina de las clulas beta del pncreas. b) Bloque la accin del pptido C con lo que disminuye la accin perifrica de la insulina. c) Bloquea a la leptina. d) Estimula el glucagn del hepatocito.

Goodman & Gilmans. The Pharmacology Diseases and Therapeutics. USA: Medical Publishing Division. Mc. Graw-Hill.2005: 1700-1707.2. La Glibenclamida es un agente antidiabtico debido a su accin hipoglicemiante. Estimula la liberacin de insulina de las clulas del pncreas; aumenta los niveles de insulina mediante la reduccin de la liberacin heptica de la hormona. Incrementa la sensibilidad de los tejidos perifricos a la accin de la insulina y disminuye la glucogenlisis heptica y la gluconegnesis. Su efecto global es una reduccin de la concentracin sangunea de glucosa en pacientes diabticos cuyo pncreas es capaz de sintetizar insulina.

51.- Pseudomonas aeuroginosa es una bacteria frecuentemente responsable de: a) b) c) d) Otitis media aguda. Otitis serosa. Otitis media crnica. Otitis externa necrotizante.

La infeccin crnica del odo medio y la mastoides por lo general se desarrolla como consecuencia de otitis media aguda recurrente, aunque puede presentarse despus de cualquier enfermedad y trauma. La perforacin de la membrana timpnica es comn, suele acompaarse de cambios en la mucosa como degeneracin polipoidea y tejido de granulacin, y cambios seos como ostetis y esclerosis. La bacteriologa de la otitis media crnica difiere de la otitis media aguda. Los microorganismos ms frecuentes incluyen P. aeruginosa, especies de Proteus, Staphylococcus aureus e infecciones mixtas por anaerobios. El hallazgo clnico de la otitis media crnica es una otorrea purulenta. El drenaje puede ser continuo o intermitente, con aumento en la intensidad durante una infeccin de vas respiratorias superiores o despus de una exposicin al agua. El dolor es poco frecuente excepto durante las exacerbaciones agudas. La prdida conductiva de la audicin es resultado de la destruccin de la membrana timpnica y de la cadena de los huesecillos. El tratamiento mdico de la otitis media crnica incluye remocin continua de los restos infectados, uso de tapones para proteccin contra exposicin al agua y gotas de antibitico tpico para las exacerbaciones. La actividad de la ciprofloxacina contra Pseudomonas puede auxiliar a secar un odo crnicamente purulento, cuando se administra en dosis de 500 mg, 2 veces al da, por 1 a 6 semanas. Bibliografa Acuin J. Chronic suppurative otitis media. Clin Evid. 2006 Jun;(15):772-87. [PubMed] Agrawal S, Husein M, MacRae D. Complications of otitis media: an evolving state. J. Otolaryngol. 2005 ;34 Suppl 1:S33-9. [PubMed] Dynamed. Otitis media with efusin . Ebsco; Updated 2008 Feb [acceso 26/2/2008]. Disponible en: http://www.ebscohost.com/dynamed/ Hannley MT, Denneny JC 3rd, Holzer SS. Use of ototopical antibiotics in treating 3 common ear diseases. Otolaryngol Head Neck Surg. 2000; 122(6):934-40 Jahn AF, Abramson M. Medical managment of chronic otitis media. Otolaryngol Clin N Am 1984; 17: 673-678.

52.- Usted tiene la sospecha de diagnstico de criptosporidiosis, el examen de laboratorio de eleccin para confirmar ste es: a) b) c) d) Coproparasitoscpico De Faust. Coproparasitoscpico De Stoll. Coproparasitoscpico Directo. Frotis De Heces Teido Con Kinyoun.

Las aves infestadas con Cryptosporidium sp. no tienen historia, signos clnicos o lesiones macroscpicas especficas Para el diagnstico de criptosporidiosis existen aproximadamente 15 tcnicas para la observacin de oocistos de Cryptosporidium, pero las ms usadas son las tcnicas de Kinyoun y Ziehl-Neelsen.

Jorge Tay Zavala, Parasitologa Mdica. Editorial: Mndez Editores, Edicin: 5 Ao: 1996. Pginas: 81-96

53.- Se observa un incremento en las infecciones de vas urinarias posterior al uso de: a) b) c) d) Preservativos. Espermicidas. Esponjas vaginales. Diafragmas.

Beck W. (1997) NMS. Obstetrics and Gynecology. USA. Williams & Wilkins. Pag 243. Existen pocos efectos secundarios al uso del diafragma. Se reporta un incremento en la frecuencia de infecciones de vas urinarias probablemente resultado de compresin uretral.

54.- Masculino de 66 aos que acude al servicio de urologa, al realizar un PSA en sangre demuestra 20 ng/L. Se realiza ecografa la cual muestra un ndulo de 2 cms, localizado en el lbulo izquierdo, en el que la biopsia demuestra carcinoma.Se realizan estudios de extensin manifiesto de metstasis a distancia, adenopatas sospechosas ni invasin de rganos vecinos. Se le realiza prostatectoma radical. Cul de los siguientes parmetros tiene importancia pronstica?:

a) La estadificacin anatmica en la pieza de reseccin y el volumen total de la glndula. b) La estadificacin anatmica en la pieza de reseccin y el score de Gleason. c) La estadificacin anatmica en la pieza de reseccin y el grado tumoral segn BloomRichardson. d) El volumen tumoral estimado en comparacin con el volumen glandular total.

Grado Gleason
Los anlisis univariantes y multivariantes de factores pronsticos en el cncer de prstata identifican el ndice de Gleason como uno de los marcadores pronsticos ms significativos, con peores resultados de supervivencia, extensin tumoral y periodo libre de enfermedad cuanto ms indiferenciado est el tumor4. La utilizacin de ndices Gleason combinados (que indican la proporcin relativa de muestras con cncer de alto grado) nos proporciona una informacin pronstica ms precisa.

Si valoramos el ndice Gleason junto con el estadio clnico haremos estimaciones pronsticas an ms acertadas35. Sin embargo, se ha encontrado que cuando el tumor es de alto grado, el pronstico ser desfavorable incluso cuando exista rgano-confinacin. El ndice Gleason ms preciso se obtiene con la pieza de prostatectoma radical. Cuando se intenta estimarlo a partir de la muestra obtenida con biopsia por puncin se comete un alto porcentaje de errores, superior incluso al 50%. Algunos estudios sugieren que el error ms frecuente ocurre cuando la biopsia por puncin sugiere un Gleason <7, en los que en muchos casos, tras analizar la pieza quirrgica se clasificar como Gleason 7.

Foco de origen
La prstata est dividida en tres partes: zona perifrica, zona transicional y zona central (ver figura 1). Los tumores de la zona transicional tienen datos de mejor pronstico (malignidad, extensin del tumor, supervivencia libre de recidiva bioqumica) que los de la zona perifrica.

3.1.4. Multifocalidad
Un alto porcentaje (67%) de cnceres prostticos tienen varios focos de origen, que pueden tener distintos grados histolgicos (heterogeneidad). La existencia de multifocalidad se asocia con mayores tasas de recurrencia, y con un grado y un estadio ms avanzados.

3.1.5. Extensin extracapsular


Es un indicador de mal pronstico, con mayores tasas de fallo bioqumico y progresin de la enfermedad. Esta relacin desfavorable aumenta cuando existe un mayor nivel de invasin y penetracin de la cpsula por parte del tumor. Algunos autores creen que la importancia pronstica de la extensin extracapsular se debe a su asociacin con otras variables, como el volumen tumoral o la infiltracin de vesculas seminales, pero otros encuentran peores resultados en los pacientes con penetracin capsular, con independencia de las posibles variables loco-regionales asociadas.

3.1.6. Invasin de vesculas seminales


Es un factor de mal pronstico, asociado a mayores tasas de progresin de la enfermedad y de fallo bioqumico. Este mayor riesgo de resultados desfavorables se debe a su asociacin con otros marcadores de mal pronstico, como el ndice Gleason, extensin extracapsular, el volumen tumoral, mrgenes quirrgicos positivos o PSA preoperatorio. Puede que el significado pronstico de la invasin de vesculas seminales no sea constante, y que dependa de la zona de las vesculas que se ve afectada: si la invasin es en la porcin distal el pronstico es peor que cuando ocurre en la zona proximal.

3.1.7. Mrgenes quirrgicos positivos


Algunos estudios han encontrado que son un factor predictor de mayor riesgo de progresin de la enfermedad o fallo bioqumico. Aunque para algunos autores este efecto de los mrgenes quirrgicos positivos se debe a su asociacin con otras variables que empeoran el pronstico, como invasin de vesculas seminales, extensin extracapsular, PSA preoperatorio, grado Gleason o volumen tumoral, otros han encontrado significacin pronstica de forma independiente.

3.1.8. Volumen tumoral


Un mayor volumen tumoral en la pieza de prostatectoma se asocia a mayor riesgo de progresin de la enfermedad y de fallo bioqumico. Sin embargo, diversos estudios han encontrado que este efecto desfavorable se debe a su asociacin con varios factores pronsticos como la existencia de penetracin capsular, mrgenes quirrgicos positivos, invasin de vesculas seminales o un grado Gleason avanzado.

Bibliografa
1. 22. Wallen MJ, Linja M, Kaartinen K, Schleutker J, Visakorpi T. Androgen receptor gene mutations in hormone-refractory prostate cancer. J Pathol. 1999;189(4):55963. 2. 23. Segawa N, Mori I, Utsunomiya H, Nakamura M, Nakamura Y, Shan L, et al. Prognostic signifi cance of neuroendocrine differentiation, proliferation activity and androgen receptor expression in prostate cancer. Pathol Int. 2001;51(6):452-9. 3. 27. Miyoshi Y, Ishiguro H, Uemura H, Fujinami K, Miyamoto H, Miyoshi Y, et al. Expression of AR associated protein 55 (ARA55) and androgen receptor in prostate cancer. Prostate. 2003;56(4):280-6. 4. 25. Culig Z, Hobisch A, Cronauer MV, Radmayr C, Trapman J, Hittmair A, et al. Androgen receptor activation in prostatic tumor cell lines by insulin-like growth factor-I, keratinocyte growth factor, and epidermal growth factor. Cancer Res. 1994;54(20):5474-8. 5. 26. Sadi MV, Barrack ER. Image analysis of androgen receptor immunostaining in metastatic prostate cancer. Heterogeneity as a predictor of response to hormonal therapy. Cancer. 1993;71(8):2574-80.

55.- Una mujer de 27 aos G4 P0 con 6 SDG acude a consulta para su primera visita prenatal. Su historia obsttrica pasada es importante porque tiene tres prdidas en segundo trimestres. Refiere que en las tres ocasiones al presentarse al hospital presentaba dilacin cervical completa. No recuerda haber tenido contracciones dolorosas. Niega antecedentes mdicos y quirrgicos. El examen fsico es normal incluyendo un examen plvico que muestra un cervix largo y cerrado. Despus de una larga discusin con la paciente ella pide que se le practique un cerclaje durante este embarazo. Cual de los siguientes es el momento ms apropiado para realizarlo.

a) b) c) c)

Inmediatamente. 10 a 14 semana. 24 a 28 semanas. 32 a 36 semanas.

La incompetencia cervical se define como la incapacidad de llevar un embarazo a trmino debido a anormalidades funcionales o estructurales del cuello uterino. Esta patologa fue descrita por primera vez en 1950 por Lash y Lash (1), fecha en que comenzaron las primeras correcciones quirrgicas para resolverla. Tiene una incidencia entre 0,1 a 2% de todos los embarazos, manifestndose clnicamente por abortos repetidos del segundo trimestre o por dilatacin cervical indolora con parto prematuro. En la mayora de los casos, la etiologa es desconocida.

Cerclaje profilctico: El cerclaje profilctico generalmente se realiza basado en los antecedentes obsttricos positivos alrededor de la semana 10 a la 14 de gestacin, antes de que la dilatacin, reblandecimiento o acortamiento del cervix se lleven al cabo y antes de cualquier sangrado o contracciones. Manual de embarazo y parto de alto riesgo. Escrito por Elizabeth S. Gilbert,Judith S. Harmon.

56.- Una paciente de 22 aos G5P2A3 tiene una historia de abuso de sustancias prenatal. A las 37 sdg tiene una parto vaginal obtenindose una neonato pequeo para la edad gestacional, de gnero masculino con pqueas aperturas palpebrales, pliegues epicnticos, con cara aplanada, filtrum hipoplsico y borde del vermillon delgado. Estos hallazgos son caractersticos en neonatos cuyas madres tuvieron abuso prenatal de cual de las siguientes sustancias. a) b) c) d) Tabaquismo. Alcohol. Marihuana. Narcticos.

Criterios diagnsticos de los FASD: Desde Jacqueline Rouquette1 en adelante, el diagnstico clnico del FAS reposa en una trada de elementos semiolgicos que constituyen el conjunto nuclear de las manifestaciones del sndrome. Estas manifestaciones son: a) el retraso del crecimiento intrauterino y extrauterino; b) un dismorfismo facial caracterstico; y c) anomalas morfofuncionales del neurodesarrollo en el SNC.

La cara de los nios afectados tienen caractersticas particulares durante la primera infancia: frente pequea asociada a (y consecuencia de) la microcefalia; la raz nasal es aplanada o incluso presenta un aspecto en silla de montar similar al de la sfilis congnita; las alas de la nariz son pequeas; los ojos son diminutos (microftalmia) y estn muy prximos entre s, las hendiduras palpebrales son ms cortas que lo normal y el prpado superior frecuentemente est en ptosis; a veces, puede haber un pliegue epicanto o epicanto invertido o blefarofimosis; la regin media de la cara es pequea; el labio superior es caractersticamente fino, delgado, recto y est como arremangado hacia adentro, mostrando muy poco el bermelln (a veces puede haber labio leporino con paladar hendido o sin l); el surco nasolabial est notablemente aplanado o, incluso, es inexistente, por lo que la zona facial superior a la boca adquiere un aspecto como de planchado; los dientes pueden ser pequeos, hipoplsicos y con esmalte defectuoso; las orejas pueden tener implantacin baja y los pabellones estar malformados; al nacer suele existir un notable hirsutismo, ms marcado en la piel perifacial. Sin embargo, todas estas caractersticas faciales, que expresan defectos en el desarrollo del mesodermo facial, se suavizan con el crecimiento y pueden desaparecer, de modo que en la adultez ya no es posible distinguir a los nacidos con FAS slo por el aspecto facial. La maduracin del macizo facial se completa lentamente.

Arch Argent Pediatr 2010; 108(1):61-67 / 61 Criterios diagnsticos del sndrome alcohlico fetal y los trastornos del espectro del alcoholismo fetal Diagnostic criteria for fetal alcohol syndrome and fetal alcohol

spectrum disorders.

57.- Cul, entre las siguientes, es la prueba ms especfica para el diagnstico de miastenia grave? a) Electromiograma de fibra muscular aislada. b) Determinacin de anticuerpos anti-receptor de acetilcolina. c) TAC torcico. d) Electromiograma con estimulacin repetitiva.

La miastenia gravis es una enfermedad autoinmune que se caracteriza por presentar debilidad muscular fluctuante y fatiga de distintos grupos musculares. La miastenia gravis afecta a individuos de todas las edades, con una predileccin por mujeres entre los 20 y 40 aos. Los msculos oculares, faciales y bulbares son los ms frecuentemente afectados por la enfermedad. Los pacientes con miastenia gravis manifiestan empeoramiento de la debilidad muscular, con infecciones intercurrentes, fiebre y agotamiento fsico o emocional. La infeccin respiratoria (bacteriana o vrica) es la causa ms frecuente de provocacin. La presencia de anticuerpos contra receptores de acetilcolina en un paciente con manifestaciones clnicas compatibles con la miastenia gravis confirma el diagnstico.

El tratamiento de esta entidad es controvertido y debe ser individualizado, ya que no existe un rgimen teraputico uniforme para todos los pacientes. Entre los tratamientos disponibles testacan los frmacos anticolinestersicos, los corticosteroides, la plasmafresis, la inmunoglobulina, los inmunosupresores y la timectoma.

REFERENCIAS 1. Drachman DB. Myasthenia gravis. N Engl J Med 1994;330:1797-8102.2. Engel AG. Myasthenic syndromes. In Engel AG, Franzini-Armstrong C, eds. Myology. 2 ed. New York: McGraw-Hill;1994. P. 1798-835. 3. Engel AG. Ohno K. Milone M, Sine SM. Congenital myasthenic syndromes caused by mutations in acetylcholine receptor genes. Neurology 1997;48 (Suppl 5):S28-35. 4. Steinman L, Mantegazza R. Prospects for specific inmunotherapy in myasthenia gravis. FASEB J 1990;4:2726-31. 5. Massey JM. Treatment of acquired myasthenia gravis. Neurology 1997;48 (suppl 5):S4651.

58.- Ocasiona rechazo al alimento, irritabilidad, llanto constante, inquietud, delirio, convulsiones y estado epilptico. a) Capuln tullidor. b) T de ans estrella. c) Papaver somniferum. d) Picadura de alacrn.

La intoxicacin se presenta en forma crnica. Ms frecuente en recin nacidos y lactantes. La sintomatologa por intoxicacin de t de ans de estrella se caracteriza por el siguiente cuadro clnico: Gran irritabilidad Llanto constante Inquietud Delirio Convulsiones Rechazo al alimento Estado epilptico

El tratamiento consiste en: Estabilizar signos vitales Ayuno Soluciones parenterales Anticonvulsivos: Benzodiacepinas barbitricos Estabilizar signos vitales Ayuno Soluciones parenterales Anticonvulsivos: Benzodiacepinas barbitricos 1.- Montoya-Cabrera MA. Intoxicaciones y envenenamientos en nios. Mxico, Intersistemas , 2000. 2.-Montoya CMA. Toxicologa clnica. 2. Ed, Mxico, Mndez Editores, 1997.

59.- Una mujer de 40 aos con DM acude a consulta quejndose de descarga vaginal prurtica y blanquecina, cual de los siguientes exmenes de diagnsticos es mas til para identificar el patgeno? a) b) c) d) Wet prep. Tincion de Gram. PH. KOH.

Prueba del KOH es un procedimiento en el cual hidrxido del potasio (KOH) se utiliza detectar hongos disolviendo las clulas humanas en una cultura. La diferencia en la composicin de la pared de clula de clulas humanas y de clulas fungicidas permite que este procedimiento ayude a distinguir las dos clulas. El KOH desnaturaliza las protenas en la clula humana; solamente sigue habiendo las clulas fungicidas ser considerado debajo del microscopio. 1. Bernal B. Fisiologa y ecologa de la vagina. Rev Chil Obstet Ginecol 1986; 51:56-60. 2. Ibrcena E. Vaginosis bacteriana; diagnstico y prevalencia. XII Congreso Peruano de Obstetricia y Ginecologa 1996; 204-6. 3. Scapini JC, Guzmn CA. Deteccin de bacilos Gram negativos curvos anaerobios en pacientes con vaginosis. Obstet Ginecol Latinoam 1986; 44: 320-5. 4. Soihet S. El flujo vaginal en la consulta ginecolgica. Ginecol Obstet (Per) 1986; 30: 5060.

5. Linaldi CAS, Urbina JR, Castaeda JL. Vaginitis por Gardnerella vaginalis en nias y adolescentes. Bol Med Infant Mex 1988; 45: 101-3. 6. Sobel J. Vaginitis and vaginal flora. Controversies abound. Current Opin Infect Dis 1996; 9: 42-7. 7. Faro S. Leucorrea. Causas infecciosas e imbalances en el ecosistema vaginal. Tribuna Mdica. Cahners Healthcare ed 1996; 1 10.

60.- Son manifestaciones de prpura vascular aguda anafilactoide: a) b) c) d) Hematomas en tronco. Petequias en cara. Ppulas hemorrgicas en miembros plvicos. Hemortrosis en miembros torcicos.

La Prpura de Schnlein-Henoch tiene comienzo agudo o insidioso, los sntomas pueden seguir una secuencia de semanas o meses, ms de la mitad presenta febrcula y astenia. (1) Las lesiones por vasculitis provocan angioedema local, que puede preceder a la prpura, especialmente en los miembros inferiores en nios y adultos (zonas declives), esto se atribuye a la mayor presin hidrosttica que conducira al depsito de los inmunocomplejos. El edema agudo hemorrgico del lactante caracterizado por fiebre, lesiones cutneas purpricas grandes y edema, en su forma tpica afecta a lactantes con historia reciente de infeccin de vas respiratorias altas. La erupcin cutnea se inicia con la aparicin sbita de ppulas y placas edematosas y purpricas que van adquiriendo una morfologa anular en medalln, escarapela o iris, de distribucin simtrica, respetando el tronco y las mucosas. No hay participacin visceral.

BIBLIOGRAFIA 1.-Miller ML, Pachman LM. Vasculitis. Prpura de Schnlein-Henoch. En: Nelson. Tratado de Pediatra.17 ed. Madrid: Elsevier; 2005: p.826-830. 2.-Enciclopedia Mdica: Prpura de Henoch-Schnlein http://www.nlm.nih.gov/medlineplus/spanish/ency/article/000425.htm. 3.- J. Vila Cots, A. Gimnez Llort, J.A. Camacho Daz y A. Vila Santandreu. Seccin de

Nefrologa. Unidad Integrada de Pediatra. Hospital Sant Joan de Du-Hospital Clnic. Universidad de Barcelona. Espaa. Nefropata en la prpura de Schnlein-Henoch: estudio retrospectivo de los ltimos 25 aos An Pediatr (Barc). 2007; 66(3):290-3.

4.-Gussinye P, Catal M, Codina X, Corral S, Domnguez O, Puig A. Prpura de SchnleinHenoch: estudi retrospectiu dels darrers 20 anys. Pediatria Catalana. 2002; 62:109-14.

61.- En el diagnstico clnico de feocromocitoma Cual de las siguientes sntomas clsicos se presentan?

a) b) c) d)

Palpitaciones, diaforesis, hipertensin y cefalea. Palpitaciones, hipotensin postural y diarrea. Hipotensin postural, bradicardia y palpitaciones. Constipacin, bradicardia y diaforesis.

Manifestaciones clnicas: La hipertensin arterial es la ms frecuente. Un 60 % de estos tumores presenta hipertensin mantenida, y la mitad de estos pacientes presenta crisis o paroxismos de hipertension. El 30% restante presenta una elevacin de la tensin arterialunicamente durante el ataque. Un 10% son silentes (incidentaloma). La HTA suele ser grave, a veces maligna, y casi siempre resistente al tratamiento convencional. Los paroxismos o ataques hipertensivos pueden ser frecuentes o espordicos, los sntomas son similares en cada crisis y con el tiempo aumentan de intensidad, suelen ser sbitos y de duracin variable. Aparece cefalea, sudoracin profusa, palpitaciones, angustia, sensacin inminante de muerte, palidez, dolor torcico y abdominal acompaado de nuseas y vmitos. La tensin arterial se eleva hacia cifras alarmantes. Las crisis suelen estar desencadenadas por actividades que desplazan el contenido abdominal, el estrs psicolgico, el ejercicio fsico, los cambios posturales, los estornudos, las maniobras de Valsalva, algunos alimentos (queso), bebidas alcohlicas, la anestesia, las angiografas, la ciruga, frmacos, en otros casos no se puede determinar el factor desencadenante. Puede existir en algunos casos hipotensin ortosttica. Ms del 50% de los pacientes presenta intolerancia glucdica secundaria al efecto contrainsular de las catecolaminas. Suele desaparecer tras el tratamiento quirrgico y no suele precisar tratamiento con insulina.

Puede existir taquicardia sinusal, bradicardia sinusal, arritmiais supraventriculares y extrasstoles ventriculares. Como consecuencia del aumento del consumo de oxgeno por el miocardio inducido por las catecolaminas, puede aparecer angina e infarto. En el ECG aparecen alteraciones inespecficas del ST y de la onda T, ondas U prominentes, crecimiento ventricular izquierdo y bloqueos de rama.

62.- Al referirnos al concepto de estereotipia como sntoma psiquitrico nos referimos a la psicopatologa de: a) La orientacin. b) La memoria. c) La psicomotricidad. d) El pensamiento.

La estereotipia motora consiste en una o varias secuencias de movimiento de carcter reiterativo, que no configura un programa de accin especfico.

Est presente en trastornos psiquitricos y neurolgicos. Puede ser secundaria a frmacos antipsicticos (diskinesia tarda o al uso abusivo de sustancias psicoestimulantes como la cocana o anfetamina (punding). Cuando se trata de la reiteracin insistente de pensamientos, acompaada de una viscosidad en el desplazamiento atencional y rigidez ideativa, se constituye la forma clnica cognitiva, que puede o no acompaar a la motora. Esta perseveracin cognitiva es un signo deficitario o negativo del propio cuadro clnico de la esquizofrenia. Sin embargo el uso al largo plazo de agentes antipsicticos tpicos puede generar el sndrome deficitario por neurolpticos que incrementa el fenmeno de la perseverancia cognitiva y causa un dficit motor simultneo.

En el trastorno obsesivo compulsivola perseveracin cognitiva se presenta bajo la forma compulsiva de reiteracin de pensamientos, que puede generar rituales motores. El trastorno de Tourette tambin cursa con estereotipias, ya sean los tics motores o vocales. La enfermedad de Parkinson o la corea de Huntington tambin pueden presentar diskinesia como expresin de eventos de neuroplasticidad. En el Parkinson pueden presentarse en el curso del tratamiento con L-Dopa, y en el Huntington forman parte del cuadro clnico coreo-atetsico. Las estereotipias motoras y cognitivas pueden ser desencadenadas por aumento del tono dopaminrgico en el estriado, y pueden ser neutralizadas por agentes bloqueantes dopaminrgicos que actan en el estriado. Asimismo pueden ser generadas por la disminucin del tono dopaminrgico en el estriado.

Ejemplos de la estimulacin dopaminrgica en el estriado lo constituyen sustancias de abuso como la anfetamina o cocana, frmacos como L-dopa, apomorfina, enfermedades como la corea de Huntington y el T. de Tourette.

Ejemplos de bloqueo dopaminrgico en el estriado lo constituyen los neurolpticos tpicos usados al largo plazo que son capaces de generar diskinesia tarda.

-American Psychiatric Association. Diagnostic and Statistical Manual of Mental Disorders. 4th ed. Washington, DC: American Psychiatric Association; 1994. -Albert, M.S., Levkoff, S.E. & Reilly, C.H. (1992): The delirium symptom interview. An interview for the detection of delirium in hospitalized patients. Journal of Geriatric Psychiatry and Neurology, 5, 14-21. - American Psychiatric Association (2002). Diagnostic and statistical manual of mental disorders (DSM-IV-TR). Washington, D.C.: American Psychiatric Association. - Belsky J:K. (1996): Psicologa del envejecimiento: Teora, investigaciones e intervenciones. MASSON, S.A. - Berg R, Franzen M., Wedding D. (1990): Exploracin del deterioro orgnico cerebral. Biblioteca de Psicologa. Editorial Desclee de Brouwer, S.A., Bilbao. - Berggren D, Gustafson Y, Eriksson B et al. (1987): Postoperative confusion after anesthesia in the elderly patients with femoral neck fractures. Anesth Analg; 66: 497-504 - Breitbat W, Rosenfeld B, Roth F et al. (1997): The memorial delirium assessment scale. J Pain Symptom Manage; 13: 128-137.

63.- Motivo por el cual se administran progestgenos en los ltimos 10 a 13 das de un rgimen con estrgenos: a) b) c) d) Disminuir el riesgo de hiperplasia y neoplasia del endometrio. Disminuir el incremento de peso. Disminuir la mastalgia. Disminuir el riesgo de neoplasia mamaria maligna.

Berek J. (2002) Ginecologa de NOVAK. Mxico. Ed. Mc Graw Hill Interamericana. Pag 303 Las mujeres que estn recibiendo tratamiento de restitucin hormonal durante la menopausia pueden recibir estrgenos los primeros 25 das de cada mes. Se aade un progestgeno a menudo acetato de medroxiprogesterona, durante los 10 a 13 ltimos das

de este rgimen en un esfuerzo por reducir el riesgo de hiperplasia y neoplasia del endometrio.

64.- El factor predisponente ms importante en la mucormicosis (zigomicosis) es: a) Cetoacidosis diabtica. b) Acidosis metablica. c) Desnutricin. d) Uso de esteroides.

La mucormicosis es una infeccin infrecuente causada por hongos oportunistas que pertenecen al orden Mucorales, familia Mucoraceae, cuyas caractersticas son invasin vascular por hifas, lo que determina trombosis e infartos. En general afecta pacientes con enfermedades subyacentes y produce infecciones severas invasoras y a menudo fulminantes. Algunas condiciones de riesgo para contraer la mucormicosis son: cetoacidosis diabtica, terapia inmunosupresora, leucemia y linfomas con neutropenia prolongada. Se describen varias presentaciones clnicas, segn su localizacin anatmica, siendo la forma rinocerebral la ms frecuente, especialmente en pacientes diabticos descompensados. Aunque esta infeccin tiene una alta morbimortalidad, su pronstico ha mejorado los ltimos aos, en base a medidas terapeuticas como la oportuna correccin de los factores predisponentes, uso de anfotericina B y una ciruga precoz y agresiva. Es muy importante sospechar esta infeccin en pacientes con factores predisponentes de manera de hacer un diagnstico precoz. (AU).

1. Bjar V, Campomanes L. Manual de prctica de micologa. Lima, Universidad Nacional Mayor de San Marcos 1994.

2. Fernndez Guerrero. Casos clnicos: Enfermedades infecciosas. Barcelona, Espaa, Salvat 1991. 3. Sastre J, Prez 0 et al. Mucormicosis pulmonar en pacientes leucmicos. Sangre 1994; 39 (1). Barcelona. 4. Levy E, Bia MJ, Isolated renal mucormycosis: case report and review. J Am Soc Nephrol 1995; 5 (12): 2014. 5. Baraia J, Muoz P. Cutaneous mucormycosis in a heart transplant patient associated with a peripheral catheter, Eur J Clin Microbiol Infect Dis, sep. 1995; 14 (9); 813-815. 6. Williams JC, Schned AR. Fatal genitourinary mucormycosis in a patient with undiagnosed diabetes. Clin Infect Dis, sep. 1995; 21 (3): 682-684. 7. Sugar A. Mucormycosis. Clin Infect Dis 1992; 220 (14): 5126.

8. Mandell D, Bennett's. Principles and practice of infectious diseases, cuarta edici6n. New York 1995. 9. Singh N, G. Invasive gastrointestinal zygomycosis in a liver recipient: case report and review of zygomycosis in solid organ transplant recipients. Clin Infect Disl mar. 1995: 617620

65.- Doce das despus de una infeccin de vas areas altas un nio de 12 aos de edad desarrolla debilidad de miembros inferiores , la cual es progresiva en das hasta afectar el tronco, a la exploracin fsica se encuentra arreflexia atrofia muscular y dolor en miembros inferiores. El LCR solo muestra proteinuria, el diagnostico ms probable es:

a) Parlisis de Bell. b) Sx. de Guillian Barre. c) Enfermedad de Charcot Marie Tooth. d) Enfermedad de Werdning Hoffman.

Resumen: El sndrome de Guillain-Barr es una serie heterognea de neuropatas perifricas mediadas inmunolgicamente.

El hallazgo comn es la polirradiculopata de evolucin rpida, que se manifiesta posteriormente a un suceso disparador, que con frecuencia es un proceso infeccioso. Se manifiesta generalmente con una parlisis motora simtrica con o sin afectacin sensorial y autonmica. Su diagnstico se basa en el examen electrodiagnstico. El tratamiento consta de un tratamiento se soporte y uno especfico. Si los pacientes superan la fase aguda, la mayora recuperar la funcin. Sin embargo, la neuropata puede avanzar tan rpido que ser necesaria la intubacin endotraqueal y la ventilacin mecnica en las 24 horas posteriores al inicio del proceso El tratamiento especfico se basa en la inmunoglobulina intravenosa y la plasmafresis.

Estudios diagnsticos Las pruebas de laboratorio son de poca ayuda al inicio del proceso. Despus de la primera semana de aparicin de los sntomas, el estudio del lquido cefalorraqudeo (LCR) detecta tpicamente elevacin de las protena (mayor de 50 mL/dL), mxima entre la segund y la cuarta semanas, con escasas clulas (menor de 10 clulas mononucleares/dL.. Algunos pacientes presentan anticuerpos antiganglisidos especialmente GM1 y GM1b.

Los estudios electrofisiolgicos son los exmenes ms sensibles y especficos para el diagnstico con este estudio se demostr una gran variedad de anomalas que indican desmielinizacin multifocal, la cual incluye: 1. Velocidad de conduccin nerviosa enlentecida. 2. Bloqueo parcial de la conduccin motora. 3. Dispersin temporal anormal. 4. Latencias distales prolongadas. Revista de Posgrado de la VIa Ctedra de Medicina. N 168 Abril 2007.

BIBLIOGRAFIA 1. Valls JS, Casademont JP, Berciano BD. Enfermedades de los nervios perifricos. En: Farreras VP, Rozman. CR y col. Medicina Interna. 14 ed. Madrid: Ed. Harcourt SA; 2000: Vol 2: 1753-70. 2. Newswanger DL. Guillain-Barr Syndrome. Am Fam Physician [online] mayo 2004 [fecha de acceso 15 de febrero 2007]; 69(10) URL. 3. Melano Carranza E, Carrillo Maravilla E, Gulias Herrero A y col. Sndrome de GuillainBarr en el anciano: un estudio retrospectivo. Arch. Neurocien. (Mex DF)] junio 2004.

66.- Las espondilitis anquilosante se puede acompaar de las siguientes manifestaciones:

a) b) c) d)

Uveitis e insuficiencia pulmonar. Uveitis e insuficiencia artica. Escleromalacia e insuficiencia mitral. Epiescleritis y estenosis artica.

ESPONDILITIS ANQUILOSANTE La caracterstica principal de la espondilitis anquilosante (EA) es el desarrollo de artritis y entesitis en la columna vertebral, articulaciones sacroliacas y de las perifricas, las extremidades inferiores. Las manifestaciones articulares usuales son la uvetis, la enfermedad inflamatoria intestinal, la aortitis e insuficiencia artica, miocarditis, defectos de conduccin y la fibrosis pulmonar. La EA se asocia fuertemente con el HLA-B27.

EspondiloartropatasPacMG-1. Parte D Libro 5 Pags. 20 a 23.

67.- Una primigrvida de 31 aos de edad. Con 35SDG, acude a consulta quejndose de dolor y de en los 3 primeros dedos de su mano derecha. Ha tenido estos sntomas intermitentemente las ltimas 2 semanas. Por lo dems, el curso prenatal no ha presentado complicaciones. El examen fsico y neurolgico son normales. Cual de los siguientes es el diagnstico ms probable.

a) b) c) d)

Sndrome del tunel del carpo. EVC. Convulsiones. Fractura de mueca.

Sndrome del tnel carpiano: El tnel carpiano es un conducto estrecho entre los huesos y los ligamentos de la mueca. El nervio que controla la sensacin en los dedos y en algunos msculos de la mano pasa a travs de este tnel, junto con algunos tendones de los dedos. Cuando una persona mueve o usa la mano o la mueca una y otra vez, los tendones se pueden inflamar y comprimir el nervio contra el hueso. La presin contra el nervio causa dolor y entumecimiento en la mano y los dedos. A esto se le llama sndrome del tnel carpiano o CTS por sus siglas en ingls. Los sntomas de este sndrome incluyen: Entumecimiento u hormigueo en los dedos de una o ambas manos, excepto el dedo meique. Dolor en la mueca que tambin puede afectarle los dedos y extenderse hasta el brazo. Dolor en la mano o en la mueca que con frecuencia es peor en la noche y temprano en la maana. El sndrome del tnel carpiano puede ser causado por cualquier cosa que produzca una hinchazn que afecte el nervio; por ejemplo, un quiste en el tendn, o la artritis reumatoide. Pero en la mayora de los casos, este sndrome se debe a la inflamacin producida por el uso excesivo de los tendones debido a los movimientos repetitivos de las manos y de los dedos en una posicin con la mueca doblada. El embarazo, la diabetes, las enfermedades de la tiroides y el uso de pldoras anticonceptivas aumentan el riesgo de padecer de este sndrome. Sndrome del tnel del carpo

Carpal tunnel syndrome Fisioterapia 2004;26(3):170-85.

68.- El factor predisponente ms frecuentemente asociado a casos de aspergilosis pulmonar invasiva es: a) b) c) d) Neutropenia severa. Sarcoma de Kaposi. Retinoblastoma. Uso de esteroides.

La entrada de Aspergillus spp. en el organismo se produce por inhalacin de las esporas que se encuentran en el aire, suelo o plantas, por lo que la afectacin pulmonar es la principal presentacin de la infeccin por este hongo, que puede adoptar 5 formas clnicas diferentes: aspergilosis broncopulmonar alrgica, aspergiloma, aspergilosis crnica necrotizante, traqueobronquitis invasiva (o aspergilosis bronquial invasiva) y la aspergilosis pulmonar invasiva (API)1,2. Otras presentaciones clnicas como la sinusitis, la afectacin cutnea y las formas diseminadas a partir de pulmn que pueden afectar al cerebro, hgado, bazo y tracto gastrointestinal son mucho menos frecuentes. Entre las distintas formas de presentacin de la infeccin por Aspergillus spp., que afectan a pacientes crticos, la API constituye un reto para los mdicos intensivistas, tanto por las dificultades en su diagnstico como por la escasa efectividad de los tratamientos propuestos hasta la actualidad. Los factores de riesgo de adquisicin de una infeccin por Aspergillus spp. se reflejan en la tabla 1 tomada del trabajo de Perfect el al3. De esos factores, el ms importante es la profundidad y duracin de la neutropenia. En enfermos inmunocompetentes ingresados en las Unidades de Cuidados Intensivos (UCI), los pacientes con mayor riesgo de infectarse por Aspergillus spp. son aquellos que han estado en tratamiento con glucocorticoides.

Bibliografa 1. Soubani AO, Chandrasekar PH. The clinical spectrum of pulmonary aspergillosis. Chest. 2002;121:1988-99. 2. Lumbreras C, Gavalda J. Aspergilosis invasora, formas clnicas y tratamiento. Rev Iberoam Micol. 2003; 20:79-89 3. Perfect JR, Cox GM, Lee JY, Kauffman CA, de Repentigny L, Chapman SW, et al. The impact of culture isolation of Aspergillus species: a hospital-based survey of aspergillosis. Clin Infect Dis. 2001;33:1824-33 4. Vandewoude KH, Blot SI, Benoit D, Colardyn F, Vogelaers D. Invasive aspergillosis in critically ill patients: attributable mortality and excesses in length of ICU stay and ventilator dependence. J Hosp Infect. 2004; 56:269-76 5. Azoulay E, Thiery G, Chevret S, Moreau D, Darmon M, Bergeron A, et al. The prognosis of acute respiratory failure in critically ill cancer patients. Medicine (Baltimore). 2004; 83:360-70. 6. Janssen JJ, Strack van Schijndel RJ, van der Poest Clement EH, Ossenkoppele GJ, Thijs LG, Huijgens PC. Outcome of ICU treatment in invasive aspergillosis. Intensive Care Med. 1996;22: 1315-22.

69. - Thrombocytopenia that is caused by increased platelet destruction is most closely associated with which of the following conditions?

a) b) c) d)

Combination chemotherapy. Acute leukemia. Systemic lupus erythematosus. Excessive ethanol intake. El diagnstico de LEG se hace mediante la identificacin de manifestaciones clnicas de la enfermedad acompaadas de uno o ms autoanticuerpos tpicos (Tablas 2 y 3).

Tabla Criterios para la Clasificacin del Lupus Eritematoso Generalizado Eritema Eritema Fotosensibilidad Ulceras Artritis Serositis a. pleuritis b. pericarditis Renal: c. proteinuria persistente > 0,5 g/24 h o +++ d. cilin Neurolgicos: e. convulsiones f. psicosis Hematolgicos: g. Anemia hemoltica con reticulocitosis h. Leucocitopenia < 4.000 por dos veces i. Linfocitopenia < 1.500 por dos veces j. Trombocitopenia < 100.000

malar discoide orales

10. Inmunolgicos: a) Anti-DNA elevado b) Anti-Sm c) Antifosfolpidos: Anticardiolipinas (IgG o IgM) o Anticoagulante lpico o VDRL falso positivo por 6 meses. 11. Anticuerpos antinucleares (en ausencia de lupus por drogas)

70.- Se evala a un paciente que padece esquizofrenia resistente al tratamiento, la primera medida a considerar ser:

a) Verificar que el paciente estuviera cumpliendo el tratamiento prescrito. b) Pasar al paciente a un antipsictico atpico. c) Uso de la Terapia Electro convulsiva. d) Ingresar al paciente en un hospital.

Esquizofrenia resistente al tratamiento farmacolgico. (Pharmacological treatment resistant schizophrenia.) En la prctica clnica son tres los trminos para caracterizar a los pacientes esquizofrnicos que no mejoran con medicacin antipsictica: resistencia al tratamiento, tratamiento refractario y no respondedor. La resistencia al tratamiento no es sinnima de cronicidad ni de severidad o gravedad, de tal manera que para que un paciente sea considerado resistente deben tenerse en cuenta varios puntos: a) si la esquizofrenia es primaria o secundaria; b) la naturaleza de la misma; c) si hubo o no historia de abuso de sustancias; d) cumplimiento y tolerancia de los tratamientos, y e) presencia de signos neurolgicos menores.

Los criterios mayoritariamente aceptados para definir la resistencia al tratamiento en la esquizofrenia fueron desarrollados inicialmente por Kane en 1988. Actualmente para la evaluacin de los niveles de falta de respuesta o resistencia al tratamiento se utiliza la Brief Psychiatric Rating Scale (BPRS) y la Independent Living Skills Survey (ILSS). Para alcanzar una evolucin teraputica favorable en ensayos con frmacos antipsicticos en pacientes esquizofrnicos resistentes al tratamiento se deben seguir las siguientes directrices: Identificar claramente los sntomas y utilizar frmacos en dosificacin adecuada y tiempo suficiente. Tener en cuenta que la resistencia al tratamiento puede confundirse bien con intolerancia a la medicacin, no adherencia al tratamiento, un inadecuado apoyo social o bien un tratamiento psicosocial inapropiado.

Agotar la utilizacin de frmacos solos antes que tratamientos farmacolgicos combinados. Prevenir los efectos extrapiramidales mediante una apropiada eleccin del tratamiento primario. FUENTE: ACTAS ESPAOLAS DE PSIQUIATRA. 2006 ENE;34(1):48-54.

71.- .- Se trata de paciente femenino de 6 aos de edad la cual presenta ceguera nocturna (hemeralopia) en la exploracin oftalmolgica se observa constriccin del campo visual con escotoma anular, prdida de la agudeza y electrorretinograma anmalo asi mismo arterias estrechadas Cul de las siguientes patologas es la ms probable?

a) Retinoblastoma. b) Catarata congnita o infantil. c) Retinosis pigmentaria (retinitis pigmentaria). d) Persistencia de vtreo primario.

Retinosis pigmentaria. Bilateral, de curso lento y progresivo, comienza en la edad escolar, pudiendo causar ceguera hacia los 40 aos. Es una alteracin de los bastones. Puede presentarse aislada o asociada a otras malformaciones, como la polidactilia, el sndrome de Laurence-Moon-Bield. Se hereda bajo diferentes patrones. Clnicamente presentan mala visin nocturna (hemeralopa) y escotoma anular en el campo visual .Oftalmoscpicamente se aprecian arterias estrechadas, atrofia de papila y acumulaciones de pigmento que asemejan osteocitos en la retina perifrica. Evolucionan hacia una reduccin del campo visual y finalmente ceguera. Adems presentan complicaciones adicionales como aparicin temprana de cataratas y glaucoma.

Retinosis pigmentaria

Retinosis pigmentaria, preguntas y respuestas, Universidad Miguel Hernndez de Elche, 2007. Science Daily (ed.): Retina Transplants Show Promise In Patients With Retinal Degeneration (11-7-2008).

72.- Ante un paciente con diagnstico de glaucoma el siguiente grupo de frmacos tpicos, se encuentran contraindicados:

a) Betabloqueantes no selectivos. b) Betabloqueantes selectivos beta-1. c) Adrenrgicos alfa-2-agonistas. d) Parasimpaticolticos.

http://www.bvs.sld.cu/revistas/mgi/vol16_3_00/mgi14300.htm Se anexa direccin para para revisin de artculo.

DEFINICIN: Frmacos que inhiben las acciones de la acetilcolina debidas a la activacin de receptores muscarnicos. Slo los compuestos derivados de amonio cuaternario interfieren con las acciones de acetilcolina en los receptores nicotnicos. 2. CLASIFICACIN: 2.1. Alcaloides naturales: * Atropina y Escopolamina 2.2. Derivados sintticos y semisintticos * Aminas terciarias: Ciclopentolato, Tropicamida, Pirenzepina, Homatropina * Aminas cuaternarias: bromuro de ipratropio, metilbromuro de escopolamina, metilbromuro de homatropina, bromuro de propantelina, bromuro de metantelina. 3. ALCALOIDES NATURALES: 3.1. RELACIN ESTRUCTURA-ACTIVIDAD: Son steres orgnicos formados por la combinacin de un cido aromtico (c. trpico) y una base orgnica (tropina o escopina). 3.2. MECANISMO DE ACCIN: Son antagonistas competitivos de los receptores muscarnicos. 3.3. PROPIEDADES FARMACOLGICAS: 3.3.1. SNC:

Atropina. A dosis teraputicas produce excitacin vagal leve. Si se aumenta mucho la dosis,

la excitacin central aumenta (inquietud, irritabilidad, desorientacin, alucinaciones) y puede ir seguida de depresin, colapso circulatorio, fallo respiratorio, parlisis y coma. Escopolamina. A dosis teraputicas produce depresin del SNC (somnolencia, amnesia, fatiga y sueo). Tambin produce euforia, lo que puede originar abuso. Con altas dosis aparecen efectos excitatorios. 3.3.2. OJO: Bloquean los efectos de acetilcolina sobre el msculo esfnter del iris y el msculo ciliar del cristalino, originando midriasis (dilatacin pupilar) y cicloplejia (parlisis de la acomodacin) de larga duracin. Los efectos de escopolamina (a igualdad de dosis) son mayores que para atropina. El incremento de la presin intraocular slo es importante en pacientes con glaucoma de ngulo agudo. APLICACIONES TERAPUTICAS 5.2. OFTALMOLOGA: - exploracin de cristalino y fondo de ojo - tratamiento de la queratitis e iridociclitis - algunos tipos de estrabismo Contraindicados en ancianos y pacientes con glaucoma de ngulo estrecho! REFERENCIAS BIBLIOGRFICAS 1. 2. 3. 4. 5. Vaughan D, Taylor A, Khalid FT. Glaucoma. En: Oftalmologa general. Mxico, DF: Editorial El Manual Moderno, 1991:193-203. Cuba. Ministerio de Salud Pblica. Gua teraputica. La Habana: Editorial Orbe, 1981. . Gua teraputica para la atencin primaria de salud en Cuba. La Habana: Editorial Jos Mart, 1994. Rosenstein SE. Diccionario de especialidades farmacutica. 41 ed. Mxico, DF: DEF, 1995. Vademecum Internacional: Medicom. 34 ed. Madrid: 1993.

73.- En relacin con el cncer de endometrio cul de las siguientes respuestas es la verdadera? a) La contracepcin oral durante ms de tres aos es un factor de riesgo. b) El tratamiento con tamoxifeno es un factor de riesgo. c) La multiparidad es un factor de riesgo. d) Es menos frecuente que el cncer de ovario.

El tamoxifeno es un tratamiento hormonal muy utilizado en el cncer de mama.Cuanto ms se prolonga su uso en el tiempo, mayor riesgo existe de padecer cncer de endometrio y de grados ms avanzados y peor pronstico. El tamoxifeno viene siendo en los ltimos veinte aos el tratamiento hormonal de cncer de mama precoz y del cncer de mama metastsico. A pesar de sus buenos resultados tiene como efectos secundarios principales un mayor riesgo de tromboembolismo pulmonar, trombosis venosa profunda y 2 a 7 veces mayor riesgo de cncer de endometrio. En este ltimo caso el riesgo aumenta cuanto mayor es el tiempo de uso del tamoxifeno. Aunque se sabe que el tamoxifeno aumenta el riesgo de cncer endometrial, pocos estudios han determinado la relacin entre el riesgo y pronstico del cncer de endometrio con la dosis, duracin y uso reciente del tamoxifeno. El departamento de Epidemiologa del Instituto del Cncer holands ha realizado un estudio de casos y controles nacionales que inclua a 309 mujeres con cncer de endometrio tras haber padecido un cncer de mama y 860 con cncer de mama pero sin cncer endometrial, todas tratadas con tamoxifeno. Como conclusin de este estudio se confirma que cuanto ms prolongado es el tratamiento con tamoxifeno mayor es el riesgo de cncer de endometrio y peor pronstico tiene ya que se dan estados ms avanzados del tumor. En el caso de mujeres con cncer de mama compensa el uso del tamoxifeno , que mejora de forma importante la supervivencia y reduce la incidencia del cncer en la mama contralateral, frente al aumento del riesgo del cncer endometrial. No ocurre lo mismo con las mujeres sanas ,en ellas no est muy claro que sea beneficioso el uso del tamoxifeno como prevencin del cncer de mama por los riesgos que conlleva el tratamiento. FUENTE: " Risk and prognosis of endometrial cancer after tamoxifen for breast cancer" Liesbeth Bergman, Maureen L R Beelen, Maarten PW Gallee, Harry Hollema, Jantien Benraadt, et al. THE LANCET. Vol 356. September 9, 2000.

74.- En relacin con la gonadotropina corinica humana durante el embarazo cual de las siguientes considera correcta? a) Tiene un mximo alrededor de la 10 semana. b) Tiene su valor mnimo al final del embarazo. c) Comienza a aumentar de forma evidente a partir de la 30 semana. d) Aumenta hasta la 10 semana y luego se mantiene constante hasta el parto.

La gonadotropina corinica humana es una glucoprotena sintetizada en las clulas del sincitiotrofoblasto de la placenta, formada por dos cadenas una & y otra . Aumenta en sangre y orina poco tiempo despus de la implantacin y sirve para pruebas de embarazo. Actualmente se la denomina hCG por las siglas de su nombre en ingls: Human Chorionic Gonadotropin. La hormona tiene como objetivo mantener la funcionalidad del cuerpo lteo como ente endcrino en la secrecin de progesterona durante el primer trimestre de embarazo. La hCG es la base histrica y actual del diagnstico de embarazos, su diferenciacin de los falsos embarazos que pueden constituirse en tumores as como de los cnceres de prstata, molahidatiforme y cariocarcinoma. Los valores esperados para este procedimiento en el periodo de gestacin son los siguientes: SEMANAS DE EMBARAZO 3 4 4 5 5 -6 6 7 7 12 12 16 16 29 29 41 RANGO mIU/ml. 9 -130 75 2,600 850 20,800 4,000 100,200 11,500 289,000 18,300 137,000 1,400 53,000 940 60,000

Reference values in adults. MeSH: HEMATOLOGIC TESTS, REFERENCE VALUES, LABORATORY TEST, CLINICAL
CHEMISTRY TESTS.

75.- Un estudiante universitario de 20 aos de edad acude a consulta debido a tos seca, fiebre, cefalea y dolor muscular durante las ltimas 2 semanas. Refiere que sus compaeros de casa han desarrollado sntomas similares. Niega el uso de drogas ilcitas y no es homosexual. Su temperatura es de 38.2C, FC 90lpm, FR 18x. Se auscultan murmullo vesicular bilateral. Una RX de trax muestra opacidades intersticiales multifocales. Presenta leucocitosis y un test de aglutinina fro +. Cual de los siguientes es el patgeno ms probable? a) b) c) d) Influenza virus. Bacterias anaerbicas. Mycoplasma pneumoniae. Pneumocystis carinii.

La neumona es la principal manifestacin clnica de la infeccin por este microorganismo y tambin es el sndrome mejor descrito en la infeccin por Mycoplasma. La neumona es de inicio incidioso, con sntomas constitucionales que aumentan hasta la aparicin de la tos (2 4 das despus del inicio de la enfermedad) que se convierte en el sntoma cardinal y se caracteriza por ser seca, aunque en ocasiones tambin productiva (hemoptoica, mucopurulenta o mucoide), hay dolor torcico retroesternal, sin embargo el dolor pleurtico franco es raro, la fiebre vara de 38 a 40 grados. El cuadro clnico se puede acompaar de coriza, odinofagia y otalgia (por miringitis bullosa en la semiologa torcica por lo general se limitan a la ausculturacin de estertores, roncus y sibilancias, aunque en muchas ocasiones el examen es normal, en algunos casos la consolidacin lobar caracterizada por el aumento de la matidez en la percusin, disminucin del murmullo vesicular, estertores, soplo tubrico y broncofona de la zona afectada1,2,4. Asociado a este cuadro clnico podemos encontrar mialgias, artralgias, vmito. En el 15% de los casos, erupciones cutneas (masculares, eritematosas, maculopapulares, vesiculares), y sndrome nodoso, urticaria, pitiriasis rosada, y sndrome de Steven-Johnson2. Desde el punto de vista extrapulmonar tambin puede existir compromiso del SNC (meningoencefalitis, encefalitis, Sndrome de Guillan-Barr, psicosis, ataxia, radiculopatas)1,2,4.

A nivel articular poliartritis en rodillas, codos, tobillos y hombra1,2,4, tambin se ha reportado pericarditis y miocarditis hasta en un 4,5% de los infectados1,4. La hemlisis intravascular ocurre generalmente en pacientes con ttulos altos de crioaglutininas1,4, otras complicaciones vasculares incluyen (coagulacin intravascular diseminada) CID y fenmeno de Raynaud. Tambin s e han documentado, casos de hepatitis, pancreatitis, nefritis, polimiositis, prpura trombocitopnica1,4; algunas en un marco sptico producto de la infeccin por este microorganismo. Rayos X de trax: Presenta hallazgos variables desde exmenes normales hasta infiltrados intersticiales, paracardacos, retculonodulares, mixtos lobares o multilobares o francamente condensaciones lobares basales y efuciones pleurales en un 20-25% de los casos, sin embargo es importante recordar la pobre correlacin existente entre los sntomas referidos, la clnica encontrada, los hallazgos pulmonares y al radiografa de trax. Exmenes paraclnicos de "rutina": El cuadro hemtico generalmente presenta un recuente leucocitario normal, ocasionalmente leucocitos o leucopenia leve, el diferencial es normal o con predominio de linfocitos, es frecuente la trombocitosis y aumento de la VSG (velocidad de eritrosedimentacin), anemia hemoltica en ocasiones correlacionada con niveles altos decrioagutininas2. El Coombs directo puede ser positivo, segn el nivel de crioaglutininas circulantes. El parcial de orina revela en ocasiones el albuminuria (relacionada con la fiebre), el estudio del lquido cefalorraqudeo (L.C.R.) puede revelar pleocitosis de meningoencefalitis1,4.

76.- Paciente masculino de 39 aos de edad, alcohlico conocido, se presenta al servicio de urgencias por dolor en epigastrio de 12 horas de duracin. El episodio surgi posterior a la ingesta de alcohol, seguido de nuseas, vmito y fiebre. Los hallazgos abdominales incluyen signos de irritacin peritoneal, rigidez muscular, disminucin de los ruidos peristlticos, distensin abdominal, y hallazgos rectales normales. Los hallazgos de laboratorio incluyen leucocitosis y aumento de la amilasa y lipasa sricas. Sus radiografas muestran pocas asas dilatadas.Cul puede ser el diagnstico?

a) b) c) d)

Gastroenteritis. Pancreatitis aguda. Perforacin de lcera pptica. Ruptura de una aneurisma artica.

CLINICA Dolor abdominal (95-100%) de instauracin sbita, que se intensifica rpidamente hasta alcanzar su pico de unos minutos a una hora, continuo, y se localiza en epigastrio, donde queda fijo o se irradia a uno o ambos hipocondrios, espalda, zona periumbilical y en cinturn.. A la exploracin abdominal, el dolor a la palpacin en epigastrio siempre aparece y con frecuencia se percibe sensacin de ocupacin o empastamiento a ese nivel o incluso una masa epigstrica en algunos casos, a veces con defensa pero sin claros signos de reaccin peritoneal. Los ruidos intestinales son escasos o estn ausentes. Ictericia (25-30%) rubicundez facial, relacionada esta ltima con la liberacin de histamina y otras sustancias vasoactivas. En los casos graves, el compromiso hemodinmico es evidente, con hipotensin, taquicardia o incluso shock establecido Vmitos y nuseas estn presentes 70-90% de los casos y suele haber detencin del trnsito intestinal desde el comienzo del cuadro Distensin abdominal (60-70%) Febricula (70-85%) Hematemesis (5%) Hipotensin (20-40%) Cuadros confusionales (20-35%). El dolor suele no aparecer generalmente en los ms graves, y el cuadro clnico se ve dominado por los vmitos, sntomas de manifestaciones sistmicas, shock o por la descompensacin cetoacidtica, sin causa aparente, en un diabtico. . En los casos graves tambin estn presentes la Disnea (<10%) taquipnea, los signos de insuficiencia cardiaca, la confusin mental o la tetania, todo lo cual expresa el compromiso multisistmico. Las equimosis abdominales de localizacin periumbilical (signo de Cullen) (<5%) o en los flancos (signo de Gray-Turner) (<5%) son muy caractersticas, de la pancreatitis aguda as como manifestaciones tardas de la enfermedad, generalmente en extremidades inferiores, con aspecto similar al eritema nudoso o a la paniculitis supurada. (Necrosis grasa subcutnea (<1%) IV PRUEBAS COMPLEMENTARIAS ( En Urgencias) 1-Hemograma Leucocitosis de moderada intensidad hasta encontrase reacciones leucemoides Hemoglobina y Hematocrito, encontrndose al inicio elevado y posteriormente en relacin a reposicin de lquidos, podemos encontrar niveles inferiores. 2-Bioquimica -Hiperglucemia -Hipocalcemia: Un descenso marcado es indicador de mal Pronstico -Hiperamilasemia -Hiperbilirrubinemia (en los casos graves y en obstruccin va biliar) Rx simple Abdomen - Datos para Diagnostico Pancreatitis y or otro lado nos ayuda en el diagnstico diferencial con otros procesos (perforacin vscera hueca) Rx de Trax - Datos de Derrame pleural, Atelectasia, etc...

Ecografa abdominal

- Es tcnica de eleccin para diagnstico de litiasis biliar (causante de PA) y para deteccin y seguimiento de Pseudoquistes pancreticos - No existe relacin entre los hallazgos ecogrficos y gravedad clnica - Visualiza el Pncreas en un 60 -70 % correctamente. (Interposicin de gas)

TAC - Ms sensible y especifica que la ecografa.

- Tcnica de eleccin para diagnstico de PA y Necrosis Pancretica. - No todos los pacientes con pancreatitis aguda, precisan TC, ya que la indicacin se hace en funcin de la gravedad

77.- Femenino de 38 aos con diagnstico de endometriosis, en sta patologa podemos encontrar elevacin srica de: a) b) c) d) CA-125. Tc 99. AFP. TGP.

Beck W. (1997) NMS. Obstetrics and Gynecology. USA. Williams & Wilkins. Pag 278. Se encuentra elevacin srica de CA-125 en pacientes con endometriosis, se trata de un antgeno de superficie encontrado en derivados del epitelio celmico, incluyendo el endometrio, se correlacionan sus niveles con el grado de enfermedad y la respuesta a tratamiento, pudiendo ser un marcador de recurrencia de la enfermedad.

78.- Masculino de 43 aos con diagnstico de isosporosis el tratamiento de eleccin es: a) b) c) d) Trimetoprim sulfametoxazol. Albendazol. Metronidazol. Pentamidina.

ISOSPOROSIS o ISOSPORIASIS

Isospora belli representa un problema de salud en personas inmunocomprometidas. Su

distribucin es cosmopolita, aunque es ms comn en zonas tropicales y subtropicales, entre ellas Hait, El Salvador, Brasil, Mxico, Africa tropical, Este Medio y el sudeste de Asia.

Ciclo biolgico.

Al igual que los otros coccidios, tiene un un ciclo de vida complejo. El ooquiste inmaduro (forma diagnstica), de forma oval y translcido, mide 22 - 33 m y contiene un cuerpo esfrico, el esporoblasto, el cual se divide posteriormente en dos. Una vez eliminado con las heces fecales, en ambientes adecuados se desarrollan 2 esporoquistes, cada uno con 4 esporozotos mviles (quiste maduro, forma infectante). El protozoo habita en intestino delgado. Mecanismo de transmisin: ingesta de agua y alimentos contaminados con ooquistes esporulados. Cuadro clnico.

En pacientes inmunocompetentes, generalmente se presentan cuadros diarreicos limitados; en infantes y preescolares, puede presentarse una enfermedad severa, con diarrea acuosa, sin sangre, esteatorrea, cefalea, fiebre, dolor abdominal, vmito y prdida de peso. Algunos sujetos presentan eosinofilia. Puede haber recadas, relacionadas probablemente con ooquistes extraintestinales en reposo, no susceptibles al tratamiento. Es una enfermedad oportunista asociada al inmunocompromiso (SIDA, por ejemplo), con frecuencia con caracter crnico; la diarrea, ms abundante y lquida puede conducir a deshidratacin y desequilibrio hidroelectroltico importantes, requiriendo de hospitalizacin. Los pacientes con antecedente de infeccin por Pneumocystis jiroveci y sujetos a tratamiento preventivo con trimetoprim-sulfametoxazol estarn recibiendo a un tiempo tratamiento contra la isosporosis y las recadas. Existen reportes de la enfermedad extraintestinal en pacientes con SIDA, involucrando pulmones, hgado,vas biliares, bazo, ganglios linfticos, colon. Histopatolgicamente se observan atrofia de microvellosidades, infiltrados de eosinfilos y desorganizacin del epitelio en las criptas y en la superficie.

Diagnstico de laboratorio.

belli. Esporoblasto. en fresco Tincin cida. Chiang Mai Chiang Mai University, University, Thailand Thailand I. belli. Examen

I.

I.

belli en heces fecales. Esporoblasto. Tincin cida S.J. Upton, Kansas University

Se fundamenta en la observacin microscpica de los ooquistes no esporulados; su eliminacin con la materia fecal es irregular y en poca cantidad, por lo que son necesarios varios exmenes. Las preparaciones en fresco son tiles en la visualizacin de los ooquistes cuando se emplean contraste diferencial, epifluorescencia (deteccin de autofluorescencia). Son de mayor utilidad los mtodos de concentracin y posterior tincin de los extendidos. Las tcnicas empleadas son Ziehl-Neelsen modificado, safranina-azul de metileno, auramina-rodamina (costoso). Mtodos tales como el aspirado duodenal, Enterotest y la biopsia intestinal son tiles tambin, cuando los exmenes coproparasitoscpicos son negativos y existe evidencia clnica suficiente para ameritar su empleo. Tratamiento.

Trimetoprim-Sulfametoxazol y tratamiento de sostn. En patientes con SIDA y otras patologas que producen inmunosupresin, el tratamiento se contina en las recadas con Trimetoprim-Sulfametoxazol, 2-3d/ semana/3 semanas. Es muy importante el tratamiento antirretroviral.

Isosporiasis. eMedicine. Feb 6, 2008. Oliveira-Silva MB, et al. Seasonal profile and level of CD4+ lymphocytes in the occurrence of cryptosporidiosis and cystoisosporidiosis in HIV/AIDS patients in the Tringulo Mineiro region, Brazil. Rev. Soc. Bras. Med. Trop. [serial on the Internet]. 2007 Oct [cited 2008 Feb 10]; 40(5):512-515. Isosporiasis. DPDx. Bennett JA, Krafczyk T, Thomas JG. Canfield PM. Case study. A 36-Pound Weight Loss and Debilitating Painful Rash in a 42-Year-Old Male. Labmedicine, Aug 2004;35(8):484 487.

79.- El agente causal ms frecuentemente encontrado como causa de Nocardiosis es: a) b) c) d) Nocardia Brasiliensis. Nocardia Asteroides. Nocardia Farcinica. Nocardia Otitidis Cavarum.

La nocardiosis es una enfermedad infecciosa relativamente poco comn, con una frecuencia anuale stimada entre 500 y 1000 casos por ao en los Estados Unidos de Amrica. Las dos especies ms prevalentes son Nocardia asteroides y N. brasiliensis (10) y pueden causar un amplio espectro de manifestaciones clnicas localizadas y diseminadas. El complejo N. asteroides ha sido considerado el responsable de la mayora de las infecciones invasivas severas tales como neumona y abscesos cerebrales Post-traumatismo, diseminndose frecuentemente a otros sitios en pacientes inmunocomprometidos (3, 8). En Contraste, N. brasiliensis est generalmente asociada a infecciones de piel y partes blandas, comunicndose pocos casos de infecciones disemina- das (6, 8). Aqu se describe el caso de una nia de 4 aos de edad previamente sana, que present linfadenitis necrotizante postraumtica causada por N. asteroides y que evolucion satisfactoriamente con cefalotina.

Dr. Alejandro Bonifaz, Micologa Mdica Bsica, Editorial: Mndez Cervantes, Edicin: 11991. Pginas 389-395

80.-El dolor abdominal intermitente, la anemia, la proteinuria, la necrosis tubular, la esterilidad, los trastornos de la conducta, la polineuropata perifrica se encuentran en la intoxicacin por:

a) Organoclorados. b) Sulfuro de hidrogeno. c) Plomo. d) Etilenglicol.

La intoxicacin por plomo es la ms comn de las exposiciones a metales, el cual tiene muchos usos, las fuentes ms frecuentes vienen de las minas y del reciclado de materiales conteniendo plomo. Este metal es absorbido por pulmones y del tracto gastrointestinal. El mecanismo de accin es por unin a los grupos sulfhidrilo y txico para las enzimas dependientes de zinc. Diagnstico: La toxicidad aguda se presenta luego de una exposicin respiratoria a altas concentraciones, con encefalopata, insuficiencia renal y sntomas gastrointestinales. La toxicidad crnica es la ms frecuente y se manifiesta con compromiso multisistmico: hematopoytico, del sistema nervioso, gastrointestinal, rin y sistema reproductor. (Astenia, dolor abdominal, irritabilidad, nusea, vmitos, prdida de peso, cefalea, anemia, neuropata perifrica, ribete de Burton, IRC, proteinuria, Nefritis intersticial,etc). En los exmenes auxiliares podemos encontrar anemia, punteado basfilo, aumento del cido rico, etc.

REFERENCIA BIBLIOGRFICAS 1. Keogh JP y Boyer LV. Lead en Sullivan y Krieger editores: Clinical Environmental Health and Toxic Exposures. Lippincott Williams & Wilkins, 2da edicin, 2001 2. Decreto Supremo N 019-98-MTC. Dispone eliminar del mercado la oferta de gasolina 95 RON con plomo y reducir el lmite mximo de contenido de plomo en la gasolina 84 RON. (14/07/98) 3. Shannon Michael. Lead en Haddad, Shanon y Winchester editores: Clinical Management of Poisoning and Drug Overdose. WB Saunders, 3ra edicin, 1998. 4. Krantz A, Dorevitch S. Metal exposure and common chronic diseases: A guide for the clinician. Dis Mon 2004; 50:215- 262. 5. Nogu S. Burton s Line. N Engl J Med 2006; 354:e21.

6. Rempel D. The lead-exposed worker. JAMA 1989; 262:532-4.

81.- El tratamiento de eleccin del cncer papilar de tiroides es: a) b) c) d) Tiroidectoma total y yodo radioactivo. Hemitiroidectoma y radioterapia externa. Quimioterapia y radioterapia. Yodo radioactivo.

El tratamiento para el cncer papilar usualmente incluye: Ciruga - para extirpar la glndula total o parcialmente (tiroidectoma). Terapia de hormona de la tiroides - para suprimir la secrecin por la glndula pituitaria de ms hormona estimulante de la tiroides, la cual puede estimular la recurrencia del cncer papilar. Administracin de yodo radioactivo - para destruir cualquier remanente de tejido tiroideo.

Bibliografa 1.Sherman, S.I., Thyroid carcinoma. Lancet, 2003. 361(9356): p. 501-11. 2.Jemal, A., et al., Cancer statistics, 2004. CA Cancer .J Clin, 2004. 54(1): p.8-29. 3.Fardella, C., Jimenez M, Gonzlez H, Len A, Goi I, et al. Caractersticas de presentacin del microcarcinoma papilar del tiroides: Experiencia retrospectiva de los ltimos 12 aos. Rev. Md. Chile, 2005. 133: p. 1305- 1310. 4.Mosso L, Jimenez M, Gonzlez H, Solar A, Torres J, Fardella C. Microcarcinoma tiroideo de evolucin agresiva. Rev. Md. Chile. 133: 232- 326, 2005.

82.- Cual de las siguientes es una aseveracin cierta en relacin al osteosarcoma:

a) b)

Es muy raro que produzca metastasis. Lo ms comn es que aparezca en la metafisis distal del fmur y proximal de la tibia. c) Es mas frecuente en mujeres en la segunda y tercera decada de la vida. d) Se ha asociado a traumatismos.

El osteosarcoma se define como un tumor mesenquimal maligno en el que las clulas cancerosas producen matriz sea. Es el tumor seo maligno primario ms comn, a excepcin del mieloma y el linfoma, y representa el 20% de los tumores seos primarios. El osteosarcoma se produce en todos los grupos de edad, pero su distribucin es bimodal; el 75% de los casos se produce en pa-cientes menores de 20 aos. El segundo grupo ms pequeo se produce en ancianos, que frecuentemente tienen patologas que se saben estn asociadas al desarrollo de osteosarcoma. Su localizacin ms frecuente es la regin metafisiaria de los huesos largos, preferentemente en miembros inferiores (metafisis distal del fmur y proximal de la tibia). Se caracteriza por un crecimiento rapido del tumor con extensin locore-gional y difusin metastsica a pulmn; generalmente, 20% de los pacientes presentan metstasis en el momento del diagnstico

BIBLIOGRAFIA KUMAR Vinay, Abbas Abul, Fausto Nelson, Patologa funcional y estructural, MadridEspaa, Elsevier, 7ma Edicin, Pag.1297-1298. Documento Osteosarcoma. GOODMAN. Snyder. Patologa mdica para fisioterapeutas. Espaa. McGraw-Hill Interamericana. 3ra Edicin. 2002. Pag. 368. PICCI P. Sangiorgi, P. Calroda, L.. Benassi M. S y Campanacci M.Histo-patologia del osterosarcoma- Centro de Tumores seos. Instituto Ortop-dico Rizzoli, Bolonia, Italia.

83.- Cul de las siguientes aseveraciones respecto a la hipertensin renovascular es correcta? a) Se relaciona con mayor liberacin de renina. b) Se ve a menudo en jvenes como complicacin de enfermedad fibromuscular. c) Es fcil de controlar. d) Rara vez produce hipertensin grave.

Allen R. M. MMS Medicina Interna. 5. Edicin. National Medical Series. Mc. Graw Hill. 2006. (captulo 6, parte I: XIV C, parte III: II B 1). La hipertensin renovascular se relaciona con un incremento en la liberacin de renina. La disminucin del flujo sanguneo renal estimula la liberacin renal de cantidades en aumento de renina, que activan el sistema renina-angiotensina-aldosterona y producen hipertensin. El captopril (IECAS) impide la conversin de angiotensina I a angiotensina II (un vasoconstrictor que se esperara fuera de especial eficacia en esta enfermedad).

Las mujeres jvenes, mucho ms a menudo que los varones jvenes, presentan obstruccin fibromuscular de las arterias renales, lo que provoca hipertensin reno-vascular. Tpicamente sta causa hipertensin muy grave y es notablemente difcil de controlar; este hecho hace sospechar que hay hipertensin renovascular.

84.- Cul de las siguientes aseveraciones describe mejor a los pacientes con apnea obstructiva durante el sueo? a) Tienen diferentes presentaciones clnicas que los sujetos con otras formas de apnea durante el sueo. b) Responden bien al tratamiento con estimulantes respiratorios. c) Responden bien a la presin positiva continua en las vas nasales. d) No tienden a quedarse dormidos durante el da.

Allen R. M. MMS Medicina Interna. 5. Edicin. National Medical Series. Mc. Graw Hill. 2006. (captulo 2 XV B-E). Los pacientes con apnea obstructiva evolucionan bien con pre positiva continua por va nasal (CPAP), que acta como frula de la parte posterior obstruida de la faringe personas con apnea durante el sueo, ya sea central, obstructiva, o mixta, son indistinguibles desde el pi de vista clnico; se requieren estudios del sueo para averiguar qu tipo de apnea est presente. Aun cuado algunos pacientes con apnea obstructiva pueden mejorar con estimulantes respiratorios, otros requieren m das ms drsticas. Los enfermos con todas las formas de apnea tienen somnolencia durante el da y, por razones desconocidas, estn en riesgo de sufrir cor pulmonale.

85.- Contraindicacin absoluta para el remplazo hormonal en la postmenopausia: a) Hipertrigliceridemia > 400mg/dl. b) Colecistitis aguda. c) Cncer de mama y enfermedad heptica activa. d) HTAS.

sangrado vaginal de causa desconocida, enfermedad heptica activa, tromboembolismo venoso o historia de cncer endometrial. La hipertrigliceridemia y la enfermedad vesicular activa son contraindicaciones relativas.

Braunwald, Eugene et al (2006). Harrisons principles of internal medicine. New York: McGraw-Hill. Pag 2212. Son contraindicaciones absolutas para el reemplazo hormonal,

86.- Factores de riesgo en el recin nacido para hemorragia suprarrenal: a) Macrosmicos y nacidos por parto de nalgas. b) Macrosmicos y nacidos por cesrea. c) Prematuro y nacidos por parto de nalgas. d) Prematuro y nacidos por cesrea.

Manual CTO Pediatra, p.1295. La hemorragia suprarrenal afecta a productos macrosmicos nacidos por parto de nalgas y se manifiesta con anemia, hipoglucemia, trastornos electrolticos y anemia.

87.- Recin nacido de 32 semanas de gestacin, que presenta de manera sbita hipotensin, anemia y abombamiento fontanelar. A al EF presenta una FC de 100 x, una FR de 20 x y comienza a convulsionar Cul es el diagnstico ms probable? a) Hemorragia de la matriz germinal. b) Leucomalacia periventricular. c) Hidrocefalia. d) Hipotiroidismo congnito.

Manual CTO pediatra, 7 edicin p.1295 La matriz germinal es una estructura del cerebro inmaduro hasta la semana 34 de gestacin. Su lesin produce hemorragia. Se manifiesta con deterioro del estado general, aparicin de pausas de apnea, bradicardia, hipoventilacin, convulsiones, disminucin del reflejo del Moro y fontanelas a tensin. Debe sospecharse ante un paciente pretrmino que presenta de forma sbita anemia, hipotensin y abombamiento fontanelar. Puede derivar en la aparicin de leucomalacia periventricular.

88.- Cul de los siguientes es un factor de riesgo que requiere la realizacin del test de OSullivan al comienzo del embarazo? a) IMC<19. b) Gestante menor de 25 aos. c) Glucemia en ayunas >126 mg/dl en dos ocasiones. d) Infeccin de vas urinarias de repeticin.

McPhee S, Papadakis M, Tierney L. Currrent Medical diagnosis and treatment. McGraw Hill 2007 p. 803. Los factores de riesgo que requieren la realizacin del test de OSullivan son: obesidad (IMC>30), padres o hermanos con DM, hipertensin crnica, preeclampsia, candiadiasis, IVUs de repeticin, antecedentes de diabetes gestacional o intolerancia a la glucosa, y antecedentes obsttricos: abortos de repeticin, fetos muertos, muerte neonatal no explicada, prematuridad, malformaciones congnitas, hidramnios, macrosoma fetal. La deteccin de una glucemia basal > 126 mg/dl en 2 ocasiones hace el diagnstico de diabetes gestacional directamente, sin necesidad de realizar el test, al igual que dos valores al azar de 200 mg/dl. (CTO 665, 666).

89.- Paciente masculino de 52 aos con diagnstico de esquizofrenia desde hace 25 aos, tratado con diversos neurolpticos en dosis elevadas. Hace dos aos inici con movimientos repetitivos de labios y lengua, y movimientos coreoatetsicos de piernas y tronco que no han mejorado a pesar de mltiples frmacos. Cul de estos factores es predisponente de discinesia tarda?

a) Administracin prolongada de neurolpticos y anticolinrgicos. b) El diagnstico de Esquizofrenia. c) Una distimia agregada. d) Una probable neoplasia cerebral.

Kaplan H I, Sadock B J. Sinopsis de Psiquiatra. Panamericana. 8 edicin. Madrid 2000. p. 711. Los factores predisponentes son la edad avanzada, existe mayor probabilidad en pacientes mayores de 50 aos, y la administracin prolongada de neurolpticos y anticolinrgicos, que ocurre varios meses o aos despus de iniciado el tratamiento con agentes del bloqueo de la dopamina. Se cree que es por aumento de las concentraciones sanguneas circulantes de dopamina, aumento de liberacin de dopamina desde las neuronas dopaminrgicas, aumento de la capacidad de reaccin a la dopamina a nivel de sitios de receptores postsinpticos y sntesis de nuevos receptores de dopamina de bajo umbral.

90.- El tratamiento de eleccin en el sndrome de hiperinfeccin por Strogyloides es:

a) b) c) d)

Triclabendazol. Prazicuantel. Ivermectina. Metronidazol

El tratamiento de strongyloides se dirige a la erradicacin de la infeccin.Estudios recientes han demostrado un tratamiento efectivo con el uso de Ivermectina100 a 200 mcg/kg/da, 2 das. V.O. por lo que es el frmaco de eleccin en la estrongiloidosis no complicada, presentando menos efectos secundarios queel tiabendazol 50mg xkg divididos en 2 tomas v.o. x 2 das. El tratamiento en hiperinfeccin debe prolongarse de 7 a 10 das. Nelson Tratado de Pediatria Escrito por Richard E. Behrman, Robert M. Kliegman,Hal B. Jenson pag. 1160 -2004.

91.-Masculino de 39 aos diagnosticado de esclerodermia La siguiente es una manifestacin patognomnica de esta enfermedad:

a) b) c) d)

Sndrome de asa ciega Pseudoobstruccin intestinal. Divrticulos de boca ancha en el colon. Disfuncin del esfnter esofgico inferior.

Definicin: Enfermedad adquirida, rara, crnica multisistmica caracterizada por engrosamiento y fibrosis de la piel, compromiso digestivo, pulmonar y vascular ( F. De Raynaud) Epidemiologa: Baja Incidencia: Chile 4-12 pac/milln hab. En USA 19 casos/milln Relacin-mujer___hombre=__15-20:1 Mayor incidencia entre 2 y 5 decada.

CLASIFICACIN Esclerodermia sistmica difusa. Esclerodermia sistmica limitada (CREST) Eslerodermia Localizada Sd. De Sobreposicin (EMTC) Mesenquimopata Indiferenciada. Sd. Esclerodermia like. Desordenes asociados al Fascitis difusa con Eosinofilia. ambiente (quarzo,vinil clorido,mercurio,solventes) (Morfea y Esclerodermia Lineal)

Formas epidmicas (Sd. Mialgia eosinofilia y sd. Aceite toxico) Escleredema (DMID) Alteraciones gastrointestinales en esclerodermia. El compromiso visceral ms conspicuo le corresponde al aparato digestivo. El esfago perder su natural elasticidad y aparecern signos de disfuncin: pirosis, regurgitacin, dolor retroesternal o bien una sensacin de disfagia. Estos sntomas no son universales y solamente la mitad de los pacientes se percata del problema aunque la alteracin peristltica del tercio inferior del esfago se puede constatar radiolgicamente. El resto del aparato digestivo tambin se afecta: el estmago y el intestino delgado con atona, el grueso con divertculos de boca ancha. Lecturas: Esclerosis sistmica y tracto gastrointestinal. Abordaje diagnstico y teraputico Olga Lidia Vera-Lastraa Departamento de Medicina Interna. Hospital de Especialidades Antonio Fraga Mouret. Centro Mdico Nacional La Raza. IMSS. Mxico DF. Mxico.

92.- Los antimalricos se utilizan en el lupus eritematoso para tratar las manifestaciones:

a) b) c) d)

Neurolgicas. Cutneas y articulares. Cardacas. Digestivas.

Son un grupo de medicamentos (tambin llamados antipaldicos) que se han usado clsicamente para tratar el paludismo o malaria, enfermedad parasitaria que afecta a ms de 500 millones de personas en el tercer mundo. Algunos antimalricos (cloroquina e hidroxicloroquina) se han mostrado tiles en el tratamiento de algunas enfermedades reumticas (artritis reumatoide, lupus eritematoso sistmico, reumatismo palindrmico, Sindrome de Sjgren, artritis psorisica, sarcoidosis, osteoartrosis erosiva,...). Estos medicamentos tienen mltiples mecanismos de accin, algunos no bien entendidos. Tienen efecto antiinfeccioso (base del tratamiento del paludismo), antiiinflamatorio y sobre la funcin inmune inhibiendo la activacin de linfocitos, monocitos y la produccin de factores mediadores de la inflamacin. Otros efectos conocidos son su capacidad de disminuir los niveles de lpidos y de inhibir la agregacin de plaquetas, ayudando a prevenir fenmenos trombticos.

Lecturas: ANTIMALRICOS COMO DROGAS INDUCTORAS DE REMISIN EN LA ARTRITIS REUMATOIDEA. Jos Flix Restrepo Surez1, Renato Guzmn Moreno2, Antonio Iglesias Gamarra3 1. Mdico Internista_Reumatlogo. Profesor Asociado de Medicina Interna y Reumatologa. Coordinador Unidad de Reumatologa. Universidad Nacional de Colombia. 2. Internista _ Reumatolgo. Clnica Salud-Coop-Bogot.

93. - A 60-year-old woman presents to a physician complaining of a swelling in her neck. Her past medical history is significant for rheumatoid arthritis and Sjgren syndrome. Physical examination reveals a mildly nodular, firm, rubbery goiter. Total serum thyroxine (T4) is 10 mg/dL, and third-generation thyroid-stimulating hormone (TSH) testing shows a level of 1.2 mIU/mL. Antithyroid peroxidase antibody titers are high. Which of the following is the most likely diagnosis?

a) b) c) d)

Euthyroid sick syndrome. Graves disease. Hashimoto thyroiditis. Silent lymphocytic thyroiditis.

Tiroiditis de Hashimoto

Es la causa ms frecuente de hipotiroidismo en las zonas donde no existe dficit dietario de yodo y los sntomas de hipotiroidismo suelen ser el motivo de su diagnstico. Es frecuente su asociacin con otras enfermedades autoinmunes. Es caracterstica la presencia de un pequeo bocio, firme, irregular y no doloroso al tacto con sensacin de plenitud en la garganta. Los Ac anti-TPO (anticuerpos antiperoxidasa) son positivos en el 90% de pacientes y los antitiroglobulina en un 20 a 50%. La VSG es normal o casi normal. La ecografa muestra una glndula hipoecognica sin ndulos. Una vez el hipotiroidismo est establecido debe tratarse con hormona tiroidea a dosis sustitutiva: 50 microgramos diarios inicialmente (25 si edad avanzada, enfermedades cardiovasculares o consuntivas o niveles previos muy bajos de T4 libre) aumentando 25 mcrgr/ da cada 15 a 30 das segn los mismos criterios, hasta 75-100 micrgr/da, ajustndose posteriormente segn controles que deben dilatarse al menos 2-3 meses desde el inicio del tratamiento ya que las respuestas son lentas. Los pacientes con hipotiroidismo subclnico con niveles altos de Ac anti-TPO debe ser tratados tambin por su frecuente evolucin a hipotiroidismo clnico y mayor riesgo arteriosclertico por elevacin lipdica. Si el bocio es grande debe administrase hormona tiroidea en dosis

suficiente para frenar la TSH, lo que puede hacer disminuir en 6 meses hasta un 30% el tamao de la glndula.

Si se observa un ndulo tiroideo debe ser estudiado mediante PAAF a fin de descartar carcinoma tiroideo o linfoma (raro, pero con un riesgo relativo de 67 en este tipo de tiroiditis)

Servicio de Medicina Interna. Hospital la Paz. Universidad Autnoma. Madrid Sierra Santos L, Sendino Revuelta A, Pacheco Cuadros R, Aparicio Jabalquinto G, Barbado Hernndez FJ. Tiroiditis de Hashimoto y sndrome de Sjgren. Asociacin o sndrome mltiple autoinmune? An Med Interna (Madrid) 2001; 18: 86-87 HASHIMOTOS THYROIDITIS AND SJGRENS SYNDROME. ASSOCIATION OR MULTIPLE AUTOIMMUNE SYNDROME?

Lectura recomendada. Tiroiditis de Hashimoto y sndrome de Sjgren. Asociacin o sndrome mltiple autoinmune? L. SIERRA SANTOS, A. SENDINO REVUELTA, R. PACHECO CUADROS, G. APARICIO JABALQUINTO, F. J. BARBADO HERNNDE

Bibliografa Beers MH, Berkow R. Editores. El manual Merck. 10 ed. Madrid: Harcourt; 1999. p. 96-7. Fauci AS, Braunwald E, Isselbacher KJ, Wilson JD, Martn JB, Kasper DL et al. Editores. Harrison Principios de Medicina Interna. 14 ed. Madrid: McGraw Hill; 1998. p. 2310-2. Larsen PR, Davies TF. Hipotiroidismo y tiroiditis. En: Larsen PR, Kronemberg HM, Melmed S, Polonsky KS. Williams Tratado de Endocrinologa. 10 ed. Madrid: Elsevier; 2004. p. 465500. Pearce EN, Farwell AP, Braverman LE. Thyroiditis. N Engl J Med 2003; 348:2646-55

94. - An 18-year-old woman complains of myalgias, a sore throat, and painful mouth sores for 3 daysduration. Her temperature is 38.2 C (100.8 F), blood pressure is 110/80 mm Hg, pulse is 84/min, respirations are 15/min. Her gingival are edematous and erythematous, and there are vesicles on her right upper and lower lips. Her pharynx is mildly erythematous but without exudates, and there is tender mobile cervical lymphadenopathy. Her breath is not fetid, and the dentition is normal. Which of the following is the most likely causal agent?

a) Actinomyces israelii. b) Herpes simplex virus 1. c) Nocardia asteroids. d) Streptococcus pyogenes.

Las infecciones por herpes simplex son comunes en la prctica diaria, y con frecuencia el paciente acude a los servicios de urgencia. Estas infecciones son ocasionalmente recurrentes, generalmente dolorosas y asociadas con sntomas sistmicos, por lo cual el mdico de urgencias debe estar familiarizado con el cuadro clnico y su manejo. Existen dos variedades de virus del Herpes simplex (VHS) capaces de causar infeccin en el hombre: el tipo 1 (VHS-1) y el tipo 2 (VHS-2) que se distinguen entre s por varias caractersticas, incluyendo sus comportamientos clnico y epidemiolgico, antigenicidad, composicin del ADN y la sensibilidad a diferentes agentes fsicos y qumicos (Cuadro No.1) Cuadro No. 1 DIFERENCIAS ENTRE LOS VIRUS HERPES SIMPLEX TIPOS 1 Y 2 Caractersticas clnicas VHS-1 VHS-2 Genital Genita PerianaL Neonata Paroniquia l VHS-2 especfico

Va de transmisin Oral Sndromes Oral-facial caractersticos Ocular Encefalitis Paroniquia Reactividad a antgenos VHS-1 especfico monoclonales especficos

El VHS-1 es de localizacin primordialmente extragenital, con predileccin por los tejidos de origen ectodrmico, mientras que el VHS-2 corresponde al "Herpes progenitalis" descrito por separado, dentro de las infecciones de transmisin sexual. La regin oral es la localizacin habitual del Herpes simplex 1, el cual es causa frecuente de lesiones orofaciales recurrentes y de otro tipo de enfermedades (encefalitis). EPIDEMIOLOGIA El ser humano es el nico reservorio natural conocido del virus herpes simplex, aunque algunos animales de experimentacin pueden infectarse con facilidad. La infeccin primaria del VHS-1 ocurre sobre todo durante la infancia, mientras que el tipo 2 se presenta en la adolescencia y adultos jvenes activos sexualmente. Las tasas de infeccin son inversamente proporcionales al estrato socioeconmico.

El principal mecanismo de transmisin es el contacto directo con las secreciones infectadas. El VHS-1 se trasmite por saliva y el VHS-2 por va genital. Aunque los ttulos virales son ms altos cuando existen lesiones activas, tambin es frecuente la liberacin viral en infectados asintomticos. Por lo tanto, la transmisin viral puede efectuarse an en ausencia de lesiones activas. La persistencia de la infeccin y la recurrencia de las lesiones son un fenmeno frecuente tanto para el VHS-1 como para el VHS-2 y por lo comn se producen por reactivacin endgena. Los factores que la precipitan van desde la luz solar, el viento, traumatismos locales, fiebre, menstruaciones y hasta estrs emocional.

DIAGNOSTICO Cuadro Clnico. Los cuadros clnicos causados por este virus se suelen dividir en dos grupos: el debido a la infeccin primaria y el correspondiente a la infeccin recurrente. En el primer grupo se incluyen la gingivoestomatitis aguda, la vulvovaginitis aguda y la infeccin herptica del ojo, que puede llegar a queratitis. Las recurrentes se circunscriben al "Herpes labialis", queratitis, blefaritis y queratoconjuntivitis. Todos los cuadros son autolimitados, pero tanto las formas primarias como las recurrentes, se pueden complicar. Una de estas complicaciones es la Encefalitis herptica y el Eczema herpeticum. Infeccin primaria. El primer contacto clnico de infeccin por virus del herpes simple suele ser el ms grave. Los enfermos aquejan fiebre, malestar general, artralgias y por ltimo la presencia de un grupo de vesculas sobre una base eritematosa, dolorosa, inflamada y sensible. La gingivoestomatitis es la manifestacin ms comn, cuya gravedad vara desde la erosin de pequeas reas a la ulceracin extensa de la boca, lengua y encas. La infeccin puede ser bastante grave como para dificultar la ingesta de alimentos y lquidos (odinofagia). La curacin tiene lugar en 7 a 14 das, a menos que las lesiones se sobreinfecten con estafilocos o estreptococos.

Infeccin recurrente. Generalmente existe prurito, dolor o molestias focales que preceden la aparicin de las vesculas. Las vesculas se rompen espontneamente despus de unos cuantos das y sanan en una semana sin dejar secuelas. LECTURAS RECOMENDADAS 1. Callen JP, Cooper Ma. Dermatologic emergences. Emerg Med. Clin North Am 3:641, 1985 2. Guzmn M. Herpes simple, varicela zoster. En: Medicina Interna. Segunda Edicin. Editado por F Chalem, JE Escandn, J Campos, R Esguerra. Fundacin Instituto de Reumatologa e Inmunologa. Editorial Presencia Ltda. Santaf de Bogot, 1992 3. Guerra Flecha J, Lizarraga Bonelli S. Enfermedades de transmisin sexual: herpes genital Trib Med 79:29, 1989

4. Jaramillo AC. Infecciones virales de la piel y sus anexos. En: Fundamentos de Medicina. Enfermedades Infecciosas. Cuarta edicin. Corporacin para Investigaciones Biolgicas. CIB. Medelln, 1989

95.- Masculino de 48 aos de edad, presenta erupcin cutnea descamativa asintomtica, con afectacin del cuero cabelludo, cejas, pestaas, eminencias malares y pliegues nasolabiales. En la exploracin se pone de manifiesto una descamacin seborreica sobre una piel moderadamente eritematosa en las reas afectadas. En la regin anterior del trax se observa un moderado enrojecimiento con descamacin. El diagnstico ms probable es: a) Dermatitis seborreica. b) Psoriasis. c) Lupus eritematoso. d) Tia corporis.

CLNICA La dermatitis seborreica se puede clasificar en: 1) Dermatitis Seborreica en la infancia: a) Costra lctea. B) Dermatitis seborreica infantil. C) Falsa tia amiantcea. D) Eritrodermia descamativa de Leiner-Mousses. 2. Dermatitis Seborreica en el adulto: Pitiriasis cpitis. B) Pitiriasis esteatoide. C) Dermatitis seborreica facial, mediotracica y medioescapular y pbica. 3. Dermatitis

placas escamosas, untuosas al tacto, de color grisceo que localizan en cuero cabelludo (Fig. 1). Tambin pueden existir lesiones eritemato-descamativas centrofaciales y mediotorcicas. La falsa tia amiantcea se manifiesta como gruesas escamocostras en cuero cabelludo, de color blanco-grisceo que al despegarse dejan zonas hmedas. La eritrodermia descamativa de Leiner Mousses comienza de forma repentina, a los pocos meses de vida, con lesiones en cuero cabelludo, evolucionando de forma rpida a lesiones eritematosas y descamativas que ocupan casi todo el tegumento.

Seborreica en enfermos inmunocomprometidos. La costra lctea se manifiesta en los primeros meses de vida y se manifiesta mediante

Suele acompaarse de candidiasis de pliegues, infecciones locales y sistmicas, diarreas. A veces el pronstico es malo. Las manifestaciones clnicas de la dermatitis seborreica infantil o clsica del adulto slo se manifiestan en la edad de presentacin. Conforman lesiones eritemato-descamativas que localizan en la zona de insercin de cuero cabelludos, pliegues retroauriculares, conducto auditivo externo, surcos nasolabiales y nasogeniamos, cejas, glabela, zona mediotorcia e interescapular, axilas, ingles, ombligo, pubis . Suele ser pruriginosa. En pacientes HIV (+) las manifestacionesclnicas de la dermatitis seborreica son mucho ms intensas.

DERMATITIS SEBORREICA Dres. E. Herrera y G. Ruz del Portal.

96.- Mecanismo de transmisin de la escabiasis: a) Persona a persona, fomites. b) Agua contaminada, piojos. c) Carne mal cocida de animal contaminado. d) Ganado vacuno al hombre.

Definicin: Sarna o escabiosis es la enfermedad producida por la presencia en la piel del hombre del ectoparsito Sarcoptes scabiei. An cuando existen variedades de esta misma especie que atacan a los animales domsticos (var.cati, canis, suis, equis y otros) la variedad hominis es la nica que afecta al hombre en forma permanente. Mecanismos de transmisin El hombre es el principal reservorio del Sarcoptes scabiei variedad hominis. La propagacin es de persona a persona a travs del contacto directo de la piel, incluido el contacto sexual, de por lo menos 10 minutos. La transmisin mediante prendas ntimas y ropa de cama es rara y se produce si estn contaminadas con caros juveniles o adultos de personas infestadas, que han usado la ropa hasta 48 horas antes. La transmisin sexual es un mecanismo posible, pero poco frecuente 16. Las personas pueden infestarse por los caros de los animales de compaa, principalmente los perros 17. Referencias 1. Heukelbach J, Feldmeier H. Scabies. Lancet 2006; 367: 1767-74 2. Van Neste DJ. Inmunology of Scabies. Parasitology Today 1986; 2: 194-6. 3. Walton S, Currie BJ. Problems in diagnosing scabies, a global disease in human and animal populations. Clin Microbiol Rev 2007; 20: 268-79. 4. Saavedra T. Sarna y otras acarosis. Atas A. Parasitologa Mdica. 1 Edicin, Santiago de Chile. Edit Mditerrneo, 1999. Captulo 54, pgs 484-9. 5. Meinking TL, Taplin D. Advance in pediculosis, scabies and other mite infestations. Adv Dermatol 1990; 5: 131-52. 6. Chosidow O. Scabies and pediculosis. Lancet 2000; 355: 819-26. 7. Commens CA. We can get rid of scabies: new treatment available soon. Med J Aust 1994;160: 317-8. 8. Chosidow O. Scabies. N Engl J Med 2006; 354: 1718-27. 9. Huynh TH, Norman RA. Scabies and pediculosis. Dermatol Clin 2004; 22: 7-11. 10. Neira P, Arenas C, Neira G, Valenzuela N, Subercaseaux B. Infecciones parasitarias en nios de una escuela de Reaca Alto. Rev Med Valparaso 1981, 34: 56-60.

97.- Femenino de ao y medio de edad refiere la madre que inici con tos seca, rinorrea hialina, temperatura axilar de 37.8C. Por la tarde muestra tos intensa y en accesos, dolor subesternal al toser, estridor inspiratorio; al explorarle se le observa plido, con aleteo nasal, hundimiento intercostal bilateral a la inspiracin y taquicardia. Faringe hipermica roja; en trax se escuchan algunos estertores gruesos diseminados en ambos hemitorax y disminucin del murmullo vesicular. Usted sospecha de:

a) Bronquiolitis. b) Asma bronquial. c) Neumona bilateral. d) Laringotraqueobronquitis.

La laringotraqueobronquitis tambin conocida como laringitis subgltica o crup vrico, es una enfermedad infecciosa aguda de etiologa viral que afecta fundamentalmente a la laringe y a las estructuras que se localizan por debajo de las cuerdas vocales, en donde los tejidos blandos subglticos inflamados producen obstruccin de la va area en grado variable. Este padecimiento afecta con mayor frecuencia al grupo de edad comprendido entre seis meses y tres aos, con un pico de incidencia a los dos aos de edad; no hay predominio de sexo, y se presenta ms frecuentemente en otoo e invierno. CUADRO CLNICO El cuadro inicial es el de una rinofaringitis e incluye irritacin nasal, coriza, fiebre generalmente menor de 39 C, tos seca y odinofagia, con poca o nula afeccin del estado general; 24 a 48 horas despus aparece disfona, la tos se hace "crupal" (traqueal, perruna o en ladrido) y se presentan en forma gradual estridor inspiratorio creciente (estridor larngeo) y signos de dificultad respiratoria de intensidad variable. En la exploracin fsica son evidentes la disfona, el estridor larngeo que en ocasiones es audible a distancia, la disminucin del murmullo vesicular y datos de dificultad respiratoria.1-6. Forbes describe la progresin del cuadro clnico de la siguiente manera: Etapa 1: Fiebre, ronquera, tos crupal y estridor inspiratorio al molestar al paciente. Etapa 2: Estridor respiratorio continuo, participacin de msculos accesorios de la respiracin con retraccin de costillas inferiores y de tejidos blandos del cuello. Etapa 3: Signos de hipoxia e hipercapnia, inquietud, ansiedad, palidez, diaforesis y taquipnea. Etapa 4: Cianosis intermitente, cianosis permanente, paro respiratorio. Por lo general la enfermedad es leve, en pocas ocasiones progresa ms all de la etapa 1, ms del 95% reciben tratamiento ambulatorio; del 5% que requiere hospitalizacin slo del 1 al 1.5% requieren medidas de apoyo ventilatorio (intubacin endotraqueal o traqueostoma).

La duracin del padecimiento es muy variable, desde tres a siete das en casos leves, hasta siete a catorce das en casos graves 1. PAC Infecciones de Vas Respiratorias en Nios Parte B Libro 2

98.- Una mujer de 55 aos, se queja de presin plvica y una masa en la entrada vaginal, hace 3 aos tuvo su ltimo periodo menstrual. No tomo terapia de reemplazo, tiene dificultad para evacuar. Tiene una tos crnica y una historia de tabaquismo positivo a razn de 30 cajetillas ao. Ha tenido 3 partos y el ultimo bebe pes 4,500 gramos. Tiene una orina de 60 cc. Cul de los siguientes hallazgos es ms probable que encuentres en la exploracin pelvica? a) b) c) d) Rectocele. Cistocele. Enterocele. Uretrocele.

El Rectocele es una hernia de la pared anterior del Recto hacia la porcin posterior de la vagina. La incidencia real de esta deficiencia anatmica es desconocida y en muchas ocasiones es un resultado del paso del tiempo. Es un hallazgo muy frecuente del examen perineal, siendo en mltiples ocasiones asintomtico. El rectocele puede ser un hallazgo importante del sndrome de Obstruccin Defecatoria (SOD). No debe tomarse como una deficiencia anatmica nica, sino como parte importante de un problema anatomo-fisiolgico complejo. Un principio importante es la etiologa y anatomo-patologa del rectocele. Existen varias teoras sin consenso principal. La existencia, deficiencia o alteraciones del septo rectovaginal son controversias importantes. No existe una fascia visceral que separe el recto de la vagina o que forme un septo especfico. Existe frontera entre donde termina la pared anterior del recto y donde comienza la pared de la vagina, pero mltiples estudios no han encontrado un septo rectovaginal especfico. El septo puede estar formado de una pelcula casi transparente hasta una pared de consistencia fibromuscular fuerte. El rectocele es un hallazgo comn. Se presenta en el 80% de las pacientes femeninas y 13% de los masculinos en una defeco grafa (> a 1cm.) Entre mas grande es el rectocele, mayor son los sntomas asociados, Dificultad en la evacuacin, constipacin crnica, dolor rectal y perineal, sensacin de masa y en ocasiones sangrado. La necesidad de presin manual para ayudar a la evacuacin o para vaciar el rectocele es comn en ms del 50% de los pacientes. El diagnostico se hace con un simple tacto rectal e inspeccin vaginal, pero debe recordarse que rara vez es el rectocele un hallazgo aislado. La presencia cistocele, peritoneocele, enterocele u otros prolapsos perineales y problemas funcionales deben ser descartados antes de proponer la reparacin quirrgica del rectocele. Defeco grafa, Pruebas de funcin fisiolgica del piso plvico, pruebas de funcin urinaria, evaluacin del esfnter anorectal por ultrasonido y hasta la resonancia magntica han sido propuestos antes de la ciruga.

Bibliografa:

Rectocele: Pathogenesis and surgical managment. Zbar AP, Linemann A, Fritsch H,

Stapled transanal rectal resection to treat obstructed defecation caused by rectal intussusseption and rectocele. Renzi A, Izzo D, Di Sanrno (26) 21:661-667 Rectocele repair using biomaterial augmentation. Altman D, Melgren A, Zetterstrom J.
Obstet Gynecol (2005) 60(11)753-760.

Beer-Gabel M, Pescatori M. Int J Colorectal Dis. (2003) 18:369-384. Evaluation and Treatment of Women with rectocele. Cundiff GW, Fenner D, Obstetrics and Ginecology 104(6): 1403-1416

99.- La causa mas frecuente de dolor de retropi (taln) en el paciente peditrico es a) Osgood Schlater. b) Enfermedad de Khler. c) Enfermedad de Sever. d) Enfermedad de Snding Larsen.

Esta enfermedad se llama de Sever en honor al mdico que la describi por primera vez en 1912. Se denomina enfermedad de Sever, apofisitis posterior del calcneo u osteocondritis calcnea a la inflamacin e irritacin del cartlago de crecimiento del hueso calcneo que cursa con dolor localizado en el taln. Existen diversos factores que condicionan su desarrollo, tales como la edad, el exceso deportivo, la retraccin de la musculatura posterior de la pierna o un calzado inadecuado. No obstante, con la colocacin de una pieza elevadora de taln as como con reposo fsico durante 2-3 semanas y terapia antiinflamatoria, se resuelven la mayora de los casos consultados en la bibliografa. As mismo, se hace necesaria la planificacin deportiva anual de los nios para evitar la aparicin de esta patologa BIBLIOGRAFIA Lowell and Winters's Pediatric Orthopaedics Chapter 29. Vincent S. Mosca: The Foot Raymond T. Morrissy and Stuart L. Weinstein editors Fifht edition, Volume 2, pag 1206 Lippincott Williams and Wilkins, 2001.

100. - A 20-year-old man comes to the physician he has noticed blood in his urine on several occasions in the past year. Each episode of hematuria occurred in association with an upper respiratory tract infection or a flulike illness. Physical examination is unremarkable. A urine dipstick test shows mild proteinuria and microhematuria. Serum levels of electrolytes, creatinine, and blood urea nitrogen are within normal limits. Serum levels of IgA are elevated. Which of the following is the most likely diagnosis?

a) Berger disease. b) Goodpasture syndrome. c) Postinfectious glomerulonephritis. d) Wegener granulomatosis.

HEMATURIA GLOMERULAR AISLADA PERSISTENTE:

Definicin Presencia de hematuria de origen glomerular (con acantocitos, y ocasionalmente cilindros hemticos), sin otro elemento de inflamacin glomerular, es decir, sin HTA, ni edema, con proteinuria menor a 1 gm /24 hs, y funcin renal normal, estable.

En caso de hematuria persistente, aislada, las posibilidades principales es que se trate de Glomerulonefritis por IgA (lo ms frecuente), enfermedad de membrana basal fina, Sd. de Alport, (raro) o el inicio de una glomerulonefritis crnica (< probable) (15) .Tambin puede tratarse de una GN Post-Streptoccica, con persistencia de hematuria.

A continucacin se describirn brevemente: Nefropata por IgA: Enfermedad de Berger y sndrome de Shnlein-Henoch, se considern por algunos como espectro de una misma enfermedad, siendo Berger la forma limitada al rin. Es la Glomerulonefritis aguda ms frecuente. Se da entre los 15 y 35 aos, y es 3 veces ms frecuente en hombres, tiene tres patrones clnicos de presentacin: 1. Como hematuria macroscpica, 24 48 hs. posterior a cuadro respiratorio alto, asociado a dolor lumbar. Hematuria dura 2 6 das, rara vez presenta HTA o deterioro de funcin renal, y tiene proteinuria mnima. Recurre hasta en un 50%. De esta forma se presenta un 40-50%.

2. Como hematuria microscpica, con proteinuria leve, detectada solo en Sed. Orina, por rutina. 30 40 % se presenta as.

3. Como Sd. Nefrtico (HTA, Edema, Hematuria), pudiendo evolucionar algunos de estos casos como GNRP. Se presentan as un 10 %.

Generalmente cursan con proteinuria menor a 1-2 gm/24 hs,. Pocos desarrollan Sd. nefrtico (10 %) Existe un grupo de pacientes (20 40 %), que evoluciona con falla renal progresiva, llegando a IRT en 5 25 aos. Se ha detectado como factor de riesgo de esta evolucin la presencia de Sd. nefrtico o proteinuria > 1gm/24h, , edad, HTA, deterioro de la funcin renal, ausencia de hematuria macroscpica. Si no tiene ninguno de estos factores, tiene poca probabilidad de desarrollar falla renal, y no se ha demostrado que el tratamiento, en este grupo de bajo riesgo, altere la evolucin de la enfermedad. Al laboratorio, el complemento es generalmente normal, rara vez elevado. Existe aumento de IgA circulante entre un 30 50 %, sin ser especfico de esta enfermedad. Para el diagnstico, se requiere de biopsia renal, donde se identifican por inmunohistoqumica, la presencia de depsitos de IgA. Biopsia de piel carece de especificidad y de sensibilidad para diagnstico de enfermedad de Berger.

BIBLIOGRAFA 1.Jennette C, Falk R: Small Vesel Vasculitides. N Engl J Med 337:1512, 1997. 2.Kashtan, CE Alport Sndrome and thin glomerular basement Membrane disease. JAM Soc Nephrol 1998;9:1736. 3.Falk R et al: Primary glomerular disease, en Breneer & Rectors, The Kidney, 6a ed, BM Brenner (editor). Phyladelfia, Saunders, 2000. pp 1263-1349. 4.Hricik, DE et al: Glomerulonephritis. N Eng J Med 339:889,1998. 5.Antony BF. Attack rates of acute nephritis after type 49 streptoccocal infection of the skin and of the respiratory tract. J Clin Inv, 1969;48:1697. 6.Oliviera DBG. Poststreptococcal glomerulonephritis:getting to know an old enemy. Clin Rxp Immunol 1997;107:8-10. 7.LangeK, et all. Evidence for the in situ origin of poststreptococcal glomerulonephritis: glomerular localization of endpstreptosin and the clinical significance of the subsequent antibody response. Clin Nephrol 1983;19:3-10. 8.Rodriguez-Iturbe, B. Epidemic poststreptococcal glomerulonephritis, Kidney int 1984, 25:129. 9.Tejani A. Poststreptococcal glomerulonephritis: current clinical and pathologic concepts. Nephron 1990;55:15. 10. Potter EV. Twelve to seventeen year follow up of patients with poststreptococcal acute glomerulonephritis in Trinidad. N Engl J Med 1982; 307:725-9.

S-ar putea să vă placă și